SlideShare a Scribd company logo
1 of 309
MINING
NOVEL
GATE PREVIOS QUESTION PAPERS
Mining Novel is an online education platform that gives Test series.
Preparing for GATE, Coal India Subsidiaries and All Mining Exams...
2022
MINING NOVEL
Welcome To Mining Novel !!
Mining Novel is an online education platform that gives free lecture
videos by combining the most enthusiastic teachers, engaging content,
and Test series.
Preparing for GATE, Coal India Subsidiaries and All Mining
Examinations. We provide you Test series and Structured classes with a
powerful study plan that will help you prepare with confidence.
Mining Novel is an inbound store for all MINING things – Mining
sirdar/ Overman exams Preparation Tips, Strategies, Motivation,
Important Updates, Free Notes, and much more that will astonish you.
Our experts are committed to giving the best guidance for All Mining
Exam preparation by analyzing the crucial needs of the students. So,
Be a Part and begin your “Amazing journey”!!
We believe that education can change everyone’s life and we want
everyone to have the opportunity. We just want to take out a few
minutes of your everyday life, and these moments will do wonders!
Tune in and Be Inspired!!
GATE PREVIOS QUESTION PAPERS
GATE PREVIOS QUESTION PAPERS
GATE
PREVIOS QUESTION PAPERS
(2022 to 2007)
Page 1
GATE 2022 General Aptitude (GA)
Q.1 – Q.5 Carry ONE mark each.
Q.1 Mr. X speaks _________ Japanese _________ Chinese.
(A) neither / or
(B) either / nor
(C) neither / nor
(D) also / but
Q.2 A sum of money is to be distributed among P, Q, R, and S in the
proportion 5 : 2 : 4 : 3, respectively.
If R gets ₹ 1000 more than S, what is the share of Q (in ₹)?
(A) 500
(B) 1000
(C) 1500
(D) 2000
Page 2
Q.3 A trapezium has vertices marked as P, Q, R and S (in that order anticlockwise).
The side PQ is parallel to side SR.
Further, it is given that, PQ = 11 cm, QR = 4 cm, RS = 6 cm and SP = 3 cm.
What is the shortest distance between PQ and SR (in cm)?
(A) 1.80
(B) 2.40
(C) 4.20
(D) 5.76
Page 3
Q.4 The figure shows a grid formed by a collection of unit squares. The unshaded
unit square in the grid represents a hole.
What is the maximum number of squares without a “hole in the interior” that
can be formed within the 4 × 4 grid using the unit squares as building blocks?
(A) 15
(B) 20
(C) 21
(D) 26
Page 4
Q.5 An art gallery engages a security guard to ensure that the items displayed are
protected. The diagram below represents the plan of the gallery where the
boundary walls are opaque. The location the security guard posted is identified
such that all the inner space (shaded region in the plan) of the gallery is within
the line of sight of the security guard.
If the security guard does not move around the posted location and has a 360o
view, which one of the following correctly represents the set of ALL possible
locations among the locations P, Q, R and S, where the security guard can be
posted to watch over the entire inner space of the gallery.
(A) P and Q
(B) Q
(C) Q and S
(D) R and S
Page 5
Q. 6 – Q. 10 Carry TWO marks each.
Q.6 Mosquitoes pose a threat to human health. Controlling mosquitoes using
chemicals may have undesired consequences. In Florida, authorities have used
genetically modified mosquitoes to control the overall mosquito population. It
remains to be seen if this novel approach has unforeseen consequences.
Which one of the following is the correct logical inference based on the
information in the above passage?
(A) Using chemicals to kill mosquitoes is better than using genetically modified
mosquitoes because genetic engineering is dangerous
(B) Using genetically modified mosquitoes is better than using chemicals to kill
mosquitoes because they do not have any side effects
(C) Both using genetically modified mosquitoes and chemicals have undesired
consequences and can be dangerous
(D) Using chemicals to kill mosquitoes may have undesired consequences but it is
not clear if using genetically modified mosquitoes has any negative
consequence
Page 6
Q.7
Consider the following inequalities.
(i) 2𝑥 − 1 > 7
(ii) 2𝑥 − 9 < 1
Which one of the following expressions below satisfies the above two
inequalities?
(A) 𝑥 ≤ −4
(B) −4 < 𝑥 ≤ 4
(C) 4 < 𝑥 < 5
(D) 𝑥 ≥ 5
Q.8
Four points P(0, 1), Q(0, −3), R(−2, −1), and S(2, −1) represent the vertices
of a quadrilateral.
What is the area enclosed by the quadrilateral?
(A) 4
(B) 4 2
(C) 8
(D) 8√2
Page 7
Q.9 In a class of five students P, Q, R, S and T, only one student is known to have
copied in the exam. The disciplinary committee has investigated the situation
and recorded the statements from the students as given below.
Statement of P: R has copied in the exam.
Statement of Q: S has copied in the exam.
Statement of R: P did not copy in the exam.
Statement of S: Only one of us is telling the truth.
Statement of T: R is telling the truth.
The investigating team had authentic information that S never lies.
Based on the information given above, the person who has copied in the exam is
(A) R
(B) P
(C) Q
(D) T
Page 8
Q.10 Consider the following square with the four corners and the
center marked as P, Q, R, S and T respectively.
Let X, Y and Z represent the following operations:
X: rotation of the square by 180 degree with respect to the
S-Q axis.
Y: rotation of the square by 180 degree with respect to the P-R axis.
Z: rotation of the square by 90 degree clockwise with respect to the axis
perpendicular, going into the screen and passing through the point T.
Consider the following three distinct sequences of operation (which are applied
in the left to right order).
(1) XYZZ
(2) XY
(3) ZZZZ
Which one of the following statements is correct as per the information
provided above?
(A) The sequence of operations (1) and (2) are equivalent
(B) The sequence of operations (1) and (3) are equivalent
(C) The sequence of operations (2) and (3) are equivalent
(D) The sequence of operations (1), (2) and (3) are equivalent
GATE 2022 Mining Engineering (MN)
Page 9
Q.11 – Q.35 Carry ONE mark Each
Q.11 The value of lim
→
( )
is
(A) 0
(B) 1
(C) −𝑛
(D) 𝑛
Q.12 A velocity field in Cartesian coordinate system is expressed as
𝒗 = 𝑥𝚤̂ + 𝑦𝚥̂ + 𝑝(𝑧)𝑘, where 𝑝(0) = 0
If div 𝒗 = 0, 𝑝(𝑧) is
(A) 0
(B) −2𝑧
(C) 2
(D) 2𝑧
GATE 2022 Mining Engineering (MN)
Page 10
Q.13 The constant term of the Fourier coefficients of the periodic function
𝑓(𝑥) =
−𝑘, for − 𝜋 < 𝑥 < 0
𝑘, for 0 < 𝑥 < 𝜋
, 𝑓(𝑥 + 2𝜋) = 𝑓(𝑥) and 𝑘 = 𝑐𝑜𝑛𝑠𝑡𝑎𝑛𝑡
is
(A) 𝑘
(B) 2𝑘
(C) 2𝜋
(D) 0
Q.14 Two vectors x and y are shown in the figure. The projection vector of x on y is
(A) 𝒙 𝒚
𝒚 𝒚
𝒚
(B) 𝒙 × 𝒚
(C) 𝒙 × 𝒚
𝒚 𝒚
(D) 𝒙 𝒚
𝒙 𝒙
𝒙
GATE 2022 Mining Engineering (MN)
Page 11
Q.15 A deposit has the grade attribute X ∈ [0, 30] with a density function 𝑓(𝑥). For a
cut-off grade 𝑥 , the proportion of the ore in the deposit is given by
(A)
𝑓(𝑥)𝑑𝑥 − 𝑓(𝑥)𝑑𝑥
(B) 1
2
𝑓(𝑥)𝑑𝑥 − 𝑓(𝑥)𝑑𝑥
(C) 1
2
𝑓(𝑥)𝑑𝑥 + 𝑓(𝑥)𝑑𝑥
(D)
𝑓(𝑥)𝑑𝑥
Q.16 The drilling technique applicable for mineral exploration is
(A) Percussive drilling
(B) Tricone roller drilling
(C) Rotary-percussive drilling
(D) Diamond core drilling
GATE 2022 Mining Engineering (MN)
Page 12
Q.17 Match the rock with its metamorphosed form
Igneous/ Sedimentary rock Metamorphic rock
P Granite I Quartzite
Q Limestone II Gneiss
R Sandstone III Schist
S Gabbro IV Marble
(A) P-II, Q-IV, R-I, S-III
(B) P-III, Q-I, R-IV, S-II
(C) P-IV, Q-III, R-I, S-II
(D) P-II, Q-III, R-IV, S-I
Q.18 Identify the WRONG statement:
Break-even stripping ratio
(A) takes into account the maximum pit slope that is safe
(B) helps in determining the volume of the overburden
(C) presents the maximum possible mine size that is economical
(D) takes into account the life of the mine
GATE 2022 Mining Engineering (MN)
Page 13
Q.19 A square pattern of blasting is shown in the figure. For the case of simultaneous
blast, identify the zone of no fragmentation
(A) P
(B) Q
(C) R
(D) S
GATE 2022 Mining Engineering (MN)
Page 14
Q.20 In the truss shown in the figure, the force in the member BD, in kN is_____.
(A) 7
(B) 5
(C) 2
(D) 0
GATE 2022 Mining Engineering (MN)
Page 15
Q.21 The correct vertical stress profile in the case of tributary area method for pillar
design is
(A) I
(B) II
(C) III
(D) IV
GATE 2022 Mining Engineering (MN)
Page 16
Q.22 The bottom section of a stoping block has dimensions 200 m × 40 m. If the
modified RMR of rock mass is 50, the appropriate method of mining on the basis
of Laubscher’s chart in the figure is
(A) Shrinkage stoping
(B) Cut and fill
(C) Block Caving
(D) Sublevel stoping
GATE 2022 Mining Engineering (MN)
Page 17
Q.23 Match the machine with its component.
Machine Component
P Continuous miner I Flight bar
Q Jack hammer II Lemniscate link
R AFC III Loading apron
S Shield support IV Rifle bar
(A) P-III, Q-IV, R-I, S-II
(B) P-IV, Q-III, R-I, S-II
(C) P-III, Q-IV, R-II, S-I
(D) P-IV, Q-III, R-II, S-I
Q.24 Which one of the following is NOT a notifiable disease as per Indian mining
legislation?
(A) Silicosis
(B) Noise induced hearing loss
(C) Nystagmus
(D) Asbestosis
GATE 2022 Mining Engineering (MN)
Page 18
Q.25 If the ambient lapse rate is higher than the dry adiabatic lapse rate, the atmosphere
is
(A) stable
(B) neutral
(C) unstable
(D) strongly stable
Q.26 Identify the WRONG statement:
The application of controlled air recirculation in an underground work place can
(A) increase the air velocity at the work place
(B) lead to increased concentration of contaminants in the work place
(C) require the installation of an additional fan in the system
(D) lead to overall ventilation cost savings
GATE 2022 Mining Engineering (MN)
Page 19
Q.27 The correct order of pavement layers for a haul road from top to bottom is
(A) Wearing course → Base → Sub base → Sub grade
(B) Wearing course → Sub base → Base → Sub grade
(C) Wearing course → Sub grade → Sub base → Base
(D) Wearing course → Base → Sub grade → Sub base
Q.28 A mining company produces iron ore and sells to another company. Royalty to be
paid is on the basis of
(A) quantity of ore produced
(B) quantity of ore sold
(C) difference between the quantities of ore produced and sold
(D) net profit
Q.29 The cost of a screw compressor with an estimated life of 15 years is ₹21,00,000. If
the depreciation of the compressor charged, using ‘sum-of-the-years-digits’
method, at the end of 4th year is ₹2,00,000, the salvage value, in ₹ is_______.
(round off to one decimal place)
GATE 2022 Mining Engineering (MN)
Page 20
Q.30 A safety device consists of two independent critical components 𝑋 and 𝑋 . The
failure of any one or both of these components can cause an accident. The failure
probabilities of components 𝑋 and 𝑋 are 0.2 and 0.1, respectively. The probability
of occurrence of an accident is_______. (round off to two decimal places)
Q.31 In a levelling survey, a reading is taken as 2.25 m. However, along the line of sight
there is deflection of 20 cm with respect to vertical position of the staff. The correct
reading, in m is_______. (round off to two decimal places)
Q.32 Water flows through a vertical sand column of cross sectional area 4000 mm2 and
length 300 mm. For a water head of 600 mm, quantity of seepage water is
100 mm3/min. The hydraulic conductivity of the sand column, in mm/min
is_______. (round off to three decimal places)
GATE 2022 Mining Engineering (MN)
Page 21
Q.33 The modified Lauffer diagram as shown in the figure relates to roof span, RMR and
stand-up time. In a metal mine, roof span of a drive is 4 m. If the RMR of the rock
mass changes from 40 to 60, then the stand-up time increases by a factor
of________. (round off to two decimal places)
GATE 2022 Mining Engineering (MN)
Page 22
Q.34 In a friction winder, the skip accelerates to a steady speed over a time span of 15 s
from the start. The torque vs. time diagram for the winding cycle is shown in the
figure. The deceleration time in seconds is_______. (round off to one decimal place)
Q.35 At a measurement station, the air quality parameters PM2.5, NO2 and O3 have the
AQI sub-index values as 180, 96, and 84, respectively. The AQI for the station
is_______. (in integer)
GATE 2022 Mining Engineering (MN)
Page 23
Q.36 – Q.65 Carry TWO marks Each
Q.36 Match the drilling pattern with mining operation
Drilling Pattern Mining Operation
P I Drifting
Q II Shaft Sinking
R III Sublevel stoping
S IV Drop Raising
(A) P→II, Q→III, R→I, S→II
(B) P→III, Q→IV, R→I, S→II
(C) P→II, Q→I, R→IV, S→III
(D) P→III, Q→IV, R→II, S→I
GATE 2022 Mining Engineering (MN)
Page 24
Q.37 The closest match of the scatter plot between the variables X and Y with the
approximate attribute is
(A) P→I, Q→II, R→III, S→IV
(B) P→II, Q→I, R→IV, S→III
(C) P→III, Q→IV, R→I, S→II
(D) P→IV, Q→II, R→III, S→I
GATE 2022 Mining Engineering (MN)
Page 25
Q.38 A 3-point borehole extensometer is installed to identify the location of a single
discontinuity plane in a hanging wall rock by measuring deformations at three
locations as shown in the figure. The absolute readings of deformations measured
on two different dates are listed in the table. Based on the measured data the most
likely inference is
Date
Anchor -1
(mm)
Anchor -2
(mm)
Anchor -3
(mm)
May 2, 2021
(initial reading)
34.52 29.04 43.11
June 1, 2021 40.56 34.67 44.78
(A) Discontinuity between Anchor-1 and Anchor-2
(B) Discontinuity between Anchor-2 and Anchor-3
(C) Discontinuity between Anchor-3 and excavation boundary
(D) No noticeable discontinuity
GATE 2022 Mining Engineering (MN)
Page 26
Q.39 Match the semi-variogram shape with the model name and the property
(A) P→III→E, Q→II→F, R→IV→E, S→I→G
(B) P→II→F, Q→I→G, R→III→E, S→IV→E
(C) P→IV→G, Q→III→F, R→II→E, S→I→E
(D) P→II→E, Q→I→E, R→III→F, S→IV→G
GATE 2022 Mining Engineering (MN)
Page 27
Q.40 In a uniaxial compressive strength test, a rock sample of diameter 50 mm fails at an
angle of 60o as shown in the figure. If the peak load at failure is 120 kN, the normal
and shear stresses on failure plane respectively, in MPa are_______ and _____.
(A) 15.28 and 26.46
(B) 26.46 and 15.28
(C) 57.02 and -15.28
(D) -15.28 and 15.28
GATE 2022 Mining Engineering (MN)
Page 28
Q.41 A coal mining company examines the option of buying two types of dumpers with
the following details.
Parameter Type-1 Type-2 Constraints
Capital cost per dumper
(in ₹ crore)
3.0 4.0 Maximum capital available
for purchasing is ₹120 crore
Capacity in tonne 40 50 Minimum daily tonnage to
be hauled is 31,000
Daily trips for each dumper 20 20
Operating cost (in ₹) per tonne 300.0 200.0
In order to minimize the operating cost, the optimum fleet of dumpers of Type-1
and Type-2, respectively are
(A) 20, 15
(B) 0, 30
(C) 0, 31
(D) 40, 0
GATE 2022 Mining Engineering (MN)
Page 29
Q.42 Let 𝑓(𝑥) be a continuous and differentiable function on [3, 18]. If 𝑓(3) = −50 and
𝑓′(𝑥) ≤ 20, then the largest possible value of 𝑓(18), is_______. (in integer)
Q.43 Let ∝ (𝑇 − 𝑇), where 𝑇 and 𝑇 are temperatures in degree centigrade of a
room and thermometer, respectively, and t denotes time in minutes. A thermometer
at a reading of 2 C is brought in a room of temperature 40 C. Two minutes later,
the thermometer reads 15 C. The time elapsed in minutes when the thermometer
reads 39.5 C, is_______. (round off to two decimal places)
Q.44 In a health centre, the probability of ‘full occupancy’ of COVID beds for a day is
0.8. Assuming Binomial probability distribution, the probability of full occupancy
exactly for 5 days in a week, is_______. (round off to three decimal places)
Q.45 Following information is given for a drilling operation to be carried out for
overburden removal in a surface mine.
Volume of rock blasted per round, m3 : 3,20,000
Number of blast holes : 100
Drill hole diameter (D), mm : 200
Length of subgrade drilling : 8D
Stemming length : 25D
Bench height, m : 30
Powder factor, m3/kg : 3.2
The amount of explosive per unit length of charge in kg/m, is_______.
(round off to two decimal places)
GATE 2022 Mining Engineering (MN)
Page 30
Q.46 The shaft-top coordinates of two vertical shafts are given below. The depth of the
shaft A and B are 200 m and 149 m, respectively.
Shaft Latitude (m) Departure (m) Surface RL (m)
A N670.0 W150.0 250
B N170.0 E50.0 209
The downward gradient of the line joining the bottom of the two shafts in degrees,
is_______. (round off to two decimal places)
Q.47 The oxygen-balanced equation for explosive ANFO is given below.
3𝑁𝐻 𝑁𝑂 + 𝐶𝐻 → 7𝐻 𝑂 + 𝐶𝑂 + 3𝑁
For 100 litre of fuel oil having density 850 kg/m3, the amount of ammonium nitrate
to be mixed, in kg, is_______. (round off to two decimal places)
Q.48 Two weightless cables of equal length and cross sectional area are hanging from a
ceiling as shown in the figure. They are connected by a horizontal light bar of length
1.0 m and pulled by a force, F. The modulus of elasticity of Cable-1 and Cable-2
are 50 GPa and 200 GPa, respectively. If the deformation in both the cables is equal,
the distance 𝑥, in m is_______. (round off to one decimal place)
GATE 2022 Mining Engineering (MN)
Page 31
Q.49 A circular tunnel of radius 3 m is constructed in a hydrostatic stress field of 15 MPa.
The modulus of elasticity and Poisson’s ratio of the rock are 5 GPa and 0.25,
respectively. A uniform support pressure 𝑝 is applied at tunnel boundary to restrict
the radial deformation at the tunnel boundary to 4 mm. The value of 𝑝 in MPa
is_______. (round off to two decimal places)
Q.50 The extraction ratio during development of a bord and pillar panel is 0.15 in a flat
coal seam. The panel is further extracted by widening the galleries, and the
extraction ratio changes to 0.25. The percentage change in pillar stress, considering
tributary area method, is_______. (round off to two decimal places)
Q.51 In a small metal mine a battery powered locomotive hauls a train of mine tubs
such that:
The weight of the train of mine tubs, tonne : 3.0
The coefficient of friction between the wheels and the rails : 0.06
The coefficient of adhesion between the loco wheels and the rails : 0.2
Time required from the start to reach speed of 1.8 m/s through constant
acceleration, min : 3.0
Upward gradient to be negotiated : 1 in 20
The minimum weight of the locomotive in tonnes to meet these design
requirements, is_______. (round off to one decimal place)
Q.52 A pump lifts mine water of density 1020 kg/m3, at 250 m3/hr from a depth of
150 m. The overall pumping efficiency is 68%. Considering a head loss of 15 m
due to pipe friction and shock, the motor input power, in kW is_______.
(round off to two decimal places)
GATE 2022 Mining Engineering (MN)
Page 32
Q.53 In a surface mine bench, overburden is removed by the shovel-dumper combination.
For the dumper:
Time required at the loading station : 3.0 min
Time required at the unloading station : 1.0 min
Distance between loading and unloading stations : 4.5 km
Average speed during loaded travel : 12.0 km/hr
Average speed during empty travel : 18.0 km/hr
Minimum number of dumpers required to avoid idle time of the shovel, is _______.
(in integer)
Q.54 In a bord and pillar development panel, headings of 4.4 m × 2.5 m are advanced
using solid blasting. The average pull per round of blast is 1.2 m. On an average 12
faces are blasted per day. Density of coal is 1500 kg/m3. The mine operates in three
shifts. If the average daily employment is 330 persons, labour productivity (OMS)
of the panel in tonne, is_______. (round off to two decimal places)
Q.55 In a longwall face, the full seam thickness of 3 m is cut by a shearer with a web of
depth 0.7 m. The hauling speed of shearer during cutting is 12 m/min. The trough
cross-section of AFC is 0.4 m2 and the average loading coefficient is 0.7. In order
to evacuate coal from the face without spillage, the speed of AFC, in m/s is_______.
(round off to one decimal place)
Q.56 A city is spread over an area of 20 km × 40 km. Wind, at an average speed of
4 m/s, enters perpendicular to the 20 km long side. On a winter day the inversion
layer exists over the city at a height of 100 m. PM1 is emitted from the city at a rate
of 1 kg/s. The steady state PM1 concentration in the city air, in µg/m3, assuming
Box model, is_______. (round off to one decimal place)
GATE 2022 Mining Engineering (MN)
Page 33
Q.57 The point ‘A’ as shown in the Coward flammability diagram represents the gas
composition of a sealed-off area of a coal mine. The volume of the sealed-off area
is 10000 m3. Inert gas is proposed to be injected into the sealed-off area so that the
gas composition comes below the LEL (lower explosibility limit). The minimum
volume of the inert gas required (at the same pressure as that of the sealed-off area),
in m3 is_______. (round off to one decimal place)
Q.58 An underground workshop has dimensions of 8 m length, 6 m width and 4 m height.
Four identical luminaires are placed at the four corners of the roof. Each luminaire
is of 100 W capacity with luminous efficacy of 100 lumen/W. Light is transmitted
spherically from luminaires and there are no reflections. The illumination on the
horizontal plane at the centre of the floor, in lux is_______. (round off to two
decimal places)
Q.59 An underground AC plant requires the delivery of 250 US gpm (15.85 US gpm =
1.0 lps) of chilled water. For this purpose, ice-pellets at 0 0C temperature (latent
heat of melting, 334 kJ/kg) are mixed with water at 20 0C (specific heat
4.18 kJ/kg0C) on the surface. The mixture is adiabatically transported to the
underground location such that the water at 7 0C becomes available for the AC plant.
The requirement of ice-pellets in tonne/hr to meet the design condition, is ______.
(round off to two decimal places)
GATE 2022 Mining Engineering (MN)
Page 34
Q.60 An intake shaft has resistance of 0.05 Ns2/m8 up to a depth of 400 m. The airflow
rate is 100 m3/s and the average density is 1.2 kg/m3. A barometer reads
99.375 kPa when placed on surface. Considering acceleration due to gravity is
9.81 m/s2, the reading of the barometer at the depth of 400 m, in kPa is_______.
(round off to two decimal places)
Q.61 The net present values (NPV) of two mining project proposals A and B are as
given.
𝑁𝑃𝑉 = −0.01𝑖 − 0.02𝑖 + 4.44
𝑁𝑃𝑉 = −0.03𝑖 − 0.01𝑖 + 6.55
where, 𝑖 is discount rate.
The required rate of return for which both the proposals have equal possibility of
acceptance and rejection, is _______. (round off to two decimal places)
Q.62 The value of ∫ 𝑥𝑙𝑜𝑔(1 + 𝑥)𝑑𝑥, is_______. (round off to two decimal places)
Q.63 A coal seam of uniform thickness 12 m is dipping at an angle 30o as shown in the
figure. The ultimate pit is demarcated based on allowable instantaneous stripping
ratio of 10 m3/tonne and safe slope angle of 45o. The density of coal is
1.41 tonne/m3. The length, L in m is_______. (round off to two decimal places)
GATE 2022 Mining Engineering (MN)
Page 35
Q.64 A mine has a reserve of 150 million tonne (Mt) and is designed for a maximum
production capacity of 5 Mt per year. In the first year the production is 2 Mt and it
increases by 20% each year. The reserve in Mt that remains at the end of 15 years,
is_______. (round off to two decimal places)
Q.65 Information on Activity-Time duration of a project is provided below
Activity
Predecessor
event
Successor
event
Estimated Time Duration (weeks)
Pessimistic Most likely Optimistic
A 1 2 20 15 4
B 1 3 12 8 4
C 2 3 16 11 6
D 3 4 20 13 12
E 2 4 13 8 3
F 1 4 45 35 25
The expected project duration in weeks, is_______. (in integer)
Q. No. Session Question
Type
Subject
Name
Key/Range Mark
1 3 MCQ GA C 1
2 3 MCQ GA D 1
3 3 MCQ GA B 1
4 3 MCQ GA B 1
5 3 MCQ GA C 1
6 3 MCQ GA D 2
7 3 MCQ GA C 2
8 3 MCQ GA C 2
9 3 MCQ GA B 2
10 3 MCQ GA B 2
11 3 MCQ MN C 1
12 3 MCQ MN B 1
13 3 MCQ MN D 1
14 3 MCQ MN A 1
15 3 MCQ MN A 1
16 3 MCQ MN D 1
17 3 MCQ MN A 1
18 3 MCQ MN D 1
19 3 MCQ MN C 1
20 3 MCQ MN D 1
21 3 MCQ MN C 1
22 3 MCQ MN B 1
23 3 MCQ MN A 1
24 3 MCQ MN C 1
25 3 MCQ MN C 1
26 3 MCQ MN B 1
27 3 MCQ MN A 1
28 3 MCQ MN B 1
29 3 NAT MN 100000.0 to 100000.0 1
30 3 NAT MN 0.28 to 0.28 1
31 3 NAT MN 2.20 to 2.30 1
32 3 NAT MN 0.012 to 0.013 1
33 3 NAT MN 50.00 to 60.00 1
34 3 NAT MN 10.0 to 10.0 1
35 3 NAT MN 180 to 180 1
36 3 MCQ MN D 2
37 3 MCQ MN D 2
38 3 MCQ MN B 2
39 3 MCQ MN A 2
40 3 MCQ MN A 2
41 3 MCQ MN A 2
42 3 NAT MN 250 to 250 2
43 3 NAT MN 20.40 to 20.80 2
44 3 NAT MN 0.251 to 0.299 2
45 3 NAT MN 36.00 to 39.00 2
46 3 NAT MN 0.90 to 1.13 2
47 3 NAT MN 1453.00 to 1458.00 2
48 3 NAT MN 0.8 to 0.8 2
49 3 NAT MN 9.50 to 9.75 2
50 3 NAT MN 13.00 to 14.00 2
51 3 NAT MN 3.7 to 3.8 2
52 3 NAT MN 165.00 to 170.00 2
53 3 NAT MN 14 to 14 2
54 3 NAT MN 0.70 to 0.75 2
55 3 NAT MN 1.4 to 1.6 2
56 3 NAT MN 125.0 to 125.0 2
57 3 NAT MN 2000.0 to 2400.0 2
58 3 NAT MN 45.00 to 50.00 2
59 3 NAT MN 7.00 to 7.50 2
60 3 NAT MN 103.50 to 103.60 2
61 3 NAT MN 10.00 to 11.00 2
62 3 NAT MN 0.25 to 0.25 2
63 3 NAT MN 320.00 to 330.00 2
64 3 NAT MN 84.80 to 85.50 2
65 3 NAT MN 39 to 39 2
GA - General Aptitude
Q1 - Q5 carry one mark each.
Q.No. 1
(A)
(B)
(C)
(D)
Q.No. 2
(A)
(B)
(C)
(D)
Q.No. 3
(A)
(B)
(C)
(D)
Q.No. 4
(A)
(B)
(C)
(D)
Q.No. 5
(A)
(B)
(C)
MN: Mining Engineering
GATE 2020
Graduate Aptitude Test in Engineering 2020
IIT Delhi
Organising Institute
Home
Home Information Brochure
Information Brochure GATE International
GATE International Pre Examination
Pre Examination Important Dates
Important Dates FAQs
FAQs Contact Us
Contact Us
(D)
Q6 - Q10 carry two marks each.
Q.No. 6
(A)
(B)
(C)
(D)
Q.No. 7
(A)
(B)
(C)
(D)
Q.No. 8
(A)
(B)
(C)
(D)
Q.No. 9
(A)
(B)
(C)
(D)
Q.No. 10
(A)
(B)
(C)
(D)
MN: Mining Engineering
Q1 - Q25 carry one mark each.
Q.No. 1
(A)
(B)
(C)
(D)
Q.No. 2
(A)
(B)
(C)
(D)
Q.No. 3
(A)
(B)
(C)
(D)
Q.No. 4
(A)
(B)
(C)
(D)
Q.No. 5
(A)
(B)
(C)
(D)
Q.No. 6
(A)
(B)
(C)
(D)
Q.No. 7
(A)
(B)
(C)
(D)
Q.No. 8
(A)
(B)
(C)
(D)
Q.No. 9
(A)
(B)
(C)
(D)
Q.No. 10
(A)
(B)
(C)
(D)
Q.No. 11
(A)
(B)
(C)
(D)
Q.No. 12
(A)
(B)
(C)
(D)
Q.No. 13
(A)
(B)
(C)
(D)
Q.No. 14
(A)
(B)
(C)
(D)
Q.No. 15
(A)
(B)
(C)
(D)
Q.No. 16
Q.No. 17
Q.No. 18
Q.No. 19
Q.No. 20
Q.No. 21
Q.No. 22
Q.No. 23
Q.No. 24
Q.No. 25
Q26 - Q55 carry two marks each.
Q.No. 26
(A)
(B)
(C)
(D)
Q.No. 27
(A)
(B)
(C)
(D)
Q.No. 28
(A)
(B)
(C)
(D)
Q.No. 29
(A)
(B)
(C)
(D)
Q.No. 30
(A)
(B)
(C)
(D)
Q.No. 31
(A)
(B)
(C)
(D)
Q.No. 32
(A)
(B)
(C)
(D)
Q.No. 33
(A)
(B)
(C)
(D)
Q.No. 34
Q.No. 35
Q.No. 36
Q.No. 37
Q.No. 38
Q.No. 39
Q.No. 40
Q.No. 41
Q.No. 42
Q.No. 43
Q.No. 44
Q.No. 45
Q.No. 46
Q.No. 47
Q.No. 48
Q.No. 49
Q.No. 50
Q.No. 51
Q.No. 52
Q.No. 53
Q.No. 54
Q.No. 55
Copyright : GATE 2020, IIT Delhi
Answer Key - MN: Mining Engineering
Q.No. Session Que.Type Sec. Name Key Marks
1 3 MCQ GA C 1
2 3 MCQ GA B 1
3 3 MCQ GA D 1
4 3 MCQ GA D 1
5 3 MCQ GA B 1
6 3 MCQ GA D 2
7 3 MCQ GA C 2
8 3 MCQ GA B 2
9 3 MCQ GA B 2
10 3 MCQ GA B 2
1 3 MCQ MN C 1
2 3 MCQ MN A 1
3 3 MCQ MN D 1
4 3 MCQ MN B 1
5 3 MCQ MN B 1
6 3 MCQ MN C 1
7 3 MCQ MN A 1
8 3 MCQ MN D 1
9 3 MCQ MN A 1
10 3 MCQ MN B 1
11 3 MCQ MN D 1
12 3 MCQ MN C 1
13 3 MCQ MN C 1
14 3 MCQ MN B 1
15 3 MCQ MN D 1
16 3 NAT MN 18.0 to 20.0 1
17 3 NAT MN 2.9 to 3.1 1
18 3 NAT MN 5.4 to 5.6 1
19 3 NAT MN 0.120 to 0.125 1
20 3 NAT MN 2.4 to 2.6 1
21 3 NAT MN 51830.0 to 51850.0 1
22 3 NAT MN 0.560 to 0.580 1
23 3 NAT MN 2 to 2 1
24 3 NAT MN 5990.0 to 6010.0 1
25 3 NAT MN 0.32 to 0.34 1
26 3 MCQ MN B 2
27 3 MCQ MN D 2
28 3 MCQ MN C 2
29 3 MCQ MN A 2
30 3 MCQ MN D 2
31 3 MCQ MN C 2
32 3 MCQ MN D 2
33 3 MCQ MN D 2
GATE 2020
Graduate Aptitude Test in Engineering 2020
IIT Delhi
Organising Institute
Home
Home Information Brochure
Information Brochure GATE International
GATE International Pre Examination
Pre Examination Important Dates
Important Dates FAQs
FAQs Contact Us
Contact Us
34 3 NAT MN 0.530 to 0.550 2
35 3 NAT MN 148.0 to 154.0 2
36 3 NAT MN 385.0 to 410.0 2
37 3 NAT MN -205.0 to -195.0 2
38 3 NAT MN 22.00 to 25.00 2
39 3 NAT MN 7.2 to 7.7 2
40 3 NAT MN -1.255 to -1.165 2
41 3 NAT MN 2380.0 to 2396.0 2
42 3 NAT MN 2.10 to 2.45 2
43 3 NAT MN 3360.0 to 3480.0 2
44 3 NAT MN 71.0 to 72.0 2
45 3 NAT MN 71.0 to 73.0 2
46 3 NAT MN 87.0 to 89.0 2
47 3 NAT MN 0.44 to 0.50 2
48 3 NAT MN 100.4 to 100.6 2
49 3 NAT MN 157.0 to 161.0 2
50 3 NAT MN 248.0 to 252.0 2
51 3 NAT MN 0.020 to 0.030 2
52 3 NAT MN 67.00 to 69.00 2
53 3 NAT MN 18 to 18 2
54 3 NAT MN 7.70 to 7.95 2
55 3 NAT MN 30.8 to 31.6 2
Copyright : GATE 2020, IIT Delhi
GATE 2018 General Aptitude (GA) Set-3
GA 1/3
Q. 1 – Q. 5 carry one mark each.
Q.1 “From where are they bringing their books? ________ bringing _______ books from
_____.”
The words that best fill the blanks in the above sentence are
(A) Their, they’re, there
(B) They’re, their, there
(C) There, their, they’re
(D) They’re, there, there
Q.2 “A _________ investigation can sometimes yield new facts, but typically organized ones
are more successful.”
The word that best fills the blank in the above sentence is
(A) meandering (B) timely (C) consistent (D) systematic
Q.3 The area of a square is 𝑑. What is the area of the circle which has the diagonal of the
square as its diameter?
(A) 𝜋𝑑 (B) 𝜋𝑑2
(C)
1
4
𝜋𝑑2
(D)
1
2
𝜋𝑑
Q.4 What would be the smallest natural number which when divided either by 20 or by 42 or
by 76 leaves a remainder of 7 in each case?
(A) 3047 (B) 6047 (C) 7987 (D) 63847
Q.5 What is the missing number in the following sequence?
2, 12, 60, 240, 720, 1440, _____, 0
(A) 2880 (B) 1440 (C) 720 (D) 0
GATE 2018 General Aptitude (GA) Set-3
GA 2/3
Q. 6 – Q. 10 carry two marks each.
Q.6 In appreciation of the social improvements completed in a town, a wealthy philanthropist
decided to gift Rs 750 to each male senior citizen in the town and Rs 1000 to each female
senior citizen. Altogether, there were 300 senior citizens eligible for this gift. However,
only 8/9th
of the eligible men and 2/3rd
of the eligible women claimed the gift. How much
money (in Rupees) did the philanthropist give away in total?
(A) 1,50,000 (B) 2,00,000
(C) 1,75,000 (D) 1,51,000
Q.7 If 𝑝𝑞𝑟 ≠ 0 and 𝑝−𝑥
=
1
𝑞
, 𝑞−𝑦
=
1
𝑟
, 𝑟−𝑧
=
1
𝑝
, what is the value of the product 𝑥𝑦𝑧?
(A) −1 (B)
1
𝑝𝑞𝑟
(C) 1 (D) 𝑝𝑞𝑟
Q.8 In a party, 60% of the invited guests are male and 40% are female. If 80% of the invited
guests attended the party and if all the invited female guests attended, what would be the
ratio of males to females among the attendees in the party?
(A) 2:3 (B) 1:1 (C) 3:2 (D) 2:1
Q.9 In the figure below, ∠𝐷𝐸𝐶 + ∠𝐵𝐹𝐶 is equal to ____________ .
(A) ∠𝐵𝐶𝐷 − ∠𝐵𝐴𝐷 (B) ∠𝐵𝐴𝐷 + ∠𝐵𝐶𝐹
(C) ∠𝐵𝐴𝐷 + ∠𝐵𝐶𝐷 (D) ∠𝐶𝐵𝐴 + ∠𝐴𝐷𝐶
A
B
D
E
C
F
GATE 2018 General Aptitude (GA) Set-3
GA 3/3
Q.10 A six sided unbiased die with four green faces and two red faces is rolled seven times.
Which of the following combinations is the most likely outcome of the experiment?
(A) Three green faces and four red faces.
(B) Four green faces and three red faces.
(C) Five green faces and two red faces.
(D) Six green faces and one red face.
END OF THE QUESTION PAPER
GATE 2018 Mining Engineering
MN 1/11
Q. 1 – Q. 25 carry one mark each.
Q.1 If
cos sin
sin cos
X
 
 
 
  

 
, then T
XX is
(A)
0 1
1 0
 
 
 
(B)
1 0
0 1

 
 

 
(C)
1 0
0 1
 
 
 
(D)
0 1
1 0

 
 

 
Q.2 The values of x satisfying the following condition are
4 3
0
3 6
x
x



(A) 6, 4 (B) 4, 9 (C) 5, 6 (D) 3,7
Q.3 An azimuth of 330° corresponds to a quadrant bearing of
(A) W60o
N (B) N30o
W (C) S30o
W (D) S30o
E
Q.4 Tri-cone drill bit is a type of
(A) cross bit (B) button bit
(C) rotary roller bit (D) button drag bit
Q.5 Exposure of weak roof in junctions of a development district in a coal mine can be
decreased by
(A) increasing dimension of panel barrier
(B) stitching side walls
(C) increasing support density at junctions
(D) staggering the junctions
Q.6 The property that CANNOT be determined from uniaxial compressive strength test of a
rock sample fitted with strain gauges is
(A) cohesion (B) Poisson’s ratio
(C) modulus of elasticity (D) dilation
GATE 2018 Mining Engineering
MN 2/11
Q.7 The radial stress concentration around a long circular tunnel excavated in rock is given by
the curve
(A) m (B) n (C) o (D) p
Q.8 Ward-Leonard system is provided in the winding system in order to restrict
(A) over-winding of the cage
(B) deceleration of the cage
(C) acceleration of the cage
(D) over-speeding of the cage
Q.9 The equipment NOT related to extraction of coal from longwall face operation is
(A) AFC (B) road header (C) powered support (D) DERD shearer
Q.10 The correct figure depicting the extraction of contiguous seams in bord and pillar working
is indicated by
Q.11 The significance of ‘potentially explosive mixture’ in Coward flammability diagram is -
(A) leakage of fresh air may lead to explosive condition for the mixture
(B) leakage of firedamp may lead to explosive condition for the mixture
(C) increase in the ambient temperature may lead to explosive condition for the mixture
(D) introduction of a source of ignition may result in explosion of the mixture
/
r a
 
m
 
n
 
p
 
o
0
p
r - distance from the center
Stress
concentration
0
p
a
- in-situ stress
0
p
8m
10m
(A) (B)
(C) (D)
8m
pillar
10m
parting
parting
pillar
GATE 2018 Mining Engineering
MN 3/11
Q.12 Considering ‘I’ as ‘intake’ and ‘R’ as ‘return’, the ventilation symbol for the shaft-bottom
air-lock in a coal mine is
Q.13 From a coal seam of a mine 1000 tonnes of coal is produced per day. The seam has
inflammable gas emission rate of 14000 m3
per day. Percentage of inflammable gas in
general body of air is 0.14. The gassiness of the seam is
(A) Degree IV (B) Degree III (C) Degree II (D) Degree I
Q.14 The temperature profiles and the plume patterns that are most likely to result are given in
the figures. The dotted line represents ‘adiabatic lapse rate’ and the bold line represents
‘environmental lapse rate’. The WRONG combination is
I R
I R
I R
I R
(A)
(C)
(B)
(D)
z
(A)
(D)
z
T
(C)
(B)
z
T
z
T
T
GATE 2018 Mining Engineering
MN 4/11
Q.15 The inventory pattern shown does NOT represent the following.
(A) Inventory is replenished
instantaneously
(B) Demand decreases with time
(C) Shortage is not permitted (D) Demand is uniform
Q.16 The figure depicts a transportation problem along with the solution. The correct statement
is
(A) unbalanced problem, optimal solution
(B) unbalanced problem, sub-optimal solution
(C) balanced problem, optimal solution
(D) balanced problem, sub-optimal solution
Q.17 The degree of the differential equation
2
3
2
2 0
d x
x
dt
  is ______________.
Q.18 The slope of the line connecting the points (20, 6) and (40, 8) is ______________.
Q.19 Two contours of RL 60 m and 70 m are separated by 34 m measured along dip of the seam.
The dip of the seam in degree is ________________.
Time
Inventory
level
D2
D1
5 20
15 5
10 5
0 15 15
15
S1
S2
10 20
GATE 2018 Mining Engineering
MN 5/11
Q.20 The RL of the initial station is 200 m. If 1.54m
BS 
 and 0.45m
FS 
 , then the RL
of the last station in m is ________________.
Q.21 For the conveyor belt drive shown, the tension on the tight side  
1
T is double that of the
slack side. The coefficient of friction between belt and drum is 0.21. The minimum angle
of lap (in degree) to avoid slippage of the belt is ________________.
Q.22 In a bord and pillar development district, 6 headings driven along the strike direction are
surrounded by panel barriers on the dip and rise sides. The maximum possible number of
faces for the panel is ___________________.
Q.23 A roadway in a mine, a single light source as shown, emits light uniformly in all directions.
The floor level illumination at station A is 20 lux. The floor level illumination at a point B
in lux is ______________.
Q.24 A system of two identical components connected in series has reliability of 0.25. The
reliability of each component is _______________.
Q.25 The operational status of an HEMM in a shift is shown in the diagram. The availability of
the machine in % is _____________________.
1
T
2
T
400
A B
0 1 8
3 5 6 7.5
Breakdown state
Shiftduration hours
GATE 2018 Mining Engineering
MN 6/11
Q. 26 – Q. 55 carry two marks each.
Q.26 If c is a constant, the solution of the differential equation 4 9 0
dy
y x
dx
  is
(A)
2 2
81 16
x y
c
  (B)
2 2
16 81
x y
c
  (C)
2 2
9 4
x y
c
  (D)
2 2
4 9
x y
c
 
Q.27 Match the following blasting elements with the corresponding initiators.
Blasting Elements Initiators
P Electric detonator 1 Match stick
Q Safety fuse 2 Booster
R Detonating fuse 3 Exploder
S Non cap-sensitive explosive 4 Ordinary detonator
(A) P-2, Q-3, R-4, S-1 (B) P-3, Q-1, R-4, S-2
(C) P-3, Q-1, R-2, S-4 (D) P-1, Q-4, R-2, S-3
Q.28 The following plot is developed for a rock type after a series of triaxial tests. The uniaxial
compressive strength and tensile strength of the rock type, respectively, in MPa, are
(A) 30, 5 (B) 30, 6 (C) 24, 6 (D) 24, 5
Q.29 Match the following in the context of environmental management.
Technique Purpose
P Mulching 1 Dust control
Q Aeration 2 Noise control
R Wet-scrubbing 3 Soil conservation
S Silencer 4 Waste water treatment
(A) P-1, Q-4, R-3, S-2 (B) P-3, Q-1, R-4, S-2
(C) P-4, Q-3, R-1, S-2 (D) P-3, Q-4, R-1, S-2
1 3
6 30
 
 
3 (MPa)

1

3

1 (MPa)

GATE 2018 Mining Engineering
MN 7/11
Q.30 A panel in coal mine produces 400 tonne per day. The number of persons employed in each
of the three shifts is 110, 130 and 120. As per CMR, the minimum quantity of air that
needs to be circulated at the last ventilation connection of the panel in m3
/min is
(A) 780 (B) 900 (C) 1000 (D) 2400
Q.31 A right conical iron ore stack on level ground of height 10 m has 60% Fe. The height of the
conical stack is extended up to 20 m using iron ore of 50% Fe. The angle of repose of iron
ore is 38o
. The mean grade of the final stack in % Fe is ___________________.
Q.32 The feasible region of a linear programming problem is shown in the figure. The maximum
value of the objective function 4 3
Z X Y
  is ____________________.
Q.33 Cash flow of a project of duration 4 years is shown. The uniform income ‘R’, in Rs. crores,
at the end of 2nd
, 3rd
and 4th
years for an internal rate of return of 10% is ______________.
0 2 3 4
1
R R R
Rs. 10 crores
year
GATE 2018 Mining Engineering
MN 8/11
Q.34 The grade values of alumina at three sample locations (A, B and C) in a bauxite deposit are
as shown. Using the ‘inverse distance’ method, the computed grade in % alumina at
location ‘Z’ is ___________________.
Q.35 Economic feasibility of two methods is examined for meeting a targeted mine production.
On a yearly basis, the following cost parameters (Rs in crores) are applicable for these
methods.
Method 1 Method 2
Fixed cost 20 5
Variable cost 2X X2
The annual rate of production, ‘X’ in million tonnes for which both the methods will yield
the same operating cost is ___________________.
Q.36 A person standing 50 m away from an HEMM experiences 90 dB sound pressure level. If
the person moves to a new location that is 70 m away from the HEMM, the sound pressure
level experienced by the person, in dB, becomes _________________.
Q.37 A portion of a ventilation system has two splits as shown. Split ‘A’ has a resistance of
0.2 Ns2
/m8
and a regulator of size 2.0 m2
. The resistance of split ‘B’ in Ns2
/m8
is ________
Q.38 Air enters a bord and pillar panel at 10 ppm CO and 20.78% O2. The air at the panel return
has 80 ppm CO and 20.52% O2. The ‘Graham’s ratio’ for the status of fire in the panel
in % is __________________.
A (30%)
C (42%)
B (36%)
Z
90 m
60 m
60 m
R
A
20 m3/s
B 30 m3/s
GATE 2018 Mining Engineering
MN 9/11
Q.39 In a sublevel stope, 20 m high excavation is made with ring drilling in a 6 m wide orebody.
In an effective shift time of 6 hrs, three rings are blasted resulting in 9 m extraction length.
LHDs of 5.0 m3
bucket capacity and cycle time of 5 minutes are used for ore
transportation. The number of LHDs required for this operation is _________________.
Q.40 A coal seam lying at a depth of 200 m is developed by bord and pillar method. Pillars are
30 m centre to centre with a gallery width of 4 m. Unit weight of the overlying strata is
28 kN/m3
. If the pillar strength is 9.32 MPa, the factor of safety of pillar is ____________.
Q.41 Following information about a longwall retreating panel is given.
Panel length = 1800 m
Face length = 150 m
Depth of web = 0.6 m
Shearer travels at a speed of 1.5 m/min along the face.
Each cutting cycle requires a non-operational time of 2 h 20 min. The panel is fully
extractable. The number of working days required to extract the panel is ______________.
Q.42 In a direct rope haulage operating along a 1500 m long incline, 180 tonne of coal is hauled
in 7 hours. The average rope speed is 7.5 km/h. The set changing time for the tubs is
2 minutes each at the top and bottom of the incline. If the tub capacity is 1.0 tonne, the
number of tubs in the set is __________________.
Q.43 An SDL of 1.0 tonne capacity operates with a cycle time of 6 minutes. The dimension of
the face is 4 m x 3 m. Five blasts are conducted per shift with an average pull of 1.2 m. If
the density of blasted coal is 0.8 tonne/m3
, the time required by the SDL in the shift to lift
all the prepared coal in hours is _________________.
Q.44 A force diagram is shown below. Considering clockwise moments to be positive, the
resultant moment about A in Nm is _________________.
3 N
7 N
9 N
A
120 m
50 m
70 m
GATE 2018 Mining Engineering
MN 10/11
Q.45 The experiment to determine permeability of a soil sample is illustrated below. The cross-
sectional area of the sample is 20.0 cm2
. The permeability of the soil sample in cm/s is
______________.
Q.46 In underground face blasting, the pull is found to be 10% less than the drillhole length. The
headings of 3.6 m width are supported by prop density of 1.44/m2
. The hole length is
1.5 m, and 6 rounds of blasting are done in the panel per shift. The number of props to be
erected in a shift is ____________________.
Q.47 The immediate roof of a mine is supported by bolts of length 1.5 m, arranged in
1.2 m x 1.2 m grid pattern. If the unit weight of roof rock is 2.25 tonne/m3
and load
carrying capacity of each bolt is 7.5 tonne, the factor of safety of the support system
is _______________.
Q.48 In a level terrain, a vertical orebody of 20 m uniform width is worked by surface mining
method. The density of the ore is 2.5 tonne/m3
. The ultimate pit has a depth of 60 m, width
of 20 m at pit bottom, and a pit slope of 45o
. The overall stripping ratio for this condition in
m3
/tonne is ____________________.
Q.49 A 20 m long steel tape used for survey is found to be short by 10 cm. If the area measured
with the steel tape is 5000 m2
, the actual area in m2
is __________________.
5 cm
Water 360 ml/h
Soil sample
20 cm
GATE 2018 Mining Engineering
MN 11/11
Q.50 The following figure shows the designed blast pattern of a bench. The explosive column is
charged at 18 kg/m. If the unit weight of the blasted material is 2.5 tonne/m3
, the powder
factor for the blast in tonne/kg is _______________.
Q.51 Two points on the equator have longitudes 55o
E and 25o
W. Considering radius of earth as
6400 km, the distance between the two points in km is ________________.
Q.52 Sum of the series 5, 10, 15, ……….., 500 is __________________.
Q.53 The sample standard deviation for the following set of observations is _______________.
40, 45, 50 and 55
Q.54 For the given function     
, 3 4
f x y x y
   , the value of
f f
x y
 

 
at 2 and 1
x y
  is
_____________________.
Q.55 Given 2
6
y x x
   , the value of  
ln
d
y
dx
at 2
x  is ________________.
END OF THE QUESTION PAPER
4 m 3 m
10 m
3 m
8 m
stemming
explosive
Q.No. Type Section Key/Range Marks
1 MCQ GA B 1
2 MCQ GA A 1
3 MCQ GA D 1
4 MCQ GA C 1
5 MCQ GA B 1
6 MCQ GA B 2
7 MCQ GA C 2
8 MCQ GA B 2
9 MCQ GA A 2
10 MCQ GA C 2
1 MCQ MN C 1
2 MCQ MN D 1
3 MCQ MN B 1
4 MCQ MN C 1
5 MCQ MN D 1
6 MCQ MN A 1
7 MCQ MN D 1
8 MCQ MN D 1
9 MCQ MN B 1
10 MCQ MN C 1
11 MCQ MN A 1
12 MCQ MN A 1
13 MCQ MN B 1
Q.No. Type Section Key/Range Marks
14 MCQ MN C 1
15 MCQ MN B 1
16 MCQ MN C 1
17 NAT MN 1.0 to 1.0 1
18 NAT MN 0.1 to 0.1 1
19 NAT MN 16.5 to 17.5 1
20 NAT MN 201.09 to 201.09 1
21 NAT MN 185.0 to 195.0 1
22 NAT MN 16.0 to 16.0 1
23 NAT MN 8.5 to 9.5 1
24 NAT MN 0.5 to 0.5 1
25 NAT MN 56.0 to 56.5 1
26 MCQ MN D 2
27 MCQ MN B 2
28 MCQ MN A 2
29 MCQ MN D 2
30 MCQ MN C 2
31 NAT MN 51.0 to 52.0 2
32 NAT MN 74.0 to 74.0 2
33 NAT MN 4.4 to 4.5 2
34 NAT MN 36.0 to 37.5 2
35 NAT MN 5.0 to 5.0 2
36 NAT MN 86.75 to 87.25 2
Q.No. Type Section Key/Range Marks
37 NAT MN 0.24 to 0.28 2
38 NAT MN 2.65 to 2.75 2
39 NAT MN 3.0 to 3.0 2
40 NAT MN 1.2 to 1.3 2
41 NAT MN 500.0 to 500.0 2
42 NAT MN 12.0 to 12.0 2
43 NAT MN 5.6 to 5.8 2
44 NAT MN ‐400.0 to ‐400.0 2
45 NAT MN 0.02 to 0.02 2
46 NAT MN 41.0 to 43.0 2
47 NAT MN 1.4 to 1.6 2
48 NAT MN 1.2 to 1.2 2
49 NAT MN 4945.0 to 4955.0 2
50 NAT MN 2.00 to 2.15 2
51 NAT MN 8930.0 to 8940.0 2
52 NAT MN 25250.0 to 25250.0 2
53 NAT MN 6.4 to 6.5 2
54 NAT MN 10.0 to 10.0 2
55 NAT MN 0.40 to 0.45 2
Graduate Aptitude Test in Engineering 2017
Question Paper Name: Mining Engineering 12th Feb 2017
Subject Name:
Duration: 180
Total Marks: 100
Question Number : 2 Correct : 1 Wrong : -0.33
Question Number : 3 Correct : 1 Wrong : -0.33
Question Number :1 Correct : 1 Wrong : -0.33
Question Number : 6 Correct : 1 Wrong : -0.33
Question Number : 7 Correct : 1 Wrong : -0.33
Question Number : 8 Correct : 1 Wrong : 0
Question Number : 5 Correct : 1 Wrong : -0.33
Question Number : 4 Correct : 1 Wrong : -0.33
Question Number : 10 Correct : 1 Wrong : -0.33
Question Number : 11 Correct : 1 Wrong : -0.33
Question Number : 9 Correct : 1 Wrong : -0.33
Question Number : 13 Correct : 1 Wrong : -0.33
Question Number : 14 Correct : 1 Wrong : -0.33
Question Number : 15 Correct : 1 Wrong : -0.33
Question Number : 16 Correct : 1 Wrong : 0
Question Number : 12 Correct : 1 Wrong : -0.33
Question Number : 18 Correct : 1 Wrong : 0
Question Number : 19 Correct : 1 Wrong : 0
Question Number : 20 Correct : 1 Wrong :- 0.33
Question Number : 17 Correct : 1 Wrong : -0.33
Question Number : 22 Correct : 1 Wrong : -0.33
Question Number : 23 Correct : 1 Wrong : 0
Question Number : 21 Correct : 1 Wrong : -0.33
Question Number : 25 Correct : 1 Wrong : -0.33
Question Number : 26 Correct : 2 Wrong : 0
Question Number : 27 Correct : 2 Wrong : 0
Question Number : 24 Correct : 1 Wrong : -0.33
Question Number : 29 Correct : 2 Wrong : 0
Question Number : 30 Correct : 2 Wrong : 0
Question Number : 28 Correct : 2 Wrong : 0
Question Number : 32 Correct : 2 Wrong : 0
Question Number : 33 Correct : 2 Wrong : 0
Question Number : 31 Correct : 2 Wrong : 0
Question Number : 35 Correct : 2 Wrong : 0
Question Number : 34 Correct : 2 Wrong : 0
Question Number : 37 Correct : 2 Wrong : 0
Question Number : 38 Correct : 2 Wrong : 0
Question Number : 36 Correct : 2 Wrong : 0
Question Number : 39 Correct : 2 Wrong : 0
Question Number : 40 Correct : 2 Wrong : -0.66
Question Number : 42 Correct : 2 Wrong : -0.66
Question Number : 41 Correct : 2 Wrong : -0.66
Question Number : 44 Correct : 2 Wrong : -0.66
Question Number : 45 Correct : 2 Wrong : 0
Question Number : 43 Correct : 2 Wrong : 0
Question Number : 46 Correct : 2 Wrong : -0.66
Question Number : 47 Correct : 2 Wrong : 0
Question Number : 49 Correct : 2 Wrong : 0
Question Number : 50 Correct : 2 Wrong : 0
Question Number : 48 Correct : 2 Wrong : 0
Question Number : 52 Correct : 2 Wrong : 0
Question Number : 53 Correct : 2 Wrong : 0
Question Number : 54 Correct : 2 Wrong : -0.66
Question Number : 51 Correct : 2 Wrong : 0
Question Number : 56 Correct : 1 Wrong : -0.33
Question Number : 55 Correct : 2 Wrong : 0
Question Number : 58 Correct : 1 Wrong : -0.33
Question Number : 59 Correct : 1 Wrong : -0.33
Question Number : 60 Correct : 1 Wrong : -0.33
Question Number : 57 Correct : 1 Wrong : -0.33
Question Number : 62 Correct : 2 Wrong : -0.66
Question Number : 63 Correct : 2 Wrong : -0.66
Question Number : 61 Correct : 2 Wrong : -0.66
Question Number : 65 Correct : 2 Wrong : -0.66
Question Number : 64 Correct : 2 Wrong : -0.66
Q. No. Type Section Key Marks
1 MCQ MN C 1
2 MCQ MN B 1
3 MCQ MN D 1
4 MCQ MN A 1
5 MCQ MN B 1
6 MCQ MN C 1
7 MCQ MN D 1
8 NAT MN 2.9 to 3.0 1
9 MCQ MN C 1
10 MCQ MN B 1
11 MCQ MN C 1
12 MCQ MN A 1
13 MCQ MN A 1
14 MCQ MN D 1
15 MCQ MN B 1
16 NAT MN 60 to 62 1
17 MCQ MN A 1
18 NAT MN 5.15 to 6.0 1
19 NAT MN 113 to 116 or 1.13 to 1.16 1
20 MCQ MN D 1
21 MCQ MN B 1
22 MCQ MN C 1
23 NAT MN 835 to 840 1
24 MCQ MN B 1
25 MCQ MN D 1
26 NAT MN -2.0 to -2.0 2
27 NAT MN 32 to 32 2
28 NAT MN 0.96 to 0.98 2
29 NAT MN 123.0 to 124.5 2
30 NAT MN 174765 to 174765 2
31 NAT MN 20 to 23 2
32 NAT MN 25 to 27 2
33 NAT MN 46 to 47 2
34 NAT MN 71 to 73 2
35 NAT MN 65000 to 67000 2
36 NAT MN 0.96 to 0.97 2
37 NAT MN 1.6 to 1.7 2
38 NAT MN 5.5 to 6.0 2
39 NAT MN 19.5 to 20 2
40 MCQ MN B 2
41 MCQ MN A 2
42 MCQ MN C 2
43 NAT MN 1 to 3 2
44 MCQ MN D 2
45 NAT MN 5.5 to 6.0 2
46 MCQ MN A 2
47 NAT MN 13.5 to 14.0 2
48 NAT MN 1185 to 1189 2
49 NAT MN 2.5 to 2.8 2
50 NAT MN 36 to 37 2
51 NAT MN 290 to 292 2
52 NAT MN 935 to 936 2
53 NAT MN 0.66 to 0.68 2
54 MCQ MN D 2
55 NAT MN 5975 to 5990 2
56 MCQ GA C 1
57 MCQ GA C 1
58 MCQ GA D 1
59 MCQ GA C 1
60 MCQ GA D 1
61 MCQ GA B 2
62 MCQ GA B 2
63 MCQ GA B 2
64 MCQ GA A 2
65 MCQ GA C 2
GATE 2016 General Aptitude - GA Set-2
1/3
Q. 1 – Q. 5 carry one mark each.
Q.1 The volume of a sphere of diameter 1 unit is ________ than the volume of a cube of side 1 unit.
(A) least (B) less (C) lesser (D) low
Q.2 The unruly crowd demanded that the accused be _____________ without trial.
(A) hanged (B) hanging (C) hankering (D) hung
Q.3 Choose the statement(s) where the underlined word is used correctly:
(i) A prone is a dried plum.
(ii) He was lying prone on the floor.
(iii) People who eat a lot of fat are prone to heart disease.
(A) (i) and (iii) only (B) (iii) only (C) (i) and (ii) only (D) (ii) and (iii) only
Q.4 Fact: If it rains, then the field is wet.
Read the following statements:
(i) It rains
(ii) The field is not wet
(iii) The field is wet
(iv) It did not rain
Which one of the options given below is NOT logically possible, based on the given fact?
(A) If (iii), then (iv). (B) If (i), then (iii).
(C) If (i), then (ii). (D) If (ii), then (iv).
Q.5 A window is made up of a square portion and an equilateral triangle portion above it. The base of
the triangular portion coincides with the upper side of the square. If the perimeter of the window is
6 m, the area of the window in m2
is ___________.
(A) 1.43 (B) 2.06 (C) 2.68 (D) 2.88
GATE 2016 General Aptitude - GA Set-2
2/3
Q. 6 – Q. 10 carry two marks each.
Q.6 Students taking an exam are divided into two groups, P and Q such that each group has the same
number of students. The performance of each of the students in a test was evaluated out of 200
marks. It was observed that the mean of group P was 105, while that of group Q was 85. The
standard deviation of group P was 25, while that of group Q was 5. Assuming that the marks were
distributed on a normal distribution, which of the following statements will have the highest
probability of being TRUE?
(A) No student in group Q scored less marks than any student in group P.
(B) No student in group P scored less marks than any student in group Q.
(C) Most students of group Q scored marks in a narrower range than students in group P.
(D) The median of the marks of group P is 100.
Q.7 A smart city integrates all modes of transport, uses clean energy and promotes sustainable use of
resources. It also uses technology to ensure safety and security of the city, something which critics
argue, will lead to a surveillance state.
Which of the following can be logically inferred from the above paragraph?
(i) All smart cities encourage the formation of surveillance states.
(ii) Surveillance is an integral part of a smart city.
(iii) Sustainability and surveillance go hand in hand in a smart city.
(iv) There is a perception that smart cities promote surveillance.
(A) (i) and (iv) only (B) (ii) and (iii) only
(C) (iv) only (D) (i) only
Q.8 Find the missing sequence in the letter series.
B, FH, LNP, _ _ _ _.
(A) SUWY (B) TUVW (C) TVXZ (D) TWXZ
Q.9 The binary operation □ is defined as a □ b = ab+(a+b), where a and b are any two real numbers.
The value of the identity element of this operation, defined as the number x such that a □ x = a, for
any a, is .
(A) 0 (B) 1 (C) 2 (D) 10
GATE 2016 General Aptitude - GA Set-2
3/3
Q.10 Which of the following curves represents the function = ln(| | (| |)|
|) for | | < 2 ?
Here, represents the abscissa and represents the ordinate.
(A)
(B)
(C)
(D)
END OF THE QUESTION PAPER
GATE 2016 Mining Engineering (MN)
MN 1/14
Q. 1 – Q. 25 carry one mark each.
Q.1 The differential of the equation,
2 2
1
x y
  , with respect to x is
(A) x y
 (B) x y (C) y x
 (D) y x
Q.2 If [ ][ ] [ ]
A B I
 then
(A) [ ] [ ]T
B A
 (B) [ ] [ ]T
A B
 (C) 1
[ ] [ ]
B A 
 (D) [ ] [ ]
B A

Q.3 4
X C
 is the general integral of
(A) 3
3 x dx
 (B) 3
1
4
x dx

(C) 3
x dx
 (D) 3
4 x dx

Q.4 Sinh (x) is
(A)
4
x x
e e

(B)
2
x x
e e

(C)
2
x x
e e

(D)
4
x x
e e

Q.5 Identify the correct statement.
NONEL is used for surface connection of the blast holes in order to
(A) achieve better water resistance over detonating fuse
(B) have a precise delay timing
(C) provide noiseless shock front movement
(D) avoid deflagration
Q.6 Identify the pattern of surface blasting given in the figure. The values of delay time, in ms,
are given against each blasthole.
(A) V- cut
(B) extended V- cut
(C) row to row
(D) en echelon
100 75 50 25 25 50 75 100
50 25 0 0 0 0 25 50
GATE 2016 Mining Engineering (MN)
MN 2/14
Q.7 Identify the initiation sequence which is NOT possible for surface blasting.
(A) Detonating fuse NonelElectronic detonator
(B) Electric detonatorNonelDetonating fuse
(C) Electric detonator Detonating fuse  Nonel
(D) Electronic detonator Detonating fuse  Nonel
Q.8 Parallel holes at right angles to the face with some holes uncharged are associated with the
following shot hole pattern
(A) drag cut (B) wedge cut (C) pyramid cut (D) burn cut
Q.9 Bieniawski’s Rock Mass Rating considers the parameters: RQD, spacing of joints, condition of
joints, ground water condition, and
(A) tensile strength
(B) uniaxial compressive strength
(C) shear strength
(D) buckling strength
Q.10 A rockmass is subjected to hydrostatic pressure of 6 MPa. If each of the measured strains
xx yy zz
  
  , is 2.0 mm/m, then the bulk modulus, in GPa, is _______
Q.11 Identify the uniaxial compressive loading condition from the following four Mohr circles.
(A) (1) (B) (2) (C) (3) (D) (4)
GATE 2016 Mining Engineering (MN)
MN 3/14
Q.12 Out of the given stress-strain curves, identify the rock type that is most prone to rock burst.
(A) (1) (B) (2) (C) (3) (D) (4)
Q.13 A longwall panel of width 120 m is extracted at a depth of 200 m. Critical subsidence is reached
when the panel length becomes 150 m. If the seam were to be worked at a depth of 300 m, critical
subsidence would be observed at a panel length, in m, of _______.
Q.14 The support system followed along the goaf edge in a depillaring panel is
(A) rope stitching
(B) cable bolting
(C) wooden/steel chock
(D) hydraulic prop
Q.15 Which one of the following ropes CANNOT be an effective cable bolt?
(A) locked coil wire rope
(B) Langs lay wire rope
(C) ordinary lay wire rope
(D) bird-caged wire rope
Q.16 In metalliferous mines, the sublevel interval does NOT depend on
(A) capacity of drilling equipment
(B) capacity of loading equipment
(C) strength of rib pillar
(D) strength of wall rock
(4)
(3)
(1)
σ
ε
(2)
0,0
GATE 2016 Mining Engineering (MN)
MN 4/14
Q.17 Jack hammer does NOT contain
(A) pawl and ratchet
(B) gear box
(C) rifle bar
(D) piston
Q.18 At the inlet of a mine roadway, the dry and wet bulb temperatures of air are
0
38 Cand
0
29 C ,
respectively. At the outlet, the corresponding temperatures are
0
32 C and
0
29 C , respectively. The
process of heat transfer in the airway is described as
(A) evaporative cooling
(B) sensible cooling
(C) sensible heating
(D) dehumidification
Q.19 Underground coal mines are in principle ventilated by exhausting system, so that
(A) spontaneous heating risk is reduced
(B) fumes can be quickly removed in case of an underground fire
(C) build-up of methane concentration is decreased
(D) cool and fresh intake air can enter underground
Q.20 Identify the WRONG statement.
Pit bottom air lock
(A) prevents the short circuiting of air when the flow is reversed in coal mines
(B) has at least three doors
(C) has at least one door that has provision for latching
(D) all doors are in principle designed to open towards high pressure side of the air
Q.21 Identify the WRONG statement.
The ‘temperature inversion’ of the atmosphere in surface mines aggravates the problem of
(A) airborne dust
(B) noise
(C) ground vibrations
(D) visibility
Q.22 In a CO self rescuer, the purpose of the calcium bromide and lithium chloride mixture is to
(A) dry the incoming air
(B) convert the CO catalytically to 2
CO
(C) absorb and thereby neutralise CO
(D) cool the inhaled air from excess exothermic heat due to chemical reaction
GATE 2016 Mining Engineering (MN)
MN 5/14
Q.23 IRR of a project is the discount rate at which
(A) profit after tax is zero
(B) written down value of the project is zero
(C) revenue from the project is zero
(D) NPV is zero
Q.24 For the critical path network shown, the slack for the activity ‘b’, in months, is
(A) 4 (B) 6 (C) 9 (D) 13
Q.25 The three axes comprising the numerical codification of resources, as per the UNFC, are
(A) Economic Viability, Geological Assessment, Geotechnical Assessment
(B) Geological Assessment, Environmental Assessment, Feasibility Assessment
(C) Feasibility Assessment, Geological Assessment, Mining Assessment
(D) Economic Viability, Geological Assessment, Feasibility Assessment
Q. 26 – Q. 55 carry two marks each.
Q.26 Equations of two planes are 4
z  and 4 3
z x
  . The included angle between the two planes in
degrees, is ____________
Q.27 A force ˆ
ˆ ˆ
2 5 6
P i j k
  

acts on a particle. The particle is moved from point A to point B, where
the position vectors of A

and B

are ˆ
ˆ ˆ
6 3
i j k
  and ˆ
ˆ ˆ
4 3 2
i j k
  respectively. The work done is
____________
Q.28 The value of x in the simultaneous equations is ______________
3 2 3
2 3 3
2 4
x y z
x y z
x y z
  
   
  
GATE 2016 Mining Engineering (MN)
MN 6/14
Q.29 Two persons P and Q toss an unbiased coin alternately on an understanding that whoever gets the
head first wins. If P starts the game, then the probability of P winning the game is ____________
Q.30 Data pertaining to a surface bench blast is given below:
Burden = 3.0 m Sub-grade drilling = 1.0 m
Spacing = 4.0 m Collar stemming = 4.0 m
Bench height = 10.0 m Air decking length = 1.0 m
Density of rock = 2000 3
kg/m Linear charge concentration = 10 kg/m
The powder factor of the blast, in kg/tonne, is _______
Q.31 Match the following for a typical slurry explosive.
Chemical Purpose
P. Calcium nitrate 1. Cross linking agent
Q. Potassium dichromate 2. Gelling agent
R. TNT 3. Oxidiser
S. Starch 4. Fuel
(A) P-1, Q-2, R-3, S-4
(B) P-2, Q-4, R-3, S-1
(C) P-3, Q-1, R-4, S-2
(D) P-4, Q-3, R-2, S-1
Q.32 A 10 m thick coal block is excavated by a contractor at a cost of Rs. 40 per 3
m . The excavated
area, measured in the mine plan, is found to be
2
50 cm . If the mine plan has been drawn to a scale
of 1:1000, the payment to be made to the contractor, in lakhs of Rs., is _______
Q.33 Two vertical shafts of a mine have the following parameters:
The gradient of the drift connecting the shaft bottoms, in degrees, is_____
Shaft Shaft‐A Shaft‐B
Collar RL (m) 0.0 0.0
Depth(m) 250 200
Northing(m) 200 100
Easting(m) 100 ‐100
GATE 2016 Mining Engineering (MN)
MN 7/14
Q.34 For a station ‘A’ on the Earth’s surface, as shown in the figure, match the following
Arc Description
Q. MA 1. Longitude
R. LM 2. Co-latitude
S. PA 3. Latitude
(A) Q-2, R-3, S-1
(B) Q-3, R-1, S-2
(C) Q-2, R-1, S-3
(D) Q-3, R-2, S-1
Q.35 Match the following for the prismatic compass shown below
Component Name
P. X1 1. Agate bearing
Q. X2 2. Object vane
R. X3 3. Magnetic needle
S. X4 4. Prism
(A) P-1, Q-2, R-3, S-4
(B) P-1, Q-3, R-2, S-4
(C) P-2, Q-1, R-4, S-3
(D) P-3, Q-1, R-4, S-2
Q.36 A ladder placed against a frictionless wall at an inclination of
0
60 with horizontal, is in a state of
limiting equilibrium. The ladder has a length of 13 m and a uniform mass of 4 kg/m. The
coefficient of friction between the ladder and the floor is ______
A
P
M
L
O
P’
Reference meridian
Equator
GATE 2016 Mining Engineering (MN)
MN 8/14
Q.37 A cubical rock sample is enclosed between two fixed hard steel plates as shown in the figure below.
The modulus of elasticity and Poisson’s ratio of the rock are 2 GPa and 0.25, respectively. If the
rock is subjected to the stresses as shown in the figure, the strain in x-direction, in mm/m, is
_______.
Q.38 In a hydrostatic stress field, point A is in the middle of two circular openings as shown in the
figure. The radial stress, in MPa, at point A is _________.
Steel plate
Rock block
5 Mpa
8 MPa
x
y
z
10 MPa
10 m
2.5 m 2.5 m
A
5 MPa 5 MPa
5 MPa
5 MPa
GATE 2016 Mining Engineering (MN)
MN 9/14
Q.39 Curves (a) and (b) represent the stress distributions along the length of a ‘full column grouted bolt’
shown in the figure. Curves (a) and (b) are
(A) Tensile stress, Compressive stress
(B) Axial stress, Shear stress
(C) Compressive stress, Tensile stress
(D) Shear stress, Axial stress
Q.40 Match the following mechanical properties with the formulae
Mechanical property Formula
P. Modulus of elasticity 1. n
c tan
 

Q. Compressive strength 2. lateral longitudinal
/
 
R. Shear Strength 3. /
 
S. Poisson’s ratio 4.
2
n
F / r

(A) P-1, Q-2, R-3, S-4
(B) P-1, Q-4, R-3, S-2
(C) P-3, Q-4, R-1, S-2
(D) P-3, Q-2, R-1, S-4
Q.41 A skip of 10 tonne capacity hoists ore through a 1000 m deep shaft at a speed of 20 m/s. The skip
accelerates and decelerates at 2.0
2
m/s . The loading and unloading times for the skip are 2.5 min
and 1.5 min, respectively. The maximum hourly capacity of the hoisting system, in tonnes, is
_____________
Q.42 Match the following:
Haulage unit Safety device
P. Friction winder
Q. Drum winder
R. Direct rope haulage
S. Endless rope haulage
1. Run-away switch
2. Lilly controller
3. Regenerative braking
4. Monkey/back catch
(A) P-1, Q-2, R-3, S-4
(B) P-3, Q-2, R-1, S-4
(C) P-1, Q-3, R-4, S-2
(D) P-2, Q-3, R-1, S-4
Pickup length Anchor length
(b)
(a)
Bolt
Grouted material
Rock surface
Stress
Length
0,0
GATE 2016 Mining Engineering (MN)
MN 10/14
Q.43 In the gear assembly shown, the rpm of Gear 1 is 600. The number of teeth in Gear 1, Gear 2, Gear
3, Gear 4, Gear 5 and Gear 6 is 30, 45, 15, 20, 10 and 30, respectively. The rpm of Gear 6 is
______
Q.44 An operating surface mine is proposed to be deepened by 30 m as shown in the figure. If the
density of the ore is 2.4 3
tonne m , the incremental stripping ratio for the deepening, in
3
m tonne, is ______.
Q.45 From an openpit sump, mine water is lifted using a 250 m long straight pipeline laid along a
gradient of
0
34 . The pumping rate is 500 gpm (1 gallon = 3.8 litres). Additional head loss due to
pipe friction can be considered to be 10% of head lifted. At an overall efficiency of 70%, the
electric power consumed by the pump, in kW, is __________.
Overburden
300
300
150 m
100 m
30 m
100 m
Proposed line of deepening
300
30 m Ore
GATE 2016 Mining Engineering (MN)
MN 11/14
Q.46 With reference to Coward diagram, match the following in the context of explosibility of a mixture
of ‘normal air’ and ‘methane’.
(O2 %, CH4 %) Mixture status
P. 20.5, 2.4 1. Impossible mixture
Q. 19.0, 9.5 2. Non-explosive
R. 17.0, 19.0 3. Potentially explosive
S. 20.0, 19.5 4. Explosive
(A) P-2, Q-4, R-3, S-1
(B) P-2, Q-3, R-1, S-4
(C) P-2, Q-4, R-1, S-3
(D) P-3, Q-2, R-1, S-4
Q.47 A U-tube manometer is subjected to differential pressure as shown. If specific gravity of kerosene
is 0.8, the value of  
1 2
P P
 , in Pa, is_________.
Q.48 An air stream having an enthalpy of 100 kJ/kgda, is flowing at 20 kgda/s. It is cooled by water at
temperature
0
10 C circulating in a cooling coil at a flow rate of 10.0 l/s. If the return temperature of
water is
0
20 C , the enthalpy of the cooled air, in kJ/kgda, is ___________.
(Specific heat of water: 4.18 0
kJ kg C ; kgda: kg of dry air).
GATE 2016 Mining Engineering (MN)
MN 12/14
Q.49 The static pressure characteristic of a mine fan is as shown. If the mine resistance is 0.3 2 8
Ns m ,
the quantity generated by the fan, in 3
m s , is _______.
Q.50 In the context of ventilation plan symbols, match the following:
Symbol Description
P. 1. Temporary stopping
Q. 2. Regulator
R. 3. Air-crossing
S. 4. Ventilation stopping
(A) P-3, Q-4, R-2, S-1
(B) P-2, Q-3, R-1, S-4
(C) P-1, Q-3, R-4, S-2
(D) P-3, Q-2, R-1, S-4
Q.51 A mill concentrate, having 25% copper, is proposed to be sold at Rs. 1,25,000 per tonne. The grade
of the deposit is 0.8% Cu and the overall cost of mining and milling is Rs. 2,520 per tonne of ore.
At a recovery of 75%, the estimated profit, in Rs./tonne of concentrate, is ________.
1000
0.0 100
P
(Pa)
Q (m3
/s)
R
GATE 2016 Mining Engineering (MN)
MN 13/14
Q.52 Copper grade distribution in an ore body has the probability density function,  
f x , as shown in
the figure. The average grade of the deposit, in % Cu, is ______.
Q.53 The semivariogram shown belongs to a bauxite deposit. The expected difference in the Al2O3 (%)
values between two boreholes separated by a distance of 200 m is _______.
Q.54 A surface mine has 15 identical dumpers and two shovels. For shovel 1, the dumper cycle time is
30 min and the shovel loading time is 5 min. For shovel 2, the dumper cycle time is 32 min and the
shovel loading time is 4.0 min. Based on match factor optimisation (equitable match factor), the
ideal allocation of dumpers to shovel 1 and shovel 2, respectively is
(A) 6, 9 (B) 7, 8 (C) 9, 6 (D) 8, 7
300 m
0.0
γ (h)
Lag distance
100 (%)2
300 (%)2
GATE 2016 Mining Engineering (MN)
MN 14/14
Q.55 The composited grade value, in %, between the RLs 10 m to 20 m for the following borehole
configuration is __________.
END OF THE QUESTION PAPER
14 m
18 m
RL = 0 m
25%
35%
Q. No Type Section Key Marks
1 MCQ GA B 1
2 MCQ GA A 1
3 MCQ GA D 1
4 MCQ GA C 1
5 MCQ GA B 1
6 MCQ GA C 2
7 MCQ GA C 2
8 MCQ GA C 2
9 MCQ GA A 2
10 MCQ GA C 2
1 MCQ MN A 1
2 MCQ MN C 1
3 MCQ MN D 1
4 MCQ MN B 1
5 MCQ MN C 1
6 MCQ MN B 1
7 MCQ MN A 1
8 MCQ MN D 1
9 MCQ MN B 1
10 NAT MN 1.00 : 1.00 1
11 MCQ MN A 1
12 MCQ MN D 1
13 NAT MN 225.00 : 225.00 1
14 MCQ MN C 1
15 MCQ MN A 1
16 MCQ MN B 1
17 MCQ MN B 1
18 MCQ MN A 1
19 MCQ MN C 1
20 MCQ MN C 1
21 MCQ MN C 1
22 MCQ MN A 1
23 MCQ MN D 1
24 MCQ MN B 1
25 MCQ MN D 1
26 NAT MN 100.00 : 110.00 2
27 NAT MN 22.00 : 22.00 2
28 NAT MN 1.00 : 1.00 2
29 NAT MN 0.66 : 0.67 2
30 NAT MN 0.25 : 0.25 2
31 MCQ MN C 2
32 NAT MN 20.00 : 20.00 2
33 NAT MN 12.00 : 13.00 2
34 MCQ MN B 2
35 MCQ MN C 2
36 NAT MN 0.28 : 0.30 2
37 NAT MN 0.20 : 0.30 2
38 NAT MN 7.00 : 8.00 2
39 MCQ MN D 2
40 MCQ MN C 2
41 NAT MN 100.00 : 110.00 2
42 MCQ MN D 2
43 NAT MN 100.00 : 100.00 2
44 NAT MN 0.55 : 0.75 2
45 NAT MN 60.00 : 70.00 2
46 MCQ MN A 2
47 NAT MN -960.00 : -930.00 2
48 NAT MN 78.00 : 80.00 2
49 NAT MN 43.00 : 44.00 2
50 MCQ MN A 2
51 NAT MN 19500.00 : 20500.00 2
52 NAT MN 0.77 : 0.84 2
53 NAT MN 0.00 : 0.00 2
54 MCQ MN A 2
55 NAT MN 31.00 : 31.00 2
Graduate Aptitude Test in Engineering
Notations :
Question Paper Name: MN: MINING ENGINEERING 1st Feb shift2
Number of Questions: 65
Total Marks: 100.0
Number of Questions: 10
Section Marks: 15.0
Question Number : 1 Question Type : MCQ
Options :
Question Number : 2 Question Type : MCQ
Options :
Question Number : 3 Question Type : MCQ
Wrong answer for MCQ will result in negative marks, (-1/3) for 1 mark Questions and (-2/3) for 2 marks Questions.
Q.1 to Q.5 carry 1 mark each & Q.6 to Q.10 carry 2 marks each.
Options :
Question Number : 4 Question Type : MCQ
Options :
Question Number : 5 Question Type : MCQ
Options :
Question Number : 6 Question Type : MCQ
Options :
Question Number : 7 Question Type : MCQ
Options :
Question Number : 8 Question Type : NAT
Correct Answer :
280
Question Number : 9 Question Type : MCQ
Options :
Question Number : 10 Question Type : MCQ
Options :
Number of Questions: 55
Section Marks: 85.0
Question Number : 11 Question Type : MCQ
Options :
Question Number : 12 Question Type : MCQ
Q.11 to Q.35 carry 1 mark each & Q.36 to Q.65 carry 2 marks each.
Options :
Question Number : 13 Question Type : NAT
Correct Answer:
104 to 105
Question Number : 14 Question Type : MCQ
Options :
Question Number : 15 Question Type : MCQ
Options :
Question Number : 16 Question Type : MCQ
Options :
Question Number : 17 Question Type : MCQ
Options :
Question Number : 18 Question Type : MCQ
Options :
Question Number : 19 Question Type : MCQ
Options :
Question Number : 20 Question Type : MCQ
Options :
Question Number : 21 Question Type : NAT
Correct Answer :
4
Question Number : 22 Question Type : MCQ
Options :
Question Number : 23 Question Type : MCQ
Options :
Question Number : 24 Question Type : NAT
Correct Answer:
7900 to 7920
Question Number : 25 Question Type : MCQ
Options :
Question Number : 26 Question Type : NAT
Correct Answer:
25 to 26
Question Number : 27 Question Type : MCQ
Options :
Question Number : 28 Question Type : MCQ
Options :
Question Number : 29 Question Type : MCQ
Options :
Question Number : 30 Question Type : NAT
Correct Answer:
4.2 to 5.0
Question Number : 31 Question Type : NAT
Correct Answer :
4.6 to 4.9
Question Number : 32 Question Type : MCQ
Options :
Question Number : 33 Question Type : NAT
Correct Answer :
13.0 to 13.6
Question Number : 34 Question Type : MCQ
Options :
Question Number : 35 Question Type : NAT
Correct Answer:
0.75 to 0.81
Question Number : 36 Question Type : MCQ
Options :
Question Number : 37 Question Type : NAT
Correct Answer:
2.48 to 2.52
Question Number : 38 Question Type : MCQ
Options :
Question Number : 39 Question Type : NAT
Correct Answer:
4700
Question Number : 40 Question Type : MCQ
Options :
Question Number : 41 Question Type : MCQ
Options :
Question Number : 42 Question Type : NAT
Correct Answer :
161 to 165
Question Number : 43 Question Type : MCQ
Options :
Question Number : 44 Question Type : NAT
Correct Answer :
10.2 to 10.8
Question Number : 45 Question Type : NAT
Correct Answer :
82 to 86
Question Number : 46 Question Type : NAT
Correct Answer :
20 to 21
Question Number : 47 Question Type : NAT
Correct Answer :
286 to 293
Question Number : 48 Question Type : NAT
Correct Answer :
83 to 87
Question Number : 49 Question Type : MCQ
Options :
Question Number : 50 Question Type : NAT
Correct Answer:
3.8 to 4.2
Question Number : 51 Question Type : NAT
Correct Answer :
0.5 to 0.6
Question Number : 52 Question Type : MCQ
Options :
Question Number : 53 Question Type : NAT
Correct Answer :
106 to 113
Question Number : 54 Question Type : NAT
Correct Answer :
220 to 228
Question Number : 55 Question Type : NAT
Correct Answer:
116
Question Number : 56 Question Type : MCQ
Options :
Question Number : 57 Question Type : NAT
Correct Answer :
1.88 to 1.92
Question Number : 58 Question Type : NAT
Correct Answer :
0.8 to 0.85
Question Number : 59 Question Type : MCQ
Options :
Question Number : 60 Question Type : NAT
Correct Answer :
0.6
Question Number : 61 Question Type : MCQ
Options :
Question Number : 62 Question Type : MCQ
Options :
Question Number : 63 Question Type : NAT
Correct Answer :
19.10 to 19.25
Question Number : 64 Question Type : NAT
Correct Answer :
13 to 14
Question Number : 65 Question Type : NAT
Correct Answer :
0.055 to 0.065
GATE 2014: General Instructions during Examination
1. Total duration of the GATE examination is 180 minutes.
2. The clock will be set at the server. The countdown timer at the top right corner of
screen will display the remaining time available for you to complete the examination.
When the timer reaches zero, the examination will end by itself. You need not
terminate the examination or submit your paper.
3. Any useful data required for your paper can be viewed by clicking on the Useful
Common Data button that appears on the screen.
4. Use the scribble pad provided to you for any rough work. Submit the scribble pad at
the end of the examination.
5. You are allowed to use a non-programmable type calculator, however, sharing of
calculators is not allowed.
6. The Question Palette displayed on the right side of screen will show the status of
each question using one of the following symbols:
The Marked for Review status for a question simply indicates that you would like to look at
that question again. If a question is answered, but marked for review, then the answer will
be considered for evaluation unless the status is modified by the candidate.
Navigating to a Question :
7. To answer a question, do the following:
a. Click on the question number in the Question Palette to go to that question
directly.
b. Select an answer for a multiple choice type question by clicking on the bubble
placed before the 4 choices, namely A, B, C and D. Use the virtual numeric
keypad to enter a number as answer for a numerical type question.
c. Click on Save & Next to save your answer for the current question and then go
to the next question.
d. Click on Mark for Review & Next to save your answer for the current question
and also mark it for review, and then go to the next question.
Caution: Note that your answer for the current question will not be saved, if you navigate
to another question directly by clicking on a question number without saving the answer to
the previous question.
You can view all the questions by clicking on the Question Paper button. This feature is
provided, so that if you want you can just see the entire question paper at a glance.
Answering a Question :
8. Procedure for answering a multiple choice (MCQ) type question:
a. Choose one answer from the 4 options (A,B,C,D) given below the question,
click on the bubble placed before the chosen option.
b. To deselect your chosen answer, click on the bubble of the chosen option again
or click on the Clear Response button.
c. To change your chosen answer, click on the bubble of another option.
d. To save your answer, you MUST click on the Save & Next button.
9. Procedure for answering a numerical answer type question:
a. To enter a number as your answer, use the virtual numerical keypad.
b. A fraction (e.g. -0.3 or -.3) can be entered as an answer with or without '0'
before the decimal point. As many as four decimal points, e.g. 12.5435 or
0.003 or -932.6711 or 12.82 can be entered.
c. To clear your answer, click on the Clear Response button.
d. To save your answer, you MUST click on the Save & Next button
10. To mark a question for review, click on the Mark for Review & Next button. If an
answer is selected (for MCQ) or entered (for numerical answer type) for a question
that is Marked for Review, that answer will be considered in the evaluation unless
the status is modified by the candidate.
11. To change your answer to a question that has already been answered, first select
that question for answering and then follow the procedure for answering that type of
question.
12. Note that ONLY Questions for which answers are saved or marked for review after
answering will be considered for evaluation.
Choosing a Section :
13. Sections in this question paper are displayed on the top bar of the screen. Questions
in a Section can be viewed by clicking on the name of that Section. The Section you
are currently viewing will be highlighted.
14. A checkbox is displayed for every optional Section, if any, in the Question Paper. To
select the optional Section for answering, click on the checkbox for that Section.
15. If the checkbox for an optional Section is not selected, the Save & Next button and
the Mark for Review & Next button will NOT be enabled for that Section. You will
only be able to see questions in this Section, but you will not be able to answer
questions in the Section.
16. After clicking the Save & Next button for the last question in a Section, you will
automatically be taken to the first question of the next Section in sequence.
17. You can move the mouse cursor over the name of a Section to view the answering
status for that Section.
Changing the Optional Section :
18. After answering the chosen optional Section, partially or completely, you can change
the optional Section by selecting the checkbox for a new Section that you want to
attempt. A warning message will appear along with a table showing the number of
questions answered in each of the previously chosen optional Sections and a
checkbox against each of these Sections. Click on a checkbox against a Section that
you want to reset and then click on the RESET button. Note that RESETTING a Section
will DELETE all the answers for questions in that Section. Hence, if you think that you
may want to select this Section again later, you will have to note down your answers
for questions in that Section. If you do not want to reset the Section and want to
continue answering the previously chosen optional Section, then click on the BACK
button.
19. If you deselect the checkbox for an optional Section in the top bar, the following
warning message will appear: "Deselecting the checkbox will DELETE all the answers
for questions in this Section. Do you want to deselect this Section?” If you want to
deselect, click on the RESET button. If you do not want to deselect, click on the BACK
button.
20. You can shuffle between different Sections or change the optional Sections any
number of times.
GATE 2014 Examination
MN: Mining Engineering
Duration: 180 minutes Maximum Marks: 100
Read the following instructions carefully.
1. To login, enter your Registration Number and password provided to you. Kindly go through the various
symbols used in the test and understand their meaning before you start the examination.
2. Once you login and after the start of the examination, you can view all the questions in the question
paper, by clicking on the View All Questions button in the screen.
3. This question paper consists of 2 sections, General Aptitude (GA) for 15 marks and the subject
specific GATE paper for 85 marks. Both these sections are compulsory.
The GA section consists of 10 questions. Question numbers 1 to 5 are of 1-mark each, while question
numbers 6 to 10 are of 2-mark each.
The subject specific GATE paper section consists of 55 questions, out of which question numbers 1 to
25 are of 1-mark each, while question numbers 26 to 55 are of 2-mark each.
4. Depending upon the GATE paper, there may be useful common data that may be required for
answering the questions. If the paper has such useful data, the same can be viewed by clicking on the
Useful Common Data button that appears at the top, right hand side of the screen.
5. The computer allotted to you at the examination center runs specialized software that permits only one
answer to be selected for multiple-choice questions using a mouse and to enter a suitable number for
the numerical answer type questions using the virtual keyboard and mouse.
6. Your answers shall be updated and saved on a server periodically and also at the end of the
examination. The examination will stop automatically at the end of 180 minutes.
7. In each paper a candidate can answer a total of 65 questions carrying 100 marks.
8. The question paper may consist of questions of multiple choice type (MCQ) and numerical answer
type.
9. Multiple choice type questions will have four choices against A, B, C, D, out of which only ONE is the
correct answer. The candidate has to choose the correct answer by clicking on the bubble (⃝) placed
before the choice.
10. For numerical answer type questions, each question will have a numerical answer and there will not be
any choices. For these questions, the answer should be enteredby using the virtual keyboard that
appears on the monitor and the mouse.
11. All questions that are not attempted will result in zero marks. However, wrong answers for multiple
choice type questions (MCQ) will result in NEGATIVE marks. For all MCQ questions a wrong
answer will result in deduction of⅓ marks for a 1-mark question and ⅔ marks for a 2-mark question.
12. There is NO NEGATIVE MARKING for questions of NUMERICAL ANSWER TYPE.
13. Non-programmable type Calculator is allowed. Charts, graph sheets, and mathematical tables are NOT
allowed in the Examination Hall. You must use the Scribble pad provided to you at the examination
centre for all your rough work. The Scribble Pad has to be returned at the end of the examination.
Declaration by the candidate:
“I have read and understood all the above instructions. I have also read and understood clearly the
instructions given on the admit card and shall follow the same. I also understand that in case I am found to
violate any of these instructions, my candidature is liable to be cancelled. I also confirm that at the start of
the examination all the computer hardware allotted to me are in proper working condition”.
GATE 2014 SET- 2 General Aptitude -GA
GA 1/2
Q. 1 – Q. 5 carry one mark each.
Q.1 Choose the most appropriate word from the options given below to complete the following
sentence.
A person suffering from Alzheimer’s disease short-term memory loss.
(A) experienced (B) has experienced
(C) is experiencing (D) experiences
Q.2 Choose the most appropriate word from the options given below to complete the following
sentence.
____________ is the key to their happiness; they are satisfied with what they have.
(A) Contentment (B) Ambition (C) Perseverance (D) Hunger
Q.3 Which of the following options is the closest in meaning to the sentence below?
“As a woman, I have no country.”
(A) Women have no country.
(B) Women are not citizens of any country.
(C) Women’s solidarity knows no national boundaries.
(D) Women of all countries have equal legal rights.
Q.4 In any given year, the probability of an earthquake greater than Magnitude 6 occurring in the
Garhwal Himalayas is 0.04. The average time between successive occurrences of such earthquakes
is ____ years.
Q.5 The population of a new city is 5 million and is growing at 20% annually. How many years would
it take to double at this growth rate?
(A) 3-4 years (B) 4-5 years (C) 5-6 years (D) 6-7 years
Q. 6 – Q. 10 carry two marks each.
Q.6 In a group of four children, Som is younger to Riaz. Shiv is elder to Ansu. Ansu is youngest in the
group. Which of the following statements is/are required to find the eldest child in the group?
Statements
1. Shiv is younger to Riaz.
2. Shiv is elder to Som.
(A) Statement 1by itself determines the eldest child.
(B) Statement 2 by itself determines the eldest child.
(C) Statements 1 and 2 are both required to determine the eldest child.
(D) Statements 1 and 2 are not sufficient to determine the eldest child.
G
A
0
2
(
G
A
T
E
2
0
1
4
)
GATE 2014 SET- 2 General Aptitude -GA
GA 2/2
Q.7 Moving into a world of big data will require us to change our thinking about the merits of
exactitude. To apply the conventional mindset of measurement to the digital, connected world of
the twenty-first century is to miss a crucial point. As mentioned earlier, the obsession with
exactness is an artefact of the information-deprived analog era. When data was sparse, every data
point was critical, and thus great care was taken to avoid letting any point bias the analysis.
From “BIG DATA” Viktor Mayer-Schonberger and Kenneth Cukier
The main point of the paragraph is:
(A) The twenty-first century is a digital world
(B) Big data is obsessed with exactness
(C) Exactitude is not critical in dealing with big data
(D) Sparse data leads to a bias in the analysis
Q.8 The total exports and revenues from the exports of a country are given in the two pie charts below.
The pie chart for exports shows the quantity of each item as a percentage of the total quantity of
exports. The pie chart for the revenues shows the percentage of the total revenue generated through
export of each item. The total quantity of exports of all the items is 5 lakh tonnes and the total
revenues are 250 crore rupees. What is the ratio of the revenue generated through export of Item 1
per kilogram to the revenue generated through export of Item 4 per kilogram?
(A) 1:2 (B) 2:1 (C) 1:4 (D) 4:1
Q.9 X is 1 km northeast of Y. Y is 1 km southeast of Z. W is 1 km west of Z. P is 1 km south of W. Q is
1 km east of P. What is the distance between X and Q in km?
(A) 1 (B) √2 (C) √3 (D) 2
Q.10 10% of the population in a town is HIV+
. A new diagnostic kit for HIV detection is available; this
kit correctly identifies HIV+
individuals 95% of the time, and HIV−
individuals 89% of the time. A
particular patient is tested using this kit and is found to be positive. The probability that the
individual is actually positive is _______
END OF THE QUESTION PAPER
Item 1
11%
Item 2
20%
Item 3
19%
Item 4
22%
Item 5
12%
Item 6
16%
Exports
Item 1
12%
Item 2
20%
Item 3
23%
Item 4
6%
Item 5
20%
Item 6
19%
Revenues
G
A
0
2
(
G
A
T
E
2
0
1
4
)
GATE 2014 MINING – MN
Q. 1 – Q. 25 carry one mark each.
Q.1 A block of weight 100 kN rests on a floor as shown in the figure. The coefficient of static friction
between the block and the floor is 0.5. A force of 45 kN is applied horizontally on the block. The
static frictional force in kN is
(A) 22.5 (B) 50.0 (C) 55.0 (D) 100.0
Q.2 A spring of constant stiffness k is stretched from point A to point B (displacement u in the figure)
by a force F. The potential energy of the spring is expressed by
(A) 2
1
2
ku Fu
− (B) 2
1
2
ku Fu
+
(C) ku F
− (D) ku F
+
Q.3 If s
σ is the induced stress and i
σ is the insitu stress at a point below ground, the ‘stress
concentration’ at that point is
(A) s
i
σ
σ
(B) i
s
σ
σ
(C) i
s
σ
σ
(D) s
i
σ
σ
Q.4 The components of state of stress at a point in x−y plane are given as 5
xx
σ = MPa, 10
yy
σ = MPa
and 2
xy
τ = − MPa. The sum of the principal stresses acting on the x−y plane in MPa is _______
Q.5 The angle 50
15′
25′′
is expressed in hours, minutes, and seconds as
(A) 1ℎ
20𝑚𝑚𝑚𝑚𝑚𝑚
1.67𝑠𝑠
(B) 1ℎ
20𝑚𝑚𝑚𝑚𝑚𝑚
16.00𝑠𝑠
(C) 0ℎ
21𝑚𝑚𝑚𝑚𝑚𝑚
1.67𝑠𝑠
(D) 0ℎ
21𝑚𝑚𝑚𝑚𝑚𝑚
16.00𝑠𝑠
Q.6 A circular curve has a radius of 200 m and deflection angle of 650
. The length of the curve in m is
(A) 221 (B) 227 (C) 235 (D) 262
Q.7 The weight strength of ANFO of specific gravity 0.8 is 912 kcal/kg. The weight strength of an
emulsion explosive of specific gravity 1.2 is 850 kcal/kg. Bulk strength of the emulsion explosive
relative to ANFO in percentage is ________________
100 kN
45 kN
Coefficientof static friction = 0.5
A B F
u
k
MN 1/8
M
N
(
G
A
T
E
2
0
1
4
)
GATE 2014 MINING – MN
Q.8 In a cut-and-fill stope, the main purpose of back filling is to
(A) reduce ore dilution
(B) prevent high stress concentrations in far field domain
(C) prevent displacement due to dilation of fractured wall rock
(D) improve ore rehandling
Q.9 Bypass valve in a compressed oxygen type self-contained breathing apparatus is meant to
(A) release accumulated nitrogen in the breathing bag
(B) release excess pressure in the breathing bag
(C) supply oxygen directly to wearer in case pressure reducing valve does not function
(D) flush out the apparatus with oxygen on opening the cylinder valve
Q.10 Given S is the setting load and Y is the yield load of a hydraulic prop, the correct relationship is
(A) S < Y (B) S > Y (C) S = Y (D) S = Y2
Q.11
Solution of the differential equation
dy
ky
dx
= follows exponential decay (where k is a constant) for
[ ]
0,
x∈ ∞ if
(A) 0
k > (B) 0
k < (C) 0
k = (D) k e
=
Q.12 The value of k for which the vectors 2 3
a = i - j and 4
k +
b = i j are orthogonal to each other is ___
Q.13 Which one of the following is the most likely mode of slope failure for waste dump
(A) Circular (B) Wedge
(C) Plane (D) Toppling
Q.14 The occurrence of head in a single toss of an unbiased coin is given by a random variable X. The
variance of X is _______________
Q.15 The divergence of the vector ( )( )
x y y x
= + − +
v i j is
(A) y x
− (B) x y
− (C) 2 2
x y
− (D) 2 2
y x
−
Q.16
The
0
lim
x
x
x
→
is
(A) −1 (B) 0 (C) 1 (D) non-existent
Q.17 For Indian coal mines, the ‘maximum allowable concentration’ of respirable dust containing 7.5%
free silica in mg/m3
is
(A) 2.0 (B) 2.2 (C) 2.5 (D) 2.7
MN 2/8
M
N
(
G
A
T
E
2
0
1
4
)
GATE 2014 MINING – MN
Q.18 Given 𝑘𝑘 is the thermal conductivity, 𝜌𝜌 is density and 𝑐𝑐 is specific heat of a rock sample, the thermal
diffusivity of the rock sample is
(A)
𝑘𝑘𝑘𝑘
𝑐𝑐
(B)
𝜌𝜌𝜌𝜌
𝑘𝑘
(C)
𝑘𝑘𝑘𝑘
𝜌𝜌
(D)
𝑘𝑘
𝜌𝜌𝜌𝜌
Q.19 Cyclone, bag filter and scrubber can be used for control of
(A) water pollution (B) air pollution
(C) soil pollution (D) noise pollution
Q.20 A mine waste dump of pH 5.2 can be neutralized by adding
(A) urea (B) calcium carbonate
(C) sulphuric acid (D) sodium chloride
Q.21 A flat coal seam of thickness (t) 3 m is excavated and broken roof rock has completely filled the
space created due to extraction as shown in the figure. If the bulking factor of roof rock is 1.2, the
caving height (H) in m is ______________
Q.22 A piece of coal sample weighs 10 kg in air and 2 kg when immersed in water. The specific gravity
of the coal sample is ___________
Q.23 In a borehole log of 1.2 m in length, recovery of rock cores in cm is given below
20, 8, 15, 8, 8, 4, 3, 9, 10, 1, 5, 10
The RQD in percentage is
(A) 29.2 (B) 31.8 (C) 45.8 (D) 50.0
Q.24 An underground coal mine panel produces 520 tonnes per day deploying 220, 200 and 192 persons
in three shifts. As per CMR 1957, the minimum quantity of air in m3
/min to be delivered at the last
ventilation connection of the panel is ____________
Q.25 In a PERT network, the activities on the critical path are a, b and c. The standard deviations of the
durations of these activities are 2, 2 and 1 respectively. The variance of the project duration is
(A) 3 (B) 5 (C) 9 (D) 12
H
t
void
space
MN 3/8
M
N
(
G
A
T
E
2
0
1
4
)
GATE 2014 MINING – MN
Q. 26 – Q. 55 carry two marks each.
Q.26 A particle P is in equilibrium as shown in the figure. The magnitude in kN and the orientation θ in
degrees of the force F respectively are
(A) 52.1, 16.1 (B) 221.2, 23.2 (C) 102.3, 53.4 (D) 180.3, 73.9
Q.27 A distributed load of 4 kN/m acts on a beam of 6 m length supported by a hinge and a roller as
shown in the figure. The distance in m of the point of zero shear in the beam from the point A is __
Q.28 A dry rock sample of diameter 50 mm and length 100 mm weighs 300 g. After saturating in brine
solution of specific gravity 1.05, its weight increased to 330 g. The porosity of the rock sample in
percentage is __________
Q.29 A joint plane of length L and dip δ intersects the toe of a slope as shown in the figure. The weight
of the shaded block is W. Uniform water pressure P acts normal to the joint plane. If the cohesion
and angle of internal friction of the joint surface are c and φ respectively, then the expression for
‘safety factor’ of the shaded block is
(A)
( sin ) tan
cos
Lc W LP
W
δ φ
δ
+ −
(B)
( cos ) tan
sin
Lc W LP
W
δ φ
δ
+ +
(C)
( cos ) tan
sin
Lc W LP
W
δ φ
δ
+ −
(D)
( sin ) tan
cos
Lc W LP
W
δ φ
δ
+ +
θ
F
200 kN
150 kN
30o
x
y
p
6 m
4 kN/m
A B
P
L
δ
W
Joint plane
MN 4/8
M
N
(
G
A
T
E
2
0
1
4
)
GATE 2014 MINING – MN
Q.30 The lengths and standard errors of three sections AB, BC, and CD of a straight line AD are given
below
AB = 125.85±0.021 m; BC= 205.72±0.029 m; CD=246.21±0.025 m
The standard error in total length AD in m is
(A) ±0.0436 (B) ±0.0350 (C) ±0.0250 (D) ±0.0019
Q.31 The bearing of side AB of a regular hexagon ABCDEF is 𝑆𝑆 500
10′
𝐸𝐸 . If the station C is easterly
from the station B, the whole circle bearing of the side BC is
(A) 650
15′
25′′
(B) 690
50′
25′′
(C) 690
15′
25′′
(D) 690
50′
0"
Q.32 In a room-and-pillar stope, bench blasting is conducted using ANFO having density of 800 kg/m3
.
The specific gravity of rock is 2.5, hole diameter is 100 mm and spacing to burden ratio is 1.3. The
charge length of each blast hole is 80% of the hole length. For a desired powder factor of 0.48
kg/tonne, the spacing and burden of the blast pattern in m respectively are
(A) 2.0, 2.6 (B) 2.3, 1.8 (C) 5.2, 4.0 (D) 1.3, 1.0
Q.33 Match the following for ore handling operations in an underground metal mine
Arrangement Description
(P) Drawpoint (I) arrangement that prevents oversized rock to pass
(Q) Ore pass (II) a system of vertical or near vertical openings for
transferring ore from a stope to a single delivery point
(R) Grizzly (III) a place where ore can be loaded and removed
(S) Finger raise (IV) a vertical or inclined opening used for transferring ore
(A) P-IV, Q-III, R-II, S-I (B) P-III, Q-IV, R-I, S-II
(C) P-II, Q-IV, R-I, S-III (D) P-III, Q-I, R-II, S-IV
Q.34 The following characteristic curves (P, Q, R, S) pertain to rotary drilling in rock.
Title of the curve
I: Torque versus RPM
II: Rate of penetration versus uniaxial compressive strength of rock
III: Rate of penetration versus weight on bit
IV: Specific energy versus weight on bit
Match the curves with their titles
(A) P-III, Q-IV, R-II, S-I (B) P-II, Q-IV, R-I, S-III
(C) P-IV, Q-III, R-II, S-I (D) P-I, Q-III, R-II, S-IV
P Q R S
x
y
x
y
x
y
x
y
MN 5/8
M
N
(
G
A
T
E
2
0
1
4
)
GATE 2014 MINING – MN
Q.35 The height H of a drawpoint in a sublevel caving stope is 3.0 m. If the angle of repose (𝜑𝜑) of
broken ore is 350
, the digging depth y of the loader as shown in the figure in m is _______
Q.36 For an explosives company, the probability of producing a defective detonator is 0.02. The
probability that a lot of 50 detonators produced by the company contains at most 2 defective
detonators is __________
Q.37 The area enclosed by the curves 2
y x
= and 3
y x
= for [ ]
0,
x∈ ∞ is
(A) 1/12 (B) 1/6 (C) 1/2 (D) 1
Q.38 The value of a, for which the function below is continuous at x = 1 is
2
2 , 1
( )
4 3, 1
x ax x
f x
x x
 + ≤
= 
+ >

(A) -5 (B) 0 (C) 5 (D) 10
Q.39 The sum of the infinite series 2 3 1
n
a ar ar ar ar −
+ + + + + +
 for 1
r < is
(A) ( )
1
a r
+ (B) ( )
1
a r
− (C)
1
a
r
+
(D)
1
a
r
−
Q.40 A centrifugal pump has a discharge rate of 2000 L of water per min against a total head of 200 m. If
the pump efficiency is 75%, the input power to the pump in kW is
(A) 87.20 (B) 49.05 (C) 13.33 (D) 7.50
Q.41 A dragline is required to remove 3,00,000 m3
of rock per month on the bank volume basis.
Consider the following data for the dragline operation.
Effective working hours per month = 450
Bucket fill factor = 0.8
Cycle time = 65 s
Swell factor of the rock = 1.25
The minimum bucket capacity of the dragline in m3
is
(A) 7.70 (B) 9.63 (C) 12.04 (D) 18.80
4
Ore Waste
rock
0
45
2
ϕ
−
ϕ
H
y
MN 6/8
M
N
(
G
A
T
E
2
0
1
4
)
GATE previous year paper 2007 to 2022 (1).pdf
GATE previous year paper 2007 to 2022 (1).pdf
GATE previous year paper 2007 to 2022 (1).pdf
GATE previous year paper 2007 to 2022 (1).pdf
GATE previous year paper 2007 to 2022 (1).pdf
GATE previous year paper 2007 to 2022 (1).pdf
GATE previous year paper 2007 to 2022 (1).pdf
GATE previous year paper 2007 to 2022 (1).pdf
GATE previous year paper 2007 to 2022 (1).pdf
GATE previous year paper 2007 to 2022 (1).pdf
GATE previous year paper 2007 to 2022 (1).pdf
GATE previous year paper 2007 to 2022 (1).pdf
GATE previous year paper 2007 to 2022 (1).pdf
GATE previous year paper 2007 to 2022 (1).pdf
GATE previous year paper 2007 to 2022 (1).pdf
GATE previous year paper 2007 to 2022 (1).pdf
GATE previous year paper 2007 to 2022 (1).pdf
GATE previous year paper 2007 to 2022 (1).pdf
GATE previous year paper 2007 to 2022 (1).pdf
GATE previous year paper 2007 to 2022 (1).pdf
GATE previous year paper 2007 to 2022 (1).pdf
GATE previous year paper 2007 to 2022 (1).pdf
GATE previous year paper 2007 to 2022 (1).pdf
GATE previous year paper 2007 to 2022 (1).pdf
GATE previous year paper 2007 to 2022 (1).pdf
GATE previous year paper 2007 to 2022 (1).pdf
GATE previous year paper 2007 to 2022 (1).pdf
GATE previous year paper 2007 to 2022 (1).pdf
GATE previous year paper 2007 to 2022 (1).pdf
GATE previous year paper 2007 to 2022 (1).pdf
GATE previous year paper 2007 to 2022 (1).pdf
GATE previous year paper 2007 to 2022 (1).pdf
GATE previous year paper 2007 to 2022 (1).pdf
GATE previous year paper 2007 to 2022 (1).pdf
GATE previous year paper 2007 to 2022 (1).pdf
GATE previous year paper 2007 to 2022 (1).pdf
GATE previous year paper 2007 to 2022 (1).pdf
GATE previous year paper 2007 to 2022 (1).pdf
GATE previous year paper 2007 to 2022 (1).pdf
GATE previous year paper 2007 to 2022 (1).pdf
GATE previous year paper 2007 to 2022 (1).pdf
GATE previous year paper 2007 to 2022 (1).pdf
GATE previous year paper 2007 to 2022 (1).pdf
GATE previous year paper 2007 to 2022 (1).pdf
GATE previous year paper 2007 to 2022 (1).pdf
GATE previous year paper 2007 to 2022 (1).pdf
GATE previous year paper 2007 to 2022 (1).pdf
GATE previous year paper 2007 to 2022 (1).pdf
GATE previous year paper 2007 to 2022 (1).pdf
GATE previous year paper 2007 to 2022 (1).pdf
GATE previous year paper 2007 to 2022 (1).pdf
GATE previous year paper 2007 to 2022 (1).pdf
GATE previous year paper 2007 to 2022 (1).pdf
GATE previous year paper 2007 to 2022 (1).pdf
GATE previous year paper 2007 to 2022 (1).pdf
GATE previous year paper 2007 to 2022 (1).pdf
GATE previous year paper 2007 to 2022 (1).pdf
GATE previous year paper 2007 to 2022 (1).pdf
GATE previous year paper 2007 to 2022 (1).pdf
GATE previous year paper 2007 to 2022 (1).pdf
GATE previous year paper 2007 to 2022 (1).pdf
GATE previous year paper 2007 to 2022 (1).pdf
GATE previous year paper 2007 to 2022 (1).pdf
GATE previous year paper 2007 to 2022 (1).pdf
GATE previous year paper 2007 to 2022 (1).pdf
GATE previous year paper 2007 to 2022 (1).pdf
GATE previous year paper 2007 to 2022 (1).pdf
GATE previous year paper 2007 to 2022 (1).pdf
GATE previous year paper 2007 to 2022 (1).pdf
GATE previous year paper 2007 to 2022 (1).pdf
GATE previous year paper 2007 to 2022 (1).pdf
GATE previous year paper 2007 to 2022 (1).pdf
GATE previous year paper 2007 to 2022 (1).pdf
GATE previous year paper 2007 to 2022 (1).pdf
GATE previous year paper 2007 to 2022 (1).pdf
GATE previous year paper 2007 to 2022 (1).pdf
GATE previous year paper 2007 to 2022 (1).pdf
GATE previous year paper 2007 to 2022 (1).pdf
GATE previous year paper 2007 to 2022 (1).pdf
GATE previous year paper 2007 to 2022 (1).pdf
GATE previous year paper 2007 to 2022 (1).pdf
GATE previous year paper 2007 to 2022 (1).pdf
GATE previous year paper 2007 to 2022 (1).pdf
GATE previous year paper 2007 to 2022 (1).pdf
GATE previous year paper 2007 to 2022 (1).pdf
GATE previous year paper 2007 to 2022 (1).pdf
GATE previous year paper 2007 to 2022 (1).pdf
GATE previous year paper 2007 to 2022 (1).pdf
GATE previous year paper 2007 to 2022 (1).pdf
GATE previous year paper 2007 to 2022 (1).pdf
GATE previous year paper 2007 to 2022 (1).pdf
GATE previous year paper 2007 to 2022 (1).pdf
GATE previous year paper 2007 to 2022 (1).pdf
GATE previous year paper 2007 to 2022 (1).pdf
GATE previous year paper 2007 to 2022 (1).pdf
GATE previous year paper 2007 to 2022 (1).pdf
GATE previous year paper 2007 to 2022 (1).pdf
GATE previous year paper 2007 to 2022 (1).pdf
GATE previous year paper 2007 to 2022 (1).pdf
GATE previous year paper 2007 to 2022 (1).pdf
GATE previous year paper 2007 to 2022 (1).pdf
GATE previous year paper 2007 to 2022 (1).pdf
GATE previous year paper 2007 to 2022 (1).pdf
GATE previous year paper 2007 to 2022 (1).pdf
GATE previous year paper 2007 to 2022 (1).pdf
GATE previous year paper 2007 to 2022 (1).pdf
GATE previous year paper 2007 to 2022 (1).pdf
GATE previous year paper 2007 to 2022 (1).pdf
GATE previous year paper 2007 to 2022 (1).pdf
GATE previous year paper 2007 to 2022 (1).pdf
GATE previous year paper 2007 to 2022 (1).pdf

More Related Content

What's hot

Longwall Mining
Longwall MiningLongwall Mining
Longwall Mining
VR M
 

What's hot (20)

Methane drainage
Methane drainageMethane drainage
Methane drainage
 
Dust in mine.
Dust in mine.Dust in mine.
Dust in mine.
 
Rock mass classification or rock mass rating of rock materials in civil and m...
Rock mass classification or rock mass rating of rock materials in civil and m...Rock mass classification or rock mass rating of rock materials in civil and m...
Rock mass classification or rock mass rating of rock materials in civil and m...
 
Wire ropes used in mining
Wire ropes used in miningWire ropes used in mining
Wire ropes used in mining
 
Mine cars
Mine carsMine cars
Mine cars
 
Drilling in Surface Mine.pdf
Drilling in Surface Mine.pdfDrilling in Surface Mine.pdf
Drilling in Surface Mine.pdf
 
Longwall Mining
Longwall MiningLongwall Mining
Longwall Mining
 
Lecture 4: Underground Mining
Lecture 4: Underground MiningLecture 4: Underground Mining
Lecture 4: Underground Mining
 
Shrinkage and Vertical Crater Retreat Stoping
Shrinkage and Vertical Crater Retreat StopingShrinkage and Vertical Crater Retreat Stoping
Shrinkage and Vertical Crater Retreat Stoping
 
Opencast mining machineries
Opencast mining machineriesOpencast mining machineries
Opencast mining machineries
 
Metal Mines Blasting
Metal Mines BlastingMetal Mines Blasting
Metal Mines Blasting
 
Types of ventilation system
Types of ventilation systemTypes of ventilation system
Types of ventilation system
 
locomotive Haulage presentation
locomotive Haulage  presentationlocomotive Haulage  presentation
locomotive Haulage presentation
 
Shaft sinking 1
Shaft sinking 1Shaft sinking 1
Shaft sinking 1
 
Rock Drilling and Types of Rock Drilling
Rock Drilling and Types of Rock DrillingRock Drilling and Types of Rock Drilling
Rock Drilling and Types of Rock Drilling
 
Subsidence
SubsidenceSubsidence
Subsidence
 
Bord and pillar working with LHD/SDL
Bord and pillar working with LHD/SDLBord and pillar working with LHD/SDL
Bord and pillar working with LHD/SDL
 
LHD Load haul dump ppt
LHD Load haul dump pptLHD Load haul dump ppt
LHD Load haul dump ppt
 
Standards of Mine Closure Planning
Standards of Mine Closure PlanningStandards of Mine Closure Planning
Standards of Mine Closure Planning
 
Surface miner
Surface minerSurface miner
Surface miner
 

Similar to GATE previous year paper 2007 to 2022 (1).pdf

gate__architecture_and_planning_question_paper.pdf_2018.pdf.pdf
gate__architecture_and_planning_question_paper.pdf_2018.pdf.pdfgate__architecture_and_planning_question_paper.pdf_2018.pdf.pdf
gate__architecture_and_planning_question_paper.pdf_2018.pdf.pdf
jyotikiran33
 
(Www.entrance exam.net)-hcl placement sample paper 3
(Www.entrance exam.net)-hcl placement sample paper 3(Www.entrance exam.net)-hcl placement sample paper 3
(Www.entrance exam.net)-hcl placement sample paper 3
ronnny_jain
 
Gate 2016-2018-production-and-industrial-engineering-question-paper-and-answe...
Gate 2016-2018-production-and-industrial-engineering-question-paper-and-answe...Gate 2016-2018-production-and-industrial-engineering-question-paper-and-answe...
Gate 2016-2018-production-and-industrial-engineering-question-paper-and-answe...
nasitravi41
 

Similar to GATE previous year paper 2007 to 2022 (1).pdf (20)

gate__architecture_and_planning_question_paper.pdf_2018.pdf.pdf
gate__architecture_and_planning_question_paper.pdf_2018.pdf.pdfgate__architecture_and_planning_question_paper.pdf_2018.pdf.pdf
gate__architecture_and_planning_question_paper.pdf_2018.pdf.pdf
 
Cat sample-1
Cat sample-1Cat sample-1
Cat sample-1
 
Gate 2014 2
Gate 2014 2Gate 2014 2
Gate 2014 2
 
Pi s7-2016-p(gate2016.info)
Pi s7-2016-p(gate2016.info)Pi s7-2016-p(gate2016.info)
Pi s7-2016-p(gate2016.info)
 
APTITUDE OF ESE 2020.pdf
APTITUDE OF ESE 2020.pdfAPTITUDE OF ESE 2020.pdf
APTITUDE OF ESE 2020.pdf
 
Ugc net solutions at target ies
Ugc net solutions at target iesUgc net solutions at target ies
Ugc net solutions at target ies
 
(Www.entrance exam.net)-hcl placement sample paper 3
(Www.entrance exam.net)-hcl placement sample paper 3(Www.entrance exam.net)-hcl placement sample paper 3
(Www.entrance exam.net)-hcl placement sample paper 3
 
Gate 16--ce-s2 0
Gate 16--ce-s2 0Gate 16--ce-s2 0
Gate 16--ce-s2 0
 
imo sample papers
imo sample papersimo sample papers
imo sample papers
 
GATE-ec-question-Paper-2018.pdf-82.pdf
GATE-ec-question-Paper-2018.pdf-82.pdfGATE-ec-question-Paper-2018.pdf-82.pdf
GATE-ec-question-Paper-2018.pdf-82.pdf
 
Pi gate-2018-p (gate2016.info)
Pi gate-2018-p (gate2016.info)Pi gate-2018-p (gate2016.info)
Pi gate-2018-p (gate2016.info)
 
Mcqs first year physics notes
Mcqs first year physics notesMcqs first year physics notes
Mcqs first year physics notes
 
Gate 2016-2018-production-and-industrial-engineering-question-paper-and-answe...
Gate 2016-2018-production-and-industrial-engineering-question-paper-and-answe...Gate 2016-2018-production-and-industrial-engineering-question-paper-and-answe...
Gate 2016-2018-production-and-industrial-engineering-question-paper-and-answe...
 
Gate-Cs 2009
Gate-Cs 2009Gate-Cs 2009
Gate-Cs 2009
 
Gate-Cs 2010
Gate-Cs 2010Gate-Cs 2010
Gate-Cs 2010
 
CXC MATHEMATICS MULTIPLE CHOICE
CXC MATHEMATICS MULTIPLE CHOICECXC MATHEMATICS MULTIPLE CHOICE
CXC MATHEMATICS MULTIPLE CHOICE
 
GATE 2019 Mechanical Set 21
GATE 2019 Mechanical Set 21GATE 2019 Mechanical Set 21
GATE 2019 Mechanical Set 21
 
Ssc1e
Ssc1eSsc1e
Ssc1e
 
Nift pg technology sample question paper gat
Nift pg technology sample question paper gatNift pg technology sample question paper gat
Nift pg technology sample question paper gat
 
Practice sheet 5
Practice sheet 5Practice sheet 5
Practice sheet 5
 

Recently uploaded

Final DBMS Manual (2).pdf final lab manual
Final DBMS Manual (2).pdf final lab manualFinal DBMS Manual (2).pdf final lab manual
Final DBMS Manual (2).pdf final lab manual
BalamuruganV28
 

Recently uploaded (20)

NEWLETTER FRANCE HELICES/ SDS SURFACE DRIVES - MAY 2024
NEWLETTER FRANCE HELICES/ SDS SURFACE DRIVES - MAY 2024NEWLETTER FRANCE HELICES/ SDS SURFACE DRIVES - MAY 2024
NEWLETTER FRANCE HELICES/ SDS SURFACE DRIVES - MAY 2024
 
Insurance management system project report.pdf
Insurance management system project report.pdfInsurance management system project report.pdf
Insurance management system project report.pdf
 
Diploma Engineering Drawing Qp-2024 Ece .pdf
Diploma Engineering Drawing Qp-2024 Ece .pdfDiploma Engineering Drawing Qp-2024 Ece .pdf
Diploma Engineering Drawing Qp-2024 Ece .pdf
 
NO1 Best Powerful Vashikaran Specialist Baba Vashikaran Specialist For Love V...
NO1 Best Powerful Vashikaran Specialist Baba Vashikaran Specialist For Love V...NO1 Best Powerful Vashikaran Specialist Baba Vashikaran Specialist For Love V...
NO1 Best Powerful Vashikaran Specialist Baba Vashikaran Specialist For Love V...
 
Circuit Breakers for Engineering Students
Circuit Breakers for Engineering StudentsCircuit Breakers for Engineering Students
Circuit Breakers for Engineering Students
 
CLOUD COMPUTING SERVICES - Cloud Reference Modal
CLOUD COMPUTING SERVICES - Cloud Reference ModalCLOUD COMPUTING SERVICES - Cloud Reference Modal
CLOUD COMPUTING SERVICES - Cloud Reference Modal
 
Theory of Time 2024 (Universal Theory for Everything)
Theory of Time 2024 (Universal Theory for Everything)Theory of Time 2024 (Universal Theory for Everything)
Theory of Time 2024 (Universal Theory for Everything)
 
Developing a smart system for infant incubators using the internet of things ...
Developing a smart system for infant incubators using the internet of things ...Developing a smart system for infant incubators using the internet of things ...
Developing a smart system for infant incubators using the internet of things ...
 
Involute of a circle,Square, pentagon,HexagonInvolute_Engineering Drawing.pdf
Involute of a circle,Square, pentagon,HexagonInvolute_Engineering Drawing.pdfInvolute of a circle,Square, pentagon,HexagonInvolute_Engineering Drawing.pdf
Involute of a circle,Square, pentagon,HexagonInvolute_Engineering Drawing.pdf
 
Maximizing Incident Investigation Efficacy in Oil & Gas: Techniques and Tools
Maximizing Incident Investigation Efficacy in Oil & Gas: Techniques and ToolsMaximizing Incident Investigation Efficacy in Oil & Gas: Techniques and Tools
Maximizing Incident Investigation Efficacy in Oil & Gas: Techniques and Tools
 
Software Engineering Practical File Front Pages.pdf
Software Engineering Practical File Front Pages.pdfSoftware Engineering Practical File Front Pages.pdf
Software Engineering Practical File Front Pages.pdf
 
Passive Air Cooling System and Solar Water Heater.ppt
Passive Air Cooling System and Solar Water Heater.pptPassive Air Cooling System and Solar Water Heater.ppt
Passive Air Cooling System and Solar Water Heater.ppt
 
Raashid final report on Embedded Systems
Raashid final report on Embedded SystemsRaashid final report on Embedded Systems
Raashid final report on Embedded Systems
 
Instruct Nirmaana 24-Smart and Lean Construction Through Technology.pdf
Instruct Nirmaana 24-Smart and Lean Construction Through Technology.pdfInstruct Nirmaana 24-Smart and Lean Construction Through Technology.pdf
Instruct Nirmaana 24-Smart and Lean Construction Through Technology.pdf
 
Artificial Intelligence in due diligence
Artificial Intelligence in due diligenceArtificial Intelligence in due diligence
Artificial Intelligence in due diligence
 
SLIDESHARE PPT-DECISION MAKING METHODS.pptx
SLIDESHARE PPT-DECISION MAKING METHODS.pptxSLIDESHARE PPT-DECISION MAKING METHODS.pptx
SLIDESHARE PPT-DECISION MAKING METHODS.pptx
 
Research Methodolgy & Intellectual Property Rights Series 1
Research Methodolgy & Intellectual Property Rights Series 1Research Methodolgy & Intellectual Property Rights Series 1
Research Methodolgy & Intellectual Property Rights Series 1
 
History of Indian Railways - the story of Growth & Modernization
History of Indian Railways - the story of Growth & ModernizationHistory of Indian Railways - the story of Growth & Modernization
History of Indian Railways - the story of Growth & Modernization
 
Final DBMS Manual (2).pdf final lab manual
Final DBMS Manual (2).pdf final lab manualFinal DBMS Manual (2).pdf final lab manual
Final DBMS Manual (2).pdf final lab manual
 
Independent Solar-Powered Electric Vehicle Charging Station
Independent Solar-Powered Electric Vehicle Charging StationIndependent Solar-Powered Electric Vehicle Charging Station
Independent Solar-Powered Electric Vehicle Charging Station
 

GATE previous year paper 2007 to 2022 (1).pdf

  • 1. MINING NOVEL GATE PREVIOS QUESTION PAPERS Mining Novel is an online education platform that gives Test series. Preparing for GATE, Coal India Subsidiaries and All Mining Exams... 2022 MINING NOVEL
  • 2. Welcome To Mining Novel !! Mining Novel is an online education platform that gives free lecture videos by combining the most enthusiastic teachers, engaging content, and Test series. Preparing for GATE, Coal India Subsidiaries and All Mining Examinations. We provide you Test series and Structured classes with a powerful study plan that will help you prepare with confidence. Mining Novel is an inbound store for all MINING things – Mining sirdar/ Overman exams Preparation Tips, Strategies, Motivation, Important Updates, Free Notes, and much more that will astonish you. Our experts are committed to giving the best guidance for All Mining Exam preparation by analyzing the crucial needs of the students. So, Be a Part and begin your “Amazing journey”!! We believe that education can change everyone’s life and we want everyone to have the opportunity. We just want to take out a few minutes of your everyday life, and these moments will do wonders! Tune in and Be Inspired!!
  • 3. GATE PREVIOS QUESTION PAPERS GATE PREVIOS QUESTION PAPERS GATE PREVIOS QUESTION PAPERS (2022 to 2007)
  • 4. Page 1 GATE 2022 General Aptitude (GA) Q.1 – Q.5 Carry ONE mark each. Q.1 Mr. X speaks _________ Japanese _________ Chinese. (A) neither / or (B) either / nor (C) neither / nor (D) also / but Q.2 A sum of money is to be distributed among P, Q, R, and S in the proportion 5 : 2 : 4 : 3, respectively. If R gets ₹ 1000 more than S, what is the share of Q (in ₹)? (A) 500 (B) 1000 (C) 1500 (D) 2000
  • 5. Page 2 Q.3 A trapezium has vertices marked as P, Q, R and S (in that order anticlockwise). The side PQ is parallel to side SR. Further, it is given that, PQ = 11 cm, QR = 4 cm, RS = 6 cm and SP = 3 cm. What is the shortest distance between PQ and SR (in cm)? (A) 1.80 (B) 2.40 (C) 4.20 (D) 5.76
  • 6. Page 3 Q.4 The figure shows a grid formed by a collection of unit squares. The unshaded unit square in the grid represents a hole. What is the maximum number of squares without a “hole in the interior” that can be formed within the 4 × 4 grid using the unit squares as building blocks? (A) 15 (B) 20 (C) 21 (D) 26
  • 7. Page 4 Q.5 An art gallery engages a security guard to ensure that the items displayed are protected. The diagram below represents the plan of the gallery where the boundary walls are opaque. The location the security guard posted is identified such that all the inner space (shaded region in the plan) of the gallery is within the line of sight of the security guard. If the security guard does not move around the posted location and has a 360o view, which one of the following correctly represents the set of ALL possible locations among the locations P, Q, R and S, where the security guard can be posted to watch over the entire inner space of the gallery. (A) P and Q (B) Q (C) Q and S (D) R and S
  • 8. Page 5 Q. 6 – Q. 10 Carry TWO marks each. Q.6 Mosquitoes pose a threat to human health. Controlling mosquitoes using chemicals may have undesired consequences. In Florida, authorities have used genetically modified mosquitoes to control the overall mosquito population. It remains to be seen if this novel approach has unforeseen consequences. Which one of the following is the correct logical inference based on the information in the above passage? (A) Using chemicals to kill mosquitoes is better than using genetically modified mosquitoes because genetic engineering is dangerous (B) Using genetically modified mosquitoes is better than using chemicals to kill mosquitoes because they do not have any side effects (C) Both using genetically modified mosquitoes and chemicals have undesired consequences and can be dangerous (D) Using chemicals to kill mosquitoes may have undesired consequences but it is not clear if using genetically modified mosquitoes has any negative consequence
  • 9. Page 6 Q.7 Consider the following inequalities. (i) 2𝑥 − 1 > 7 (ii) 2𝑥 − 9 < 1 Which one of the following expressions below satisfies the above two inequalities? (A) 𝑥 ≤ −4 (B) −4 < 𝑥 ≤ 4 (C) 4 < 𝑥 < 5 (D) 𝑥 ≥ 5 Q.8 Four points P(0, 1), Q(0, −3), R(−2, −1), and S(2, −1) represent the vertices of a quadrilateral. What is the area enclosed by the quadrilateral? (A) 4 (B) 4 2 (C) 8 (D) 8√2
  • 10. Page 7 Q.9 In a class of five students P, Q, R, S and T, only one student is known to have copied in the exam. The disciplinary committee has investigated the situation and recorded the statements from the students as given below. Statement of P: R has copied in the exam. Statement of Q: S has copied in the exam. Statement of R: P did not copy in the exam. Statement of S: Only one of us is telling the truth. Statement of T: R is telling the truth. The investigating team had authentic information that S never lies. Based on the information given above, the person who has copied in the exam is (A) R (B) P (C) Q (D) T
  • 11. Page 8 Q.10 Consider the following square with the four corners and the center marked as P, Q, R, S and T respectively. Let X, Y and Z represent the following operations: X: rotation of the square by 180 degree with respect to the S-Q axis. Y: rotation of the square by 180 degree with respect to the P-R axis. Z: rotation of the square by 90 degree clockwise with respect to the axis perpendicular, going into the screen and passing through the point T. Consider the following three distinct sequences of operation (which are applied in the left to right order). (1) XYZZ (2) XY (3) ZZZZ Which one of the following statements is correct as per the information provided above? (A) The sequence of operations (1) and (2) are equivalent (B) The sequence of operations (1) and (3) are equivalent (C) The sequence of operations (2) and (3) are equivalent (D) The sequence of operations (1), (2) and (3) are equivalent
  • 12. GATE 2022 Mining Engineering (MN) Page 9 Q.11 – Q.35 Carry ONE mark Each Q.11 The value of lim → ( ) is (A) 0 (B) 1 (C) −𝑛 (D) 𝑛 Q.12 A velocity field in Cartesian coordinate system is expressed as 𝒗 = 𝑥𝚤̂ + 𝑦𝚥̂ + 𝑝(𝑧)𝑘, where 𝑝(0) = 0 If div 𝒗 = 0, 𝑝(𝑧) is (A) 0 (B) −2𝑧 (C) 2 (D) 2𝑧
  • 13. GATE 2022 Mining Engineering (MN) Page 10 Q.13 The constant term of the Fourier coefficients of the periodic function 𝑓(𝑥) = −𝑘, for − 𝜋 < 𝑥 < 0 𝑘, for 0 < 𝑥 < 𝜋 , 𝑓(𝑥 + 2𝜋) = 𝑓(𝑥) and 𝑘 = 𝑐𝑜𝑛𝑠𝑡𝑎𝑛𝑡 is (A) 𝑘 (B) 2𝑘 (C) 2𝜋 (D) 0 Q.14 Two vectors x and y are shown in the figure. The projection vector of x on y is (A) 𝒙 𝒚 𝒚 𝒚 𝒚 (B) 𝒙 × 𝒚 (C) 𝒙 × 𝒚 𝒚 𝒚 (D) 𝒙 𝒚 𝒙 𝒙 𝒙
  • 14. GATE 2022 Mining Engineering (MN) Page 11 Q.15 A deposit has the grade attribute X ∈ [0, 30] with a density function 𝑓(𝑥). For a cut-off grade 𝑥 , the proportion of the ore in the deposit is given by (A) 𝑓(𝑥)𝑑𝑥 − 𝑓(𝑥)𝑑𝑥 (B) 1 2 𝑓(𝑥)𝑑𝑥 − 𝑓(𝑥)𝑑𝑥 (C) 1 2 𝑓(𝑥)𝑑𝑥 + 𝑓(𝑥)𝑑𝑥 (D) 𝑓(𝑥)𝑑𝑥 Q.16 The drilling technique applicable for mineral exploration is (A) Percussive drilling (B) Tricone roller drilling (C) Rotary-percussive drilling (D) Diamond core drilling
  • 15. GATE 2022 Mining Engineering (MN) Page 12 Q.17 Match the rock with its metamorphosed form Igneous/ Sedimentary rock Metamorphic rock P Granite I Quartzite Q Limestone II Gneiss R Sandstone III Schist S Gabbro IV Marble (A) P-II, Q-IV, R-I, S-III (B) P-III, Q-I, R-IV, S-II (C) P-IV, Q-III, R-I, S-II (D) P-II, Q-III, R-IV, S-I Q.18 Identify the WRONG statement: Break-even stripping ratio (A) takes into account the maximum pit slope that is safe (B) helps in determining the volume of the overburden (C) presents the maximum possible mine size that is economical (D) takes into account the life of the mine
  • 16. GATE 2022 Mining Engineering (MN) Page 13 Q.19 A square pattern of blasting is shown in the figure. For the case of simultaneous blast, identify the zone of no fragmentation (A) P (B) Q (C) R (D) S
  • 17. GATE 2022 Mining Engineering (MN) Page 14 Q.20 In the truss shown in the figure, the force in the member BD, in kN is_____. (A) 7 (B) 5 (C) 2 (D) 0
  • 18. GATE 2022 Mining Engineering (MN) Page 15 Q.21 The correct vertical stress profile in the case of tributary area method for pillar design is (A) I (B) II (C) III (D) IV
  • 19. GATE 2022 Mining Engineering (MN) Page 16 Q.22 The bottom section of a stoping block has dimensions 200 m × 40 m. If the modified RMR of rock mass is 50, the appropriate method of mining on the basis of Laubscher’s chart in the figure is (A) Shrinkage stoping (B) Cut and fill (C) Block Caving (D) Sublevel stoping
  • 20. GATE 2022 Mining Engineering (MN) Page 17 Q.23 Match the machine with its component. Machine Component P Continuous miner I Flight bar Q Jack hammer II Lemniscate link R AFC III Loading apron S Shield support IV Rifle bar (A) P-III, Q-IV, R-I, S-II (B) P-IV, Q-III, R-I, S-II (C) P-III, Q-IV, R-II, S-I (D) P-IV, Q-III, R-II, S-I Q.24 Which one of the following is NOT a notifiable disease as per Indian mining legislation? (A) Silicosis (B) Noise induced hearing loss (C) Nystagmus (D) Asbestosis
  • 21. GATE 2022 Mining Engineering (MN) Page 18 Q.25 If the ambient lapse rate is higher than the dry adiabatic lapse rate, the atmosphere is (A) stable (B) neutral (C) unstable (D) strongly stable Q.26 Identify the WRONG statement: The application of controlled air recirculation in an underground work place can (A) increase the air velocity at the work place (B) lead to increased concentration of contaminants in the work place (C) require the installation of an additional fan in the system (D) lead to overall ventilation cost savings
  • 22. GATE 2022 Mining Engineering (MN) Page 19 Q.27 The correct order of pavement layers for a haul road from top to bottom is (A) Wearing course → Base → Sub base → Sub grade (B) Wearing course → Sub base → Base → Sub grade (C) Wearing course → Sub grade → Sub base → Base (D) Wearing course → Base → Sub grade → Sub base Q.28 A mining company produces iron ore and sells to another company. Royalty to be paid is on the basis of (A) quantity of ore produced (B) quantity of ore sold (C) difference between the quantities of ore produced and sold (D) net profit Q.29 The cost of a screw compressor with an estimated life of 15 years is ₹21,00,000. If the depreciation of the compressor charged, using ‘sum-of-the-years-digits’ method, at the end of 4th year is ₹2,00,000, the salvage value, in ₹ is_______. (round off to one decimal place)
  • 23. GATE 2022 Mining Engineering (MN) Page 20 Q.30 A safety device consists of two independent critical components 𝑋 and 𝑋 . The failure of any one or both of these components can cause an accident. The failure probabilities of components 𝑋 and 𝑋 are 0.2 and 0.1, respectively. The probability of occurrence of an accident is_______. (round off to two decimal places) Q.31 In a levelling survey, a reading is taken as 2.25 m. However, along the line of sight there is deflection of 20 cm with respect to vertical position of the staff. The correct reading, in m is_______. (round off to two decimal places) Q.32 Water flows through a vertical sand column of cross sectional area 4000 mm2 and length 300 mm. For a water head of 600 mm, quantity of seepage water is 100 mm3/min. The hydraulic conductivity of the sand column, in mm/min is_______. (round off to three decimal places)
  • 24. GATE 2022 Mining Engineering (MN) Page 21 Q.33 The modified Lauffer diagram as shown in the figure relates to roof span, RMR and stand-up time. In a metal mine, roof span of a drive is 4 m. If the RMR of the rock mass changes from 40 to 60, then the stand-up time increases by a factor of________. (round off to two decimal places)
  • 25. GATE 2022 Mining Engineering (MN) Page 22 Q.34 In a friction winder, the skip accelerates to a steady speed over a time span of 15 s from the start. The torque vs. time diagram for the winding cycle is shown in the figure. The deceleration time in seconds is_______. (round off to one decimal place) Q.35 At a measurement station, the air quality parameters PM2.5, NO2 and O3 have the AQI sub-index values as 180, 96, and 84, respectively. The AQI for the station is_______. (in integer)
  • 26. GATE 2022 Mining Engineering (MN) Page 23 Q.36 – Q.65 Carry TWO marks Each Q.36 Match the drilling pattern with mining operation Drilling Pattern Mining Operation P I Drifting Q II Shaft Sinking R III Sublevel stoping S IV Drop Raising (A) P→II, Q→III, R→I, S→II (B) P→III, Q→IV, R→I, S→II (C) P→II, Q→I, R→IV, S→III (D) P→III, Q→IV, R→II, S→I
  • 27. GATE 2022 Mining Engineering (MN) Page 24 Q.37 The closest match of the scatter plot between the variables X and Y with the approximate attribute is (A) P→I, Q→II, R→III, S→IV (B) P→II, Q→I, R→IV, S→III (C) P→III, Q→IV, R→I, S→II (D) P→IV, Q→II, R→III, S→I
  • 28. GATE 2022 Mining Engineering (MN) Page 25 Q.38 A 3-point borehole extensometer is installed to identify the location of a single discontinuity plane in a hanging wall rock by measuring deformations at three locations as shown in the figure. The absolute readings of deformations measured on two different dates are listed in the table. Based on the measured data the most likely inference is Date Anchor -1 (mm) Anchor -2 (mm) Anchor -3 (mm) May 2, 2021 (initial reading) 34.52 29.04 43.11 June 1, 2021 40.56 34.67 44.78 (A) Discontinuity between Anchor-1 and Anchor-2 (B) Discontinuity between Anchor-2 and Anchor-3 (C) Discontinuity between Anchor-3 and excavation boundary (D) No noticeable discontinuity
  • 29. GATE 2022 Mining Engineering (MN) Page 26 Q.39 Match the semi-variogram shape with the model name and the property (A) P→III→E, Q→II→F, R→IV→E, S→I→G (B) P→II→F, Q→I→G, R→III→E, S→IV→E (C) P→IV→G, Q→III→F, R→II→E, S→I→E (D) P→II→E, Q→I→E, R→III→F, S→IV→G
  • 30. GATE 2022 Mining Engineering (MN) Page 27 Q.40 In a uniaxial compressive strength test, a rock sample of diameter 50 mm fails at an angle of 60o as shown in the figure. If the peak load at failure is 120 kN, the normal and shear stresses on failure plane respectively, in MPa are_______ and _____. (A) 15.28 and 26.46 (B) 26.46 and 15.28 (C) 57.02 and -15.28 (D) -15.28 and 15.28
  • 31. GATE 2022 Mining Engineering (MN) Page 28 Q.41 A coal mining company examines the option of buying two types of dumpers with the following details. Parameter Type-1 Type-2 Constraints Capital cost per dumper (in ₹ crore) 3.0 4.0 Maximum capital available for purchasing is ₹120 crore Capacity in tonne 40 50 Minimum daily tonnage to be hauled is 31,000 Daily trips for each dumper 20 20 Operating cost (in ₹) per tonne 300.0 200.0 In order to minimize the operating cost, the optimum fleet of dumpers of Type-1 and Type-2, respectively are (A) 20, 15 (B) 0, 30 (C) 0, 31 (D) 40, 0
  • 32. GATE 2022 Mining Engineering (MN) Page 29 Q.42 Let 𝑓(𝑥) be a continuous and differentiable function on [3, 18]. If 𝑓(3) = −50 and 𝑓′(𝑥) ≤ 20, then the largest possible value of 𝑓(18), is_______. (in integer) Q.43 Let ∝ (𝑇 − 𝑇), where 𝑇 and 𝑇 are temperatures in degree centigrade of a room and thermometer, respectively, and t denotes time in minutes. A thermometer at a reading of 2 C is brought in a room of temperature 40 C. Two minutes later, the thermometer reads 15 C. The time elapsed in minutes when the thermometer reads 39.5 C, is_______. (round off to two decimal places) Q.44 In a health centre, the probability of ‘full occupancy’ of COVID beds for a day is 0.8. Assuming Binomial probability distribution, the probability of full occupancy exactly for 5 days in a week, is_______. (round off to three decimal places) Q.45 Following information is given for a drilling operation to be carried out for overburden removal in a surface mine. Volume of rock blasted per round, m3 : 3,20,000 Number of blast holes : 100 Drill hole diameter (D), mm : 200 Length of subgrade drilling : 8D Stemming length : 25D Bench height, m : 30 Powder factor, m3/kg : 3.2 The amount of explosive per unit length of charge in kg/m, is_______. (round off to two decimal places)
  • 33. GATE 2022 Mining Engineering (MN) Page 30 Q.46 The shaft-top coordinates of two vertical shafts are given below. The depth of the shaft A and B are 200 m and 149 m, respectively. Shaft Latitude (m) Departure (m) Surface RL (m) A N670.0 W150.0 250 B N170.0 E50.0 209 The downward gradient of the line joining the bottom of the two shafts in degrees, is_______. (round off to two decimal places) Q.47 The oxygen-balanced equation for explosive ANFO is given below. 3𝑁𝐻 𝑁𝑂 + 𝐶𝐻 → 7𝐻 𝑂 + 𝐶𝑂 + 3𝑁 For 100 litre of fuel oil having density 850 kg/m3, the amount of ammonium nitrate to be mixed, in kg, is_______. (round off to two decimal places) Q.48 Two weightless cables of equal length and cross sectional area are hanging from a ceiling as shown in the figure. They are connected by a horizontal light bar of length 1.0 m and pulled by a force, F. The modulus of elasticity of Cable-1 and Cable-2 are 50 GPa and 200 GPa, respectively. If the deformation in both the cables is equal, the distance 𝑥, in m is_______. (round off to one decimal place)
  • 34. GATE 2022 Mining Engineering (MN) Page 31 Q.49 A circular tunnel of radius 3 m is constructed in a hydrostatic stress field of 15 MPa. The modulus of elasticity and Poisson’s ratio of the rock are 5 GPa and 0.25, respectively. A uniform support pressure 𝑝 is applied at tunnel boundary to restrict the radial deformation at the tunnel boundary to 4 mm. The value of 𝑝 in MPa is_______. (round off to two decimal places) Q.50 The extraction ratio during development of a bord and pillar panel is 0.15 in a flat coal seam. The panel is further extracted by widening the galleries, and the extraction ratio changes to 0.25. The percentage change in pillar stress, considering tributary area method, is_______. (round off to two decimal places) Q.51 In a small metal mine a battery powered locomotive hauls a train of mine tubs such that: The weight of the train of mine tubs, tonne : 3.0 The coefficient of friction between the wheels and the rails : 0.06 The coefficient of adhesion between the loco wheels and the rails : 0.2 Time required from the start to reach speed of 1.8 m/s through constant acceleration, min : 3.0 Upward gradient to be negotiated : 1 in 20 The minimum weight of the locomotive in tonnes to meet these design requirements, is_______. (round off to one decimal place) Q.52 A pump lifts mine water of density 1020 kg/m3, at 250 m3/hr from a depth of 150 m. The overall pumping efficiency is 68%. Considering a head loss of 15 m due to pipe friction and shock, the motor input power, in kW is_______. (round off to two decimal places)
  • 35. GATE 2022 Mining Engineering (MN) Page 32 Q.53 In a surface mine bench, overburden is removed by the shovel-dumper combination. For the dumper: Time required at the loading station : 3.0 min Time required at the unloading station : 1.0 min Distance between loading and unloading stations : 4.5 km Average speed during loaded travel : 12.0 km/hr Average speed during empty travel : 18.0 km/hr Minimum number of dumpers required to avoid idle time of the shovel, is _______. (in integer) Q.54 In a bord and pillar development panel, headings of 4.4 m × 2.5 m are advanced using solid blasting. The average pull per round of blast is 1.2 m. On an average 12 faces are blasted per day. Density of coal is 1500 kg/m3. The mine operates in three shifts. If the average daily employment is 330 persons, labour productivity (OMS) of the panel in tonne, is_______. (round off to two decimal places) Q.55 In a longwall face, the full seam thickness of 3 m is cut by a shearer with a web of depth 0.7 m. The hauling speed of shearer during cutting is 12 m/min. The trough cross-section of AFC is 0.4 m2 and the average loading coefficient is 0.7. In order to evacuate coal from the face without spillage, the speed of AFC, in m/s is_______. (round off to one decimal place) Q.56 A city is spread over an area of 20 km × 40 km. Wind, at an average speed of 4 m/s, enters perpendicular to the 20 km long side. On a winter day the inversion layer exists over the city at a height of 100 m. PM1 is emitted from the city at a rate of 1 kg/s. The steady state PM1 concentration in the city air, in µg/m3, assuming Box model, is_______. (round off to one decimal place)
  • 36. GATE 2022 Mining Engineering (MN) Page 33 Q.57 The point ‘A’ as shown in the Coward flammability diagram represents the gas composition of a sealed-off area of a coal mine. The volume of the sealed-off area is 10000 m3. Inert gas is proposed to be injected into the sealed-off area so that the gas composition comes below the LEL (lower explosibility limit). The minimum volume of the inert gas required (at the same pressure as that of the sealed-off area), in m3 is_______. (round off to one decimal place) Q.58 An underground workshop has dimensions of 8 m length, 6 m width and 4 m height. Four identical luminaires are placed at the four corners of the roof. Each luminaire is of 100 W capacity with luminous efficacy of 100 lumen/W. Light is transmitted spherically from luminaires and there are no reflections. The illumination on the horizontal plane at the centre of the floor, in lux is_______. (round off to two decimal places) Q.59 An underground AC plant requires the delivery of 250 US gpm (15.85 US gpm = 1.0 lps) of chilled water. For this purpose, ice-pellets at 0 0C temperature (latent heat of melting, 334 kJ/kg) are mixed with water at 20 0C (specific heat 4.18 kJ/kg0C) on the surface. The mixture is adiabatically transported to the underground location such that the water at 7 0C becomes available for the AC plant. The requirement of ice-pellets in tonne/hr to meet the design condition, is ______. (round off to two decimal places)
  • 37. GATE 2022 Mining Engineering (MN) Page 34 Q.60 An intake shaft has resistance of 0.05 Ns2/m8 up to a depth of 400 m. The airflow rate is 100 m3/s and the average density is 1.2 kg/m3. A barometer reads 99.375 kPa when placed on surface. Considering acceleration due to gravity is 9.81 m/s2, the reading of the barometer at the depth of 400 m, in kPa is_______. (round off to two decimal places) Q.61 The net present values (NPV) of two mining project proposals A and B are as given. 𝑁𝑃𝑉 = −0.01𝑖 − 0.02𝑖 + 4.44 𝑁𝑃𝑉 = −0.03𝑖 − 0.01𝑖 + 6.55 where, 𝑖 is discount rate. The required rate of return for which both the proposals have equal possibility of acceptance and rejection, is _______. (round off to two decimal places) Q.62 The value of ∫ 𝑥𝑙𝑜𝑔(1 + 𝑥)𝑑𝑥, is_______. (round off to two decimal places) Q.63 A coal seam of uniform thickness 12 m is dipping at an angle 30o as shown in the figure. The ultimate pit is demarcated based on allowable instantaneous stripping ratio of 10 m3/tonne and safe slope angle of 45o. The density of coal is 1.41 tonne/m3. The length, L in m is_______. (round off to two decimal places)
  • 38. GATE 2022 Mining Engineering (MN) Page 35 Q.64 A mine has a reserve of 150 million tonne (Mt) and is designed for a maximum production capacity of 5 Mt per year. In the first year the production is 2 Mt and it increases by 20% each year. The reserve in Mt that remains at the end of 15 years, is_______. (round off to two decimal places) Q.65 Information on Activity-Time duration of a project is provided below Activity Predecessor event Successor event Estimated Time Duration (weeks) Pessimistic Most likely Optimistic A 1 2 20 15 4 B 1 3 12 8 4 C 2 3 16 11 6 D 3 4 20 13 12 E 2 4 13 8 3 F 1 4 45 35 25 The expected project duration in weeks, is_______. (in integer)
  • 39. Q. No. Session Question Type Subject Name Key/Range Mark 1 3 MCQ GA C 1 2 3 MCQ GA D 1 3 3 MCQ GA B 1 4 3 MCQ GA B 1 5 3 MCQ GA C 1 6 3 MCQ GA D 2 7 3 MCQ GA C 2 8 3 MCQ GA C 2 9 3 MCQ GA B 2 10 3 MCQ GA B 2 11 3 MCQ MN C 1 12 3 MCQ MN B 1 13 3 MCQ MN D 1 14 3 MCQ MN A 1 15 3 MCQ MN A 1 16 3 MCQ MN D 1 17 3 MCQ MN A 1 18 3 MCQ MN D 1 19 3 MCQ MN C 1 20 3 MCQ MN D 1 21 3 MCQ MN C 1 22 3 MCQ MN B 1 23 3 MCQ MN A 1 24 3 MCQ MN C 1 25 3 MCQ MN C 1 26 3 MCQ MN B 1 27 3 MCQ MN A 1 28 3 MCQ MN B 1 29 3 NAT MN 100000.0 to 100000.0 1 30 3 NAT MN 0.28 to 0.28 1 31 3 NAT MN 2.20 to 2.30 1 32 3 NAT MN 0.012 to 0.013 1 33 3 NAT MN 50.00 to 60.00 1 34 3 NAT MN 10.0 to 10.0 1 35 3 NAT MN 180 to 180 1 36 3 MCQ MN D 2 37 3 MCQ MN D 2 38 3 MCQ MN B 2 39 3 MCQ MN A 2 40 3 MCQ MN A 2 41 3 MCQ MN A 2 42 3 NAT MN 250 to 250 2 43 3 NAT MN 20.40 to 20.80 2 44 3 NAT MN 0.251 to 0.299 2
  • 40. 45 3 NAT MN 36.00 to 39.00 2 46 3 NAT MN 0.90 to 1.13 2 47 3 NAT MN 1453.00 to 1458.00 2 48 3 NAT MN 0.8 to 0.8 2 49 3 NAT MN 9.50 to 9.75 2 50 3 NAT MN 13.00 to 14.00 2 51 3 NAT MN 3.7 to 3.8 2 52 3 NAT MN 165.00 to 170.00 2 53 3 NAT MN 14 to 14 2 54 3 NAT MN 0.70 to 0.75 2 55 3 NAT MN 1.4 to 1.6 2 56 3 NAT MN 125.0 to 125.0 2 57 3 NAT MN 2000.0 to 2400.0 2 58 3 NAT MN 45.00 to 50.00 2 59 3 NAT MN 7.00 to 7.50 2 60 3 NAT MN 103.50 to 103.60 2 61 3 NAT MN 10.00 to 11.00 2 62 3 NAT MN 0.25 to 0.25 2 63 3 NAT MN 320.00 to 330.00 2 64 3 NAT MN 84.80 to 85.50 2 65 3 NAT MN 39 to 39 2
  • 41.
  • 42.
  • 43.
  • 44.
  • 45.
  • 46.
  • 47.
  • 48.
  • 49.
  • 50.
  • 51.
  • 52.
  • 53.
  • 54.
  • 55.
  • 56.
  • 57.
  • 58.
  • 59.
  • 60.
  • 61.
  • 62.
  • 63.
  • 64.
  • 65.
  • 66.
  • 67.
  • 68.
  • 69.
  • 70.
  • 71.
  • 72. GA - General Aptitude Q1 - Q5 carry one mark each. Q.No. 1 (A) (B) (C) (D) Q.No. 2 (A) (B) (C) (D) Q.No. 3 (A) (B) (C) (D) Q.No. 4 (A) (B) (C) (D) Q.No. 5 (A) (B) (C) MN: Mining Engineering GATE 2020 Graduate Aptitude Test in Engineering 2020 IIT Delhi Organising Institute Home Home Information Brochure Information Brochure GATE International GATE International Pre Examination Pre Examination Important Dates Important Dates FAQs FAQs Contact Us Contact Us
  • 73. (D) Q6 - Q10 carry two marks each. Q.No. 6 (A) (B) (C) (D) Q.No. 7 (A) (B) (C) (D) Q.No. 8 (A) (B) (C) (D) Q.No. 9 (A) (B) (C) (D) Q.No. 10
  • 74. (A) (B) (C) (D) MN: Mining Engineering Q1 - Q25 carry one mark each. Q.No. 1 (A) (B) (C) (D) Q.No. 2 (A) (B) (C) (D)
  • 75. Q.No. 3 (A) (B) (C) (D) Q.No. 4 (A) (B) (C) (D) Q.No. 5 (A) (B) (C) (D) Q.No. 6 (A) (B) (C) (D) Q.No. 7
  • 76. (A) (B) (C) (D) Q.No. 8 (A) (B) (C) (D) Q.No. 9 (A) (B) (C) (D) Q.No. 10 (A) (B) (C) (D) Q.No. 11 (A) (B) (C) (D) Q.No. 12
  • 77. (A) (B) (C) (D) Q.No. 13 (A) (B) (C) (D) Q.No. 14 (A) (B) (C) (D) Q.No. 15 (A) (B) (C) (D) Q.No. 16 Q.No. 17 Q.No. 18
  • 78. Q.No. 19 Q.No. 20 Q.No. 21 Q.No. 22 Q.No. 23 Q.No. 24 Q.No. 25 Q26 - Q55 carry two marks each. Q.No. 26 (A) (B) (C) (D) Q.No. 27
  • 79. (A) (B) (C) (D) Q.No. 28 (A) (B) (C) (D) Q.No. 29 (A) (B) (C) (D) Q.No. 30 (A) (B) (C) (D) Q.No. 31 (A) (B) (C) (D) Q.No. 32
  • 81. Q.No. 39 Q.No. 40 Q.No. 41 Q.No. 42 Q.No. 43
  • 82. Q.No. 44 Q.No. 45 Q.No. 46 Q.No. 47 Q.No. 48
  • 83. Q.No. 49 Q.No. 50 Q.No. 51 Q.No. 52
  • 85. Copyright : GATE 2020, IIT Delhi
  • 86. Answer Key - MN: Mining Engineering Q.No. Session Que.Type Sec. Name Key Marks 1 3 MCQ GA C 1 2 3 MCQ GA B 1 3 3 MCQ GA D 1 4 3 MCQ GA D 1 5 3 MCQ GA B 1 6 3 MCQ GA D 2 7 3 MCQ GA C 2 8 3 MCQ GA B 2 9 3 MCQ GA B 2 10 3 MCQ GA B 2 1 3 MCQ MN C 1 2 3 MCQ MN A 1 3 3 MCQ MN D 1 4 3 MCQ MN B 1 5 3 MCQ MN B 1 6 3 MCQ MN C 1 7 3 MCQ MN A 1 8 3 MCQ MN D 1 9 3 MCQ MN A 1 10 3 MCQ MN B 1 11 3 MCQ MN D 1 12 3 MCQ MN C 1 13 3 MCQ MN C 1 14 3 MCQ MN B 1 15 3 MCQ MN D 1 16 3 NAT MN 18.0 to 20.0 1 17 3 NAT MN 2.9 to 3.1 1 18 3 NAT MN 5.4 to 5.6 1 19 3 NAT MN 0.120 to 0.125 1 20 3 NAT MN 2.4 to 2.6 1 21 3 NAT MN 51830.0 to 51850.0 1 22 3 NAT MN 0.560 to 0.580 1 23 3 NAT MN 2 to 2 1 24 3 NAT MN 5990.0 to 6010.0 1 25 3 NAT MN 0.32 to 0.34 1 26 3 MCQ MN B 2 27 3 MCQ MN D 2 28 3 MCQ MN C 2 29 3 MCQ MN A 2 30 3 MCQ MN D 2 31 3 MCQ MN C 2 32 3 MCQ MN D 2 33 3 MCQ MN D 2 GATE 2020 Graduate Aptitude Test in Engineering 2020 IIT Delhi Organising Institute Home Home Information Brochure Information Brochure GATE International GATE International Pre Examination Pre Examination Important Dates Important Dates FAQs FAQs Contact Us Contact Us
  • 87. 34 3 NAT MN 0.530 to 0.550 2 35 3 NAT MN 148.0 to 154.0 2 36 3 NAT MN 385.0 to 410.0 2 37 3 NAT MN -205.0 to -195.0 2 38 3 NAT MN 22.00 to 25.00 2 39 3 NAT MN 7.2 to 7.7 2 40 3 NAT MN -1.255 to -1.165 2 41 3 NAT MN 2380.0 to 2396.0 2 42 3 NAT MN 2.10 to 2.45 2 43 3 NAT MN 3360.0 to 3480.0 2 44 3 NAT MN 71.0 to 72.0 2 45 3 NAT MN 71.0 to 73.0 2 46 3 NAT MN 87.0 to 89.0 2 47 3 NAT MN 0.44 to 0.50 2 48 3 NAT MN 100.4 to 100.6 2 49 3 NAT MN 157.0 to 161.0 2 50 3 NAT MN 248.0 to 252.0 2 51 3 NAT MN 0.020 to 0.030 2 52 3 NAT MN 67.00 to 69.00 2 53 3 NAT MN 18 to 18 2 54 3 NAT MN 7.70 to 7.95 2 55 3 NAT MN 30.8 to 31.6 2 Copyright : GATE 2020, IIT Delhi
  • 88.
  • 89.
  • 90.
  • 91.
  • 92.
  • 93.
  • 94.
  • 95.
  • 96.
  • 97.
  • 98.
  • 99.
  • 100.
  • 101.
  • 102.
  • 103.
  • 104.
  • 105.
  • 106. GATE 2018 General Aptitude (GA) Set-3 GA 1/3 Q. 1 – Q. 5 carry one mark each. Q.1 “From where are they bringing their books? ________ bringing _______ books from _____.” The words that best fill the blanks in the above sentence are (A) Their, they’re, there (B) They’re, their, there (C) There, their, they’re (D) They’re, there, there Q.2 “A _________ investigation can sometimes yield new facts, but typically organized ones are more successful.” The word that best fills the blank in the above sentence is (A) meandering (B) timely (C) consistent (D) systematic Q.3 The area of a square is 𝑑. What is the area of the circle which has the diagonal of the square as its diameter? (A) 𝜋𝑑 (B) 𝜋𝑑2 (C) 1 4 𝜋𝑑2 (D) 1 2 𝜋𝑑 Q.4 What would be the smallest natural number which when divided either by 20 or by 42 or by 76 leaves a remainder of 7 in each case? (A) 3047 (B) 6047 (C) 7987 (D) 63847 Q.5 What is the missing number in the following sequence? 2, 12, 60, 240, 720, 1440, _____, 0 (A) 2880 (B) 1440 (C) 720 (D) 0
  • 107. GATE 2018 General Aptitude (GA) Set-3 GA 2/3 Q. 6 – Q. 10 carry two marks each. Q.6 In appreciation of the social improvements completed in a town, a wealthy philanthropist decided to gift Rs 750 to each male senior citizen in the town and Rs 1000 to each female senior citizen. Altogether, there were 300 senior citizens eligible for this gift. However, only 8/9th of the eligible men and 2/3rd of the eligible women claimed the gift. How much money (in Rupees) did the philanthropist give away in total? (A) 1,50,000 (B) 2,00,000 (C) 1,75,000 (D) 1,51,000 Q.7 If 𝑝𝑞𝑟 ≠ 0 and 𝑝−𝑥 = 1 𝑞 , 𝑞−𝑦 = 1 𝑟 , 𝑟−𝑧 = 1 𝑝 , what is the value of the product 𝑥𝑦𝑧? (A) −1 (B) 1 𝑝𝑞𝑟 (C) 1 (D) 𝑝𝑞𝑟 Q.8 In a party, 60% of the invited guests are male and 40% are female. If 80% of the invited guests attended the party and if all the invited female guests attended, what would be the ratio of males to females among the attendees in the party? (A) 2:3 (B) 1:1 (C) 3:2 (D) 2:1 Q.9 In the figure below, ∠𝐷𝐸𝐶 + ∠𝐵𝐹𝐶 is equal to ____________ . (A) ∠𝐵𝐶𝐷 − ∠𝐵𝐴𝐷 (B) ∠𝐵𝐴𝐷 + ∠𝐵𝐶𝐹 (C) ∠𝐵𝐴𝐷 + ∠𝐵𝐶𝐷 (D) ∠𝐶𝐵𝐴 + ∠𝐴𝐷𝐶 A B D E C F
  • 108. GATE 2018 General Aptitude (GA) Set-3 GA 3/3 Q.10 A six sided unbiased die with four green faces and two red faces is rolled seven times. Which of the following combinations is the most likely outcome of the experiment? (A) Three green faces and four red faces. (B) Four green faces and three red faces. (C) Five green faces and two red faces. (D) Six green faces and one red face. END OF THE QUESTION PAPER
  • 109. GATE 2018 Mining Engineering MN 1/11 Q. 1 – Q. 25 carry one mark each. Q.1 If cos sin sin cos X             , then T XX is (A) 0 1 1 0       (B) 1 0 0 1         (C) 1 0 0 1       (D) 0 1 1 0         Q.2 The values of x satisfying the following condition are 4 3 0 3 6 x x    (A) 6, 4 (B) 4, 9 (C) 5, 6 (D) 3,7 Q.3 An azimuth of 330° corresponds to a quadrant bearing of (A) W60o N (B) N30o W (C) S30o W (D) S30o E Q.4 Tri-cone drill bit is a type of (A) cross bit (B) button bit (C) rotary roller bit (D) button drag bit Q.5 Exposure of weak roof in junctions of a development district in a coal mine can be decreased by (A) increasing dimension of panel barrier (B) stitching side walls (C) increasing support density at junctions (D) staggering the junctions Q.6 The property that CANNOT be determined from uniaxial compressive strength test of a rock sample fitted with strain gauges is (A) cohesion (B) Poisson’s ratio (C) modulus of elasticity (D) dilation
  • 110. GATE 2018 Mining Engineering MN 2/11 Q.7 The radial stress concentration around a long circular tunnel excavated in rock is given by the curve (A) m (B) n (C) o (D) p Q.8 Ward-Leonard system is provided in the winding system in order to restrict (A) over-winding of the cage (B) deceleration of the cage (C) acceleration of the cage (D) over-speeding of the cage Q.9 The equipment NOT related to extraction of coal from longwall face operation is (A) AFC (B) road header (C) powered support (D) DERD shearer Q.10 The correct figure depicting the extraction of contiguous seams in bord and pillar working is indicated by Q.11 The significance of ‘potentially explosive mixture’ in Coward flammability diagram is - (A) leakage of fresh air may lead to explosive condition for the mixture (B) leakage of firedamp may lead to explosive condition for the mixture (C) increase in the ambient temperature may lead to explosive condition for the mixture (D) introduction of a source of ignition may result in explosion of the mixture / r a   m   n   p   o 0 p r - distance from the center Stress concentration 0 p a - in-situ stress 0 p 8m 10m (A) (B) (C) (D) 8m pillar 10m parting parting pillar
  • 111. GATE 2018 Mining Engineering MN 3/11 Q.12 Considering ‘I’ as ‘intake’ and ‘R’ as ‘return’, the ventilation symbol for the shaft-bottom air-lock in a coal mine is Q.13 From a coal seam of a mine 1000 tonnes of coal is produced per day. The seam has inflammable gas emission rate of 14000 m3 per day. Percentage of inflammable gas in general body of air is 0.14. The gassiness of the seam is (A) Degree IV (B) Degree III (C) Degree II (D) Degree I Q.14 The temperature profiles and the plume patterns that are most likely to result are given in the figures. The dotted line represents ‘adiabatic lapse rate’ and the bold line represents ‘environmental lapse rate’. The WRONG combination is I R I R I R I R (A) (C) (B) (D) z (A) (D) z T (C) (B) z T z T T
  • 112. GATE 2018 Mining Engineering MN 4/11 Q.15 The inventory pattern shown does NOT represent the following. (A) Inventory is replenished instantaneously (B) Demand decreases with time (C) Shortage is not permitted (D) Demand is uniform Q.16 The figure depicts a transportation problem along with the solution. The correct statement is (A) unbalanced problem, optimal solution (B) unbalanced problem, sub-optimal solution (C) balanced problem, optimal solution (D) balanced problem, sub-optimal solution Q.17 The degree of the differential equation 2 3 2 2 0 d x x dt   is ______________. Q.18 The slope of the line connecting the points (20, 6) and (40, 8) is ______________. Q.19 Two contours of RL 60 m and 70 m are separated by 34 m measured along dip of the seam. The dip of the seam in degree is ________________. Time Inventory level D2 D1 5 20 15 5 10 5 0 15 15 15 S1 S2 10 20
  • 113. GATE 2018 Mining Engineering MN 5/11 Q.20 The RL of the initial station is 200 m. If 1.54m BS   and 0.45m FS   , then the RL of the last station in m is ________________. Q.21 For the conveyor belt drive shown, the tension on the tight side   1 T is double that of the slack side. The coefficient of friction between belt and drum is 0.21. The minimum angle of lap (in degree) to avoid slippage of the belt is ________________. Q.22 In a bord and pillar development district, 6 headings driven along the strike direction are surrounded by panel barriers on the dip and rise sides. The maximum possible number of faces for the panel is ___________________. Q.23 A roadway in a mine, a single light source as shown, emits light uniformly in all directions. The floor level illumination at station A is 20 lux. The floor level illumination at a point B in lux is ______________. Q.24 A system of two identical components connected in series has reliability of 0.25. The reliability of each component is _______________. Q.25 The operational status of an HEMM in a shift is shown in the diagram. The availability of the machine in % is _____________________. 1 T 2 T 400 A B 0 1 8 3 5 6 7.5 Breakdown state Shiftduration hours
  • 114. GATE 2018 Mining Engineering MN 6/11 Q. 26 – Q. 55 carry two marks each. Q.26 If c is a constant, the solution of the differential equation 4 9 0 dy y x dx   is (A) 2 2 81 16 x y c   (B) 2 2 16 81 x y c   (C) 2 2 9 4 x y c   (D) 2 2 4 9 x y c   Q.27 Match the following blasting elements with the corresponding initiators. Blasting Elements Initiators P Electric detonator 1 Match stick Q Safety fuse 2 Booster R Detonating fuse 3 Exploder S Non cap-sensitive explosive 4 Ordinary detonator (A) P-2, Q-3, R-4, S-1 (B) P-3, Q-1, R-4, S-2 (C) P-3, Q-1, R-2, S-4 (D) P-1, Q-4, R-2, S-3 Q.28 The following plot is developed for a rock type after a series of triaxial tests. The uniaxial compressive strength and tensile strength of the rock type, respectively, in MPa, are (A) 30, 5 (B) 30, 6 (C) 24, 6 (D) 24, 5 Q.29 Match the following in the context of environmental management. Technique Purpose P Mulching 1 Dust control Q Aeration 2 Noise control R Wet-scrubbing 3 Soil conservation S Silencer 4 Waste water treatment (A) P-1, Q-4, R-3, S-2 (B) P-3, Q-1, R-4, S-2 (C) P-4, Q-3, R-1, S-2 (D) P-3, Q-4, R-1, S-2 1 3 6 30     3 (MPa)  1  3  1 (MPa) 
  • 115. GATE 2018 Mining Engineering MN 7/11 Q.30 A panel in coal mine produces 400 tonne per day. The number of persons employed in each of the three shifts is 110, 130 and 120. As per CMR, the minimum quantity of air that needs to be circulated at the last ventilation connection of the panel in m3 /min is (A) 780 (B) 900 (C) 1000 (D) 2400 Q.31 A right conical iron ore stack on level ground of height 10 m has 60% Fe. The height of the conical stack is extended up to 20 m using iron ore of 50% Fe. The angle of repose of iron ore is 38o . The mean grade of the final stack in % Fe is ___________________. Q.32 The feasible region of a linear programming problem is shown in the figure. The maximum value of the objective function 4 3 Z X Y   is ____________________. Q.33 Cash flow of a project of duration 4 years is shown. The uniform income ‘R’, in Rs. crores, at the end of 2nd , 3rd and 4th years for an internal rate of return of 10% is ______________. 0 2 3 4 1 R R R Rs. 10 crores year
  • 116. GATE 2018 Mining Engineering MN 8/11 Q.34 The grade values of alumina at three sample locations (A, B and C) in a bauxite deposit are as shown. Using the ‘inverse distance’ method, the computed grade in % alumina at location ‘Z’ is ___________________. Q.35 Economic feasibility of two methods is examined for meeting a targeted mine production. On a yearly basis, the following cost parameters (Rs in crores) are applicable for these methods. Method 1 Method 2 Fixed cost 20 5 Variable cost 2X X2 The annual rate of production, ‘X’ in million tonnes for which both the methods will yield the same operating cost is ___________________. Q.36 A person standing 50 m away from an HEMM experiences 90 dB sound pressure level. If the person moves to a new location that is 70 m away from the HEMM, the sound pressure level experienced by the person, in dB, becomes _________________. Q.37 A portion of a ventilation system has two splits as shown. Split ‘A’ has a resistance of 0.2 Ns2 /m8 and a regulator of size 2.0 m2 . The resistance of split ‘B’ in Ns2 /m8 is ________ Q.38 Air enters a bord and pillar panel at 10 ppm CO and 20.78% O2. The air at the panel return has 80 ppm CO and 20.52% O2. The ‘Graham’s ratio’ for the status of fire in the panel in % is __________________. A (30%) C (42%) B (36%) Z 90 m 60 m 60 m R A 20 m3/s B 30 m3/s
  • 117. GATE 2018 Mining Engineering MN 9/11 Q.39 In a sublevel stope, 20 m high excavation is made with ring drilling in a 6 m wide orebody. In an effective shift time of 6 hrs, three rings are blasted resulting in 9 m extraction length. LHDs of 5.0 m3 bucket capacity and cycle time of 5 minutes are used for ore transportation. The number of LHDs required for this operation is _________________. Q.40 A coal seam lying at a depth of 200 m is developed by bord and pillar method. Pillars are 30 m centre to centre with a gallery width of 4 m. Unit weight of the overlying strata is 28 kN/m3 . If the pillar strength is 9.32 MPa, the factor of safety of pillar is ____________. Q.41 Following information about a longwall retreating panel is given. Panel length = 1800 m Face length = 150 m Depth of web = 0.6 m Shearer travels at a speed of 1.5 m/min along the face. Each cutting cycle requires a non-operational time of 2 h 20 min. The panel is fully extractable. The number of working days required to extract the panel is ______________. Q.42 In a direct rope haulage operating along a 1500 m long incline, 180 tonne of coal is hauled in 7 hours. The average rope speed is 7.5 km/h. The set changing time for the tubs is 2 minutes each at the top and bottom of the incline. If the tub capacity is 1.0 tonne, the number of tubs in the set is __________________. Q.43 An SDL of 1.0 tonne capacity operates with a cycle time of 6 minutes. The dimension of the face is 4 m x 3 m. Five blasts are conducted per shift with an average pull of 1.2 m. If the density of blasted coal is 0.8 tonne/m3 , the time required by the SDL in the shift to lift all the prepared coal in hours is _________________. Q.44 A force diagram is shown below. Considering clockwise moments to be positive, the resultant moment about A in Nm is _________________. 3 N 7 N 9 N A 120 m 50 m 70 m
  • 118. GATE 2018 Mining Engineering MN 10/11 Q.45 The experiment to determine permeability of a soil sample is illustrated below. The cross- sectional area of the sample is 20.0 cm2 . The permeability of the soil sample in cm/s is ______________. Q.46 In underground face blasting, the pull is found to be 10% less than the drillhole length. The headings of 3.6 m width are supported by prop density of 1.44/m2 . The hole length is 1.5 m, and 6 rounds of blasting are done in the panel per shift. The number of props to be erected in a shift is ____________________. Q.47 The immediate roof of a mine is supported by bolts of length 1.5 m, arranged in 1.2 m x 1.2 m grid pattern. If the unit weight of roof rock is 2.25 tonne/m3 and load carrying capacity of each bolt is 7.5 tonne, the factor of safety of the support system is _______________. Q.48 In a level terrain, a vertical orebody of 20 m uniform width is worked by surface mining method. The density of the ore is 2.5 tonne/m3 . The ultimate pit has a depth of 60 m, width of 20 m at pit bottom, and a pit slope of 45o . The overall stripping ratio for this condition in m3 /tonne is ____________________. Q.49 A 20 m long steel tape used for survey is found to be short by 10 cm. If the area measured with the steel tape is 5000 m2 , the actual area in m2 is __________________. 5 cm Water 360 ml/h Soil sample 20 cm
  • 119. GATE 2018 Mining Engineering MN 11/11 Q.50 The following figure shows the designed blast pattern of a bench. The explosive column is charged at 18 kg/m. If the unit weight of the blasted material is 2.5 tonne/m3 , the powder factor for the blast in tonne/kg is _______________. Q.51 Two points on the equator have longitudes 55o E and 25o W. Considering radius of earth as 6400 km, the distance between the two points in km is ________________. Q.52 Sum of the series 5, 10, 15, ……….., 500 is __________________. Q.53 The sample standard deviation for the following set of observations is _______________. 40, 45, 50 and 55 Q.54 For the given function      , 3 4 f x y x y    , the value of f f x y      at 2 and 1 x y   is _____________________. Q.55 Given 2 6 y x x    , the value of   ln d y dx at 2 x  is ________________. END OF THE QUESTION PAPER 4 m 3 m 10 m 3 m 8 m stemming explosive
  • 120. Q.No. Type Section Key/Range Marks 1 MCQ GA B 1 2 MCQ GA A 1 3 MCQ GA D 1 4 MCQ GA C 1 5 MCQ GA B 1 6 MCQ GA B 2 7 MCQ GA C 2 8 MCQ GA B 2 9 MCQ GA A 2 10 MCQ GA C 2 1 MCQ MN C 1 2 MCQ MN D 1 3 MCQ MN B 1 4 MCQ MN C 1 5 MCQ MN D 1 6 MCQ MN A 1 7 MCQ MN D 1 8 MCQ MN D 1 9 MCQ MN B 1 10 MCQ MN C 1 11 MCQ MN A 1 12 MCQ MN A 1 13 MCQ MN B 1
  • 121. Q.No. Type Section Key/Range Marks 14 MCQ MN C 1 15 MCQ MN B 1 16 MCQ MN C 1 17 NAT MN 1.0 to 1.0 1 18 NAT MN 0.1 to 0.1 1 19 NAT MN 16.5 to 17.5 1 20 NAT MN 201.09 to 201.09 1 21 NAT MN 185.0 to 195.0 1 22 NAT MN 16.0 to 16.0 1 23 NAT MN 8.5 to 9.5 1 24 NAT MN 0.5 to 0.5 1 25 NAT MN 56.0 to 56.5 1 26 MCQ MN D 2 27 MCQ MN B 2 28 MCQ MN A 2 29 MCQ MN D 2 30 MCQ MN C 2 31 NAT MN 51.0 to 52.0 2 32 NAT MN 74.0 to 74.0 2 33 NAT MN 4.4 to 4.5 2 34 NAT MN 36.0 to 37.5 2 35 NAT MN 5.0 to 5.0 2 36 NAT MN 86.75 to 87.25 2
  • 122. Q.No. Type Section Key/Range Marks 37 NAT MN 0.24 to 0.28 2 38 NAT MN 2.65 to 2.75 2 39 NAT MN 3.0 to 3.0 2 40 NAT MN 1.2 to 1.3 2 41 NAT MN 500.0 to 500.0 2 42 NAT MN 12.0 to 12.0 2 43 NAT MN 5.6 to 5.8 2 44 NAT MN ‐400.0 to ‐400.0 2 45 NAT MN 0.02 to 0.02 2 46 NAT MN 41.0 to 43.0 2 47 NAT MN 1.4 to 1.6 2 48 NAT MN 1.2 to 1.2 2 49 NAT MN 4945.0 to 4955.0 2 50 NAT MN 2.00 to 2.15 2 51 NAT MN 8930.0 to 8940.0 2 52 NAT MN 25250.0 to 25250.0 2 53 NAT MN 6.4 to 6.5 2 54 NAT MN 10.0 to 10.0 2 55 NAT MN 0.40 to 0.45 2
  • 123. Graduate Aptitude Test in Engineering 2017 Question Paper Name: Mining Engineering 12th Feb 2017 Subject Name: Duration: 180 Total Marks: 100
  • 124. Question Number : 2 Correct : 1 Wrong : -0.33 Question Number : 3 Correct : 1 Wrong : -0.33 Question Number :1 Correct : 1 Wrong : -0.33
  • 125. Question Number : 6 Correct : 1 Wrong : -0.33 Question Number : 7 Correct : 1 Wrong : -0.33 Question Number : 8 Correct : 1 Wrong : 0 Question Number : 5 Correct : 1 Wrong : -0.33 Question Number : 4 Correct : 1 Wrong : -0.33
  • 126. Question Number : 10 Correct : 1 Wrong : -0.33 Question Number : 11 Correct : 1 Wrong : -0.33 Question Number : 9 Correct : 1 Wrong : -0.33
  • 127. Question Number : 13 Correct : 1 Wrong : -0.33 Question Number : 14 Correct : 1 Wrong : -0.33 Question Number : 15 Correct : 1 Wrong : -0.33 Question Number : 16 Correct : 1 Wrong : 0 Question Number : 12 Correct : 1 Wrong : -0.33
  • 128. Question Number : 18 Correct : 1 Wrong : 0 Question Number : 19 Correct : 1 Wrong : 0 Question Number : 20 Correct : 1 Wrong :- 0.33 Question Number : 17 Correct : 1 Wrong : -0.33
  • 129. Question Number : 22 Correct : 1 Wrong : -0.33 Question Number : 23 Correct : 1 Wrong : 0 Question Number : 21 Correct : 1 Wrong : -0.33
  • 130. Question Number : 25 Correct : 1 Wrong : -0.33 Question Number : 26 Correct : 2 Wrong : 0 Question Number : 27 Correct : 2 Wrong : 0 Question Number : 24 Correct : 1 Wrong : -0.33
  • 131. Question Number : 29 Correct : 2 Wrong : 0 Question Number : 30 Correct : 2 Wrong : 0 Question Number : 28 Correct : 2 Wrong : 0
  • 132. Question Number : 32 Correct : 2 Wrong : 0 Question Number : 33 Correct : 2 Wrong : 0 Question Number : 31 Correct : 2 Wrong : 0
  • 133. Question Number : 35 Correct : 2 Wrong : 0 Question Number : 34 Correct : 2 Wrong : 0
  • 134. Question Number : 37 Correct : 2 Wrong : 0 Question Number : 38 Correct : 2 Wrong : 0 Question Number : 36 Correct : 2 Wrong : 0
  • 135. Question Number : 39 Correct : 2 Wrong : 0 Question Number : 40 Correct : 2 Wrong : -0.66
  • 136. Question Number : 42 Correct : 2 Wrong : -0.66 Question Number : 41 Correct : 2 Wrong : -0.66
  • 137. Question Number : 44 Correct : 2 Wrong : -0.66 Question Number : 45 Correct : 2 Wrong : 0 Question Number : 43 Correct : 2 Wrong : 0
  • 138. Question Number : 46 Correct : 2 Wrong : -0.66 Question Number : 47 Correct : 2 Wrong : 0
  • 139. Question Number : 49 Correct : 2 Wrong : 0 Question Number : 50 Correct : 2 Wrong : 0 Question Number : 48 Correct : 2 Wrong : 0
  • 140. Question Number : 52 Correct : 2 Wrong : 0 Question Number : 53 Correct : 2 Wrong : 0 Question Number : 54 Correct : 2 Wrong : -0.66 Question Number : 51 Correct : 2 Wrong : 0
  • 141. Question Number : 56 Correct : 1 Wrong : -0.33 Question Number : 55 Correct : 2 Wrong : 0
  • 142. Question Number : 58 Correct : 1 Wrong : -0.33 Question Number : 59 Correct : 1 Wrong : -0.33 Question Number : 60 Correct : 1 Wrong : -0.33 Question Number : 57 Correct : 1 Wrong : -0.33
  • 143. Question Number : 62 Correct : 2 Wrong : -0.66 Question Number : 63 Correct : 2 Wrong : -0.66 Question Number : 61 Correct : 2 Wrong : -0.66
  • 144. Question Number : 65 Correct : 2 Wrong : -0.66 Question Number : 64 Correct : 2 Wrong : -0.66
  • 145. Q. No. Type Section Key Marks 1 MCQ MN C 1 2 MCQ MN B 1 3 MCQ MN D 1 4 MCQ MN A 1 5 MCQ MN B 1 6 MCQ MN C 1 7 MCQ MN D 1 8 NAT MN 2.9 to 3.0 1 9 MCQ MN C 1 10 MCQ MN B 1 11 MCQ MN C 1 12 MCQ MN A 1 13 MCQ MN A 1 14 MCQ MN D 1 15 MCQ MN B 1 16 NAT MN 60 to 62 1 17 MCQ MN A 1 18 NAT MN 5.15 to 6.0 1 19 NAT MN 113 to 116 or 1.13 to 1.16 1 20 MCQ MN D 1 21 MCQ MN B 1 22 MCQ MN C 1 23 NAT MN 835 to 840 1 24 MCQ MN B 1 25 MCQ MN D 1 26 NAT MN -2.0 to -2.0 2 27 NAT MN 32 to 32 2 28 NAT MN 0.96 to 0.98 2 29 NAT MN 123.0 to 124.5 2 30 NAT MN 174765 to 174765 2 31 NAT MN 20 to 23 2 32 NAT MN 25 to 27 2 33 NAT MN 46 to 47 2 34 NAT MN 71 to 73 2 35 NAT MN 65000 to 67000 2 36 NAT MN 0.96 to 0.97 2
  • 146. 37 NAT MN 1.6 to 1.7 2 38 NAT MN 5.5 to 6.0 2 39 NAT MN 19.5 to 20 2 40 MCQ MN B 2 41 MCQ MN A 2 42 MCQ MN C 2 43 NAT MN 1 to 3 2 44 MCQ MN D 2 45 NAT MN 5.5 to 6.0 2 46 MCQ MN A 2 47 NAT MN 13.5 to 14.0 2 48 NAT MN 1185 to 1189 2 49 NAT MN 2.5 to 2.8 2 50 NAT MN 36 to 37 2 51 NAT MN 290 to 292 2 52 NAT MN 935 to 936 2 53 NAT MN 0.66 to 0.68 2 54 MCQ MN D 2 55 NAT MN 5975 to 5990 2 56 MCQ GA C 1 57 MCQ GA C 1 58 MCQ GA D 1 59 MCQ GA C 1 60 MCQ GA D 1 61 MCQ GA B 2 62 MCQ GA B 2 63 MCQ GA B 2 64 MCQ GA A 2 65 MCQ GA C 2
  • 147. GATE 2016 General Aptitude - GA Set-2 1/3 Q. 1 – Q. 5 carry one mark each. Q.1 The volume of a sphere of diameter 1 unit is ________ than the volume of a cube of side 1 unit. (A) least (B) less (C) lesser (D) low Q.2 The unruly crowd demanded that the accused be _____________ without trial. (A) hanged (B) hanging (C) hankering (D) hung Q.3 Choose the statement(s) where the underlined word is used correctly: (i) A prone is a dried plum. (ii) He was lying prone on the floor. (iii) People who eat a lot of fat are prone to heart disease. (A) (i) and (iii) only (B) (iii) only (C) (i) and (ii) only (D) (ii) and (iii) only Q.4 Fact: If it rains, then the field is wet. Read the following statements: (i) It rains (ii) The field is not wet (iii) The field is wet (iv) It did not rain Which one of the options given below is NOT logically possible, based on the given fact? (A) If (iii), then (iv). (B) If (i), then (iii). (C) If (i), then (ii). (D) If (ii), then (iv). Q.5 A window is made up of a square portion and an equilateral triangle portion above it. The base of the triangular portion coincides with the upper side of the square. If the perimeter of the window is 6 m, the area of the window in m2 is ___________. (A) 1.43 (B) 2.06 (C) 2.68 (D) 2.88
  • 148. GATE 2016 General Aptitude - GA Set-2 2/3 Q. 6 – Q. 10 carry two marks each. Q.6 Students taking an exam are divided into two groups, P and Q such that each group has the same number of students. The performance of each of the students in a test was evaluated out of 200 marks. It was observed that the mean of group P was 105, while that of group Q was 85. The standard deviation of group P was 25, while that of group Q was 5. Assuming that the marks were distributed on a normal distribution, which of the following statements will have the highest probability of being TRUE? (A) No student in group Q scored less marks than any student in group P. (B) No student in group P scored less marks than any student in group Q. (C) Most students of group Q scored marks in a narrower range than students in group P. (D) The median of the marks of group P is 100. Q.7 A smart city integrates all modes of transport, uses clean energy and promotes sustainable use of resources. It also uses technology to ensure safety and security of the city, something which critics argue, will lead to a surveillance state. Which of the following can be logically inferred from the above paragraph? (i) All smart cities encourage the formation of surveillance states. (ii) Surveillance is an integral part of a smart city. (iii) Sustainability and surveillance go hand in hand in a smart city. (iv) There is a perception that smart cities promote surveillance. (A) (i) and (iv) only (B) (ii) and (iii) only (C) (iv) only (D) (i) only Q.8 Find the missing sequence in the letter series. B, FH, LNP, _ _ _ _. (A) SUWY (B) TUVW (C) TVXZ (D) TWXZ Q.9 The binary operation □ is defined as a □ b = ab+(a+b), where a and b are any two real numbers. The value of the identity element of this operation, defined as the number x such that a □ x = a, for any a, is . (A) 0 (B) 1 (C) 2 (D) 10
  • 149. GATE 2016 General Aptitude - GA Set-2 3/3 Q.10 Which of the following curves represents the function = ln(| | (| |)| |) for | | < 2 ? Here, represents the abscissa and represents the ordinate. (A) (B) (C) (D) END OF THE QUESTION PAPER
  • 150. GATE 2016 Mining Engineering (MN) MN 1/14 Q. 1 – Q. 25 carry one mark each. Q.1 The differential of the equation, 2 2 1 x y   , with respect to x is (A) x y  (B) x y (C) y x  (D) y x Q.2 If [ ][ ] [ ] A B I  then (A) [ ] [ ]T B A  (B) [ ] [ ]T A B  (C) 1 [ ] [ ] B A   (D) [ ] [ ] B A  Q.3 4 X C  is the general integral of (A) 3 3 x dx  (B) 3 1 4 x dx  (C) 3 x dx  (D) 3 4 x dx  Q.4 Sinh (x) is (A) 4 x x e e  (B) 2 x x e e  (C) 2 x x e e  (D) 4 x x e e  Q.5 Identify the correct statement. NONEL is used for surface connection of the blast holes in order to (A) achieve better water resistance over detonating fuse (B) have a precise delay timing (C) provide noiseless shock front movement (D) avoid deflagration Q.6 Identify the pattern of surface blasting given in the figure. The values of delay time, in ms, are given against each blasthole. (A) V- cut (B) extended V- cut (C) row to row (D) en echelon 100 75 50 25 25 50 75 100 50 25 0 0 0 0 25 50
  • 151. GATE 2016 Mining Engineering (MN) MN 2/14 Q.7 Identify the initiation sequence which is NOT possible for surface blasting. (A) Detonating fuse NonelElectronic detonator (B) Electric detonatorNonelDetonating fuse (C) Electric detonator Detonating fuse  Nonel (D) Electronic detonator Detonating fuse  Nonel Q.8 Parallel holes at right angles to the face with some holes uncharged are associated with the following shot hole pattern (A) drag cut (B) wedge cut (C) pyramid cut (D) burn cut Q.9 Bieniawski’s Rock Mass Rating considers the parameters: RQD, spacing of joints, condition of joints, ground water condition, and (A) tensile strength (B) uniaxial compressive strength (C) shear strength (D) buckling strength Q.10 A rockmass is subjected to hydrostatic pressure of 6 MPa. If each of the measured strains xx yy zz      , is 2.0 mm/m, then the bulk modulus, in GPa, is _______ Q.11 Identify the uniaxial compressive loading condition from the following four Mohr circles. (A) (1) (B) (2) (C) (3) (D) (4)
  • 152. GATE 2016 Mining Engineering (MN) MN 3/14 Q.12 Out of the given stress-strain curves, identify the rock type that is most prone to rock burst. (A) (1) (B) (2) (C) (3) (D) (4) Q.13 A longwall panel of width 120 m is extracted at a depth of 200 m. Critical subsidence is reached when the panel length becomes 150 m. If the seam were to be worked at a depth of 300 m, critical subsidence would be observed at a panel length, in m, of _______. Q.14 The support system followed along the goaf edge in a depillaring panel is (A) rope stitching (B) cable bolting (C) wooden/steel chock (D) hydraulic prop Q.15 Which one of the following ropes CANNOT be an effective cable bolt? (A) locked coil wire rope (B) Langs lay wire rope (C) ordinary lay wire rope (D) bird-caged wire rope Q.16 In metalliferous mines, the sublevel interval does NOT depend on (A) capacity of drilling equipment (B) capacity of loading equipment (C) strength of rib pillar (D) strength of wall rock (4) (3) (1) σ ε (2) 0,0
  • 153. GATE 2016 Mining Engineering (MN) MN 4/14 Q.17 Jack hammer does NOT contain (A) pawl and ratchet (B) gear box (C) rifle bar (D) piston Q.18 At the inlet of a mine roadway, the dry and wet bulb temperatures of air are 0 38 Cand 0 29 C , respectively. At the outlet, the corresponding temperatures are 0 32 C and 0 29 C , respectively. The process of heat transfer in the airway is described as (A) evaporative cooling (B) sensible cooling (C) sensible heating (D) dehumidification Q.19 Underground coal mines are in principle ventilated by exhausting system, so that (A) spontaneous heating risk is reduced (B) fumes can be quickly removed in case of an underground fire (C) build-up of methane concentration is decreased (D) cool and fresh intake air can enter underground Q.20 Identify the WRONG statement. Pit bottom air lock (A) prevents the short circuiting of air when the flow is reversed in coal mines (B) has at least three doors (C) has at least one door that has provision for latching (D) all doors are in principle designed to open towards high pressure side of the air Q.21 Identify the WRONG statement. The ‘temperature inversion’ of the atmosphere in surface mines aggravates the problem of (A) airborne dust (B) noise (C) ground vibrations (D) visibility Q.22 In a CO self rescuer, the purpose of the calcium bromide and lithium chloride mixture is to (A) dry the incoming air (B) convert the CO catalytically to 2 CO (C) absorb and thereby neutralise CO (D) cool the inhaled air from excess exothermic heat due to chemical reaction
  • 154. GATE 2016 Mining Engineering (MN) MN 5/14 Q.23 IRR of a project is the discount rate at which (A) profit after tax is zero (B) written down value of the project is zero (C) revenue from the project is zero (D) NPV is zero Q.24 For the critical path network shown, the slack for the activity ‘b’, in months, is (A) 4 (B) 6 (C) 9 (D) 13 Q.25 The three axes comprising the numerical codification of resources, as per the UNFC, are (A) Economic Viability, Geological Assessment, Geotechnical Assessment (B) Geological Assessment, Environmental Assessment, Feasibility Assessment (C) Feasibility Assessment, Geological Assessment, Mining Assessment (D) Economic Viability, Geological Assessment, Feasibility Assessment Q. 26 – Q. 55 carry two marks each. Q.26 Equations of two planes are 4 z  and 4 3 z x   . The included angle between the two planes in degrees, is ____________ Q.27 A force ˆ ˆ ˆ 2 5 6 P i j k     acts on a particle. The particle is moved from point A to point B, where the position vectors of A  and B  are ˆ ˆ ˆ 6 3 i j k   and ˆ ˆ ˆ 4 3 2 i j k   respectively. The work done is ____________ Q.28 The value of x in the simultaneous equations is ______________ 3 2 3 2 3 3 2 4 x y z x y z x y z          
  • 155. GATE 2016 Mining Engineering (MN) MN 6/14 Q.29 Two persons P and Q toss an unbiased coin alternately on an understanding that whoever gets the head first wins. If P starts the game, then the probability of P winning the game is ____________ Q.30 Data pertaining to a surface bench blast is given below: Burden = 3.0 m Sub-grade drilling = 1.0 m Spacing = 4.0 m Collar stemming = 4.0 m Bench height = 10.0 m Air decking length = 1.0 m Density of rock = 2000 3 kg/m Linear charge concentration = 10 kg/m The powder factor of the blast, in kg/tonne, is _______ Q.31 Match the following for a typical slurry explosive. Chemical Purpose P. Calcium nitrate 1. Cross linking agent Q. Potassium dichromate 2. Gelling agent R. TNT 3. Oxidiser S. Starch 4. Fuel (A) P-1, Q-2, R-3, S-4 (B) P-2, Q-4, R-3, S-1 (C) P-3, Q-1, R-4, S-2 (D) P-4, Q-3, R-2, S-1 Q.32 A 10 m thick coal block is excavated by a contractor at a cost of Rs. 40 per 3 m . The excavated area, measured in the mine plan, is found to be 2 50 cm . If the mine plan has been drawn to a scale of 1:1000, the payment to be made to the contractor, in lakhs of Rs., is _______ Q.33 Two vertical shafts of a mine have the following parameters: The gradient of the drift connecting the shaft bottoms, in degrees, is_____ Shaft Shaft‐A Shaft‐B Collar RL (m) 0.0 0.0 Depth(m) 250 200 Northing(m) 200 100 Easting(m) 100 ‐100
  • 156. GATE 2016 Mining Engineering (MN) MN 7/14 Q.34 For a station ‘A’ on the Earth’s surface, as shown in the figure, match the following Arc Description Q. MA 1. Longitude R. LM 2. Co-latitude S. PA 3. Latitude (A) Q-2, R-3, S-1 (B) Q-3, R-1, S-2 (C) Q-2, R-1, S-3 (D) Q-3, R-2, S-1 Q.35 Match the following for the prismatic compass shown below Component Name P. X1 1. Agate bearing Q. X2 2. Object vane R. X3 3. Magnetic needle S. X4 4. Prism (A) P-1, Q-2, R-3, S-4 (B) P-1, Q-3, R-2, S-4 (C) P-2, Q-1, R-4, S-3 (D) P-3, Q-1, R-4, S-2 Q.36 A ladder placed against a frictionless wall at an inclination of 0 60 with horizontal, is in a state of limiting equilibrium. The ladder has a length of 13 m and a uniform mass of 4 kg/m. The coefficient of friction between the ladder and the floor is ______ A P M L O P’ Reference meridian Equator
  • 157. GATE 2016 Mining Engineering (MN) MN 8/14 Q.37 A cubical rock sample is enclosed between two fixed hard steel plates as shown in the figure below. The modulus of elasticity and Poisson’s ratio of the rock are 2 GPa and 0.25, respectively. If the rock is subjected to the stresses as shown in the figure, the strain in x-direction, in mm/m, is _______. Q.38 In a hydrostatic stress field, point A is in the middle of two circular openings as shown in the figure. The radial stress, in MPa, at point A is _________. Steel plate Rock block 5 Mpa 8 MPa x y z 10 MPa 10 m 2.5 m 2.5 m A 5 MPa 5 MPa 5 MPa 5 MPa
  • 158. GATE 2016 Mining Engineering (MN) MN 9/14 Q.39 Curves (a) and (b) represent the stress distributions along the length of a ‘full column grouted bolt’ shown in the figure. Curves (a) and (b) are (A) Tensile stress, Compressive stress (B) Axial stress, Shear stress (C) Compressive stress, Tensile stress (D) Shear stress, Axial stress Q.40 Match the following mechanical properties with the formulae Mechanical property Formula P. Modulus of elasticity 1. n c tan    Q. Compressive strength 2. lateral longitudinal /   R. Shear Strength 3. /   S. Poisson’s ratio 4. 2 n F / r  (A) P-1, Q-2, R-3, S-4 (B) P-1, Q-4, R-3, S-2 (C) P-3, Q-4, R-1, S-2 (D) P-3, Q-2, R-1, S-4 Q.41 A skip of 10 tonne capacity hoists ore through a 1000 m deep shaft at a speed of 20 m/s. The skip accelerates and decelerates at 2.0 2 m/s . The loading and unloading times for the skip are 2.5 min and 1.5 min, respectively. The maximum hourly capacity of the hoisting system, in tonnes, is _____________ Q.42 Match the following: Haulage unit Safety device P. Friction winder Q. Drum winder R. Direct rope haulage S. Endless rope haulage 1. Run-away switch 2. Lilly controller 3. Regenerative braking 4. Monkey/back catch (A) P-1, Q-2, R-3, S-4 (B) P-3, Q-2, R-1, S-4 (C) P-1, Q-3, R-4, S-2 (D) P-2, Q-3, R-1, S-4 Pickup length Anchor length (b) (a) Bolt Grouted material Rock surface Stress Length 0,0
  • 159. GATE 2016 Mining Engineering (MN) MN 10/14 Q.43 In the gear assembly shown, the rpm of Gear 1 is 600. The number of teeth in Gear 1, Gear 2, Gear 3, Gear 4, Gear 5 and Gear 6 is 30, 45, 15, 20, 10 and 30, respectively. The rpm of Gear 6 is ______ Q.44 An operating surface mine is proposed to be deepened by 30 m as shown in the figure. If the density of the ore is 2.4 3 tonne m , the incremental stripping ratio for the deepening, in 3 m tonne, is ______. Q.45 From an openpit sump, mine water is lifted using a 250 m long straight pipeline laid along a gradient of 0 34 . The pumping rate is 500 gpm (1 gallon = 3.8 litres). Additional head loss due to pipe friction can be considered to be 10% of head lifted. At an overall efficiency of 70%, the electric power consumed by the pump, in kW, is __________. Overburden 300 300 150 m 100 m 30 m 100 m Proposed line of deepening 300 30 m Ore
  • 160. GATE 2016 Mining Engineering (MN) MN 11/14 Q.46 With reference to Coward diagram, match the following in the context of explosibility of a mixture of ‘normal air’ and ‘methane’. (O2 %, CH4 %) Mixture status P. 20.5, 2.4 1. Impossible mixture Q. 19.0, 9.5 2. Non-explosive R. 17.0, 19.0 3. Potentially explosive S. 20.0, 19.5 4. Explosive (A) P-2, Q-4, R-3, S-1 (B) P-2, Q-3, R-1, S-4 (C) P-2, Q-4, R-1, S-3 (D) P-3, Q-2, R-1, S-4 Q.47 A U-tube manometer is subjected to differential pressure as shown. If specific gravity of kerosene is 0.8, the value of   1 2 P P  , in Pa, is_________. Q.48 An air stream having an enthalpy of 100 kJ/kgda, is flowing at 20 kgda/s. It is cooled by water at temperature 0 10 C circulating in a cooling coil at a flow rate of 10.0 l/s. If the return temperature of water is 0 20 C , the enthalpy of the cooled air, in kJ/kgda, is ___________. (Specific heat of water: 4.18 0 kJ kg C ; kgda: kg of dry air).
  • 161. GATE 2016 Mining Engineering (MN) MN 12/14 Q.49 The static pressure characteristic of a mine fan is as shown. If the mine resistance is 0.3 2 8 Ns m , the quantity generated by the fan, in 3 m s , is _______. Q.50 In the context of ventilation plan symbols, match the following: Symbol Description P. 1. Temporary stopping Q. 2. Regulator R. 3. Air-crossing S. 4. Ventilation stopping (A) P-3, Q-4, R-2, S-1 (B) P-2, Q-3, R-1, S-4 (C) P-1, Q-3, R-4, S-2 (D) P-3, Q-2, R-1, S-4 Q.51 A mill concentrate, having 25% copper, is proposed to be sold at Rs. 1,25,000 per tonne. The grade of the deposit is 0.8% Cu and the overall cost of mining and milling is Rs. 2,520 per tonne of ore. At a recovery of 75%, the estimated profit, in Rs./tonne of concentrate, is ________. 1000 0.0 100 P (Pa) Q (m3 /s) R
  • 162. GATE 2016 Mining Engineering (MN) MN 13/14 Q.52 Copper grade distribution in an ore body has the probability density function,   f x , as shown in the figure. The average grade of the deposit, in % Cu, is ______. Q.53 The semivariogram shown belongs to a bauxite deposit. The expected difference in the Al2O3 (%) values between two boreholes separated by a distance of 200 m is _______. Q.54 A surface mine has 15 identical dumpers and two shovels. For shovel 1, the dumper cycle time is 30 min and the shovel loading time is 5 min. For shovel 2, the dumper cycle time is 32 min and the shovel loading time is 4.0 min. Based on match factor optimisation (equitable match factor), the ideal allocation of dumpers to shovel 1 and shovel 2, respectively is (A) 6, 9 (B) 7, 8 (C) 9, 6 (D) 8, 7 300 m 0.0 γ (h) Lag distance 100 (%)2 300 (%)2
  • 163. GATE 2016 Mining Engineering (MN) MN 14/14 Q.55 The composited grade value, in %, between the RLs 10 m to 20 m for the following borehole configuration is __________. END OF THE QUESTION PAPER 14 m 18 m RL = 0 m 25% 35%
  • 164. Q. No Type Section Key Marks 1 MCQ GA B 1 2 MCQ GA A 1 3 MCQ GA D 1 4 MCQ GA C 1 5 MCQ GA B 1 6 MCQ GA C 2 7 MCQ GA C 2 8 MCQ GA C 2 9 MCQ GA A 2 10 MCQ GA C 2 1 MCQ MN A 1 2 MCQ MN C 1 3 MCQ MN D 1 4 MCQ MN B 1 5 MCQ MN C 1 6 MCQ MN B 1 7 MCQ MN A 1 8 MCQ MN D 1 9 MCQ MN B 1 10 NAT MN 1.00 : 1.00 1 11 MCQ MN A 1 12 MCQ MN D 1 13 NAT MN 225.00 : 225.00 1 14 MCQ MN C 1 15 MCQ MN A 1 16 MCQ MN B 1 17 MCQ MN B 1 18 MCQ MN A 1 19 MCQ MN C 1 20 MCQ MN C 1 21 MCQ MN C 1 22 MCQ MN A 1 23 MCQ MN D 1 24 MCQ MN B 1 25 MCQ MN D 1 26 NAT MN 100.00 : 110.00 2 27 NAT MN 22.00 : 22.00 2 28 NAT MN 1.00 : 1.00 2 29 NAT MN 0.66 : 0.67 2 30 NAT MN 0.25 : 0.25 2 31 MCQ MN C 2 32 NAT MN 20.00 : 20.00 2 33 NAT MN 12.00 : 13.00 2 34 MCQ MN B 2 35 MCQ MN C 2 36 NAT MN 0.28 : 0.30 2 37 NAT MN 0.20 : 0.30 2 38 NAT MN 7.00 : 8.00 2 39 MCQ MN D 2
  • 165. 40 MCQ MN C 2 41 NAT MN 100.00 : 110.00 2 42 MCQ MN D 2 43 NAT MN 100.00 : 100.00 2 44 NAT MN 0.55 : 0.75 2 45 NAT MN 60.00 : 70.00 2 46 MCQ MN A 2 47 NAT MN -960.00 : -930.00 2 48 NAT MN 78.00 : 80.00 2 49 NAT MN 43.00 : 44.00 2 50 MCQ MN A 2 51 NAT MN 19500.00 : 20500.00 2 52 NAT MN 0.77 : 0.84 2 53 NAT MN 0.00 : 0.00 2 54 MCQ MN A 2 55 NAT MN 31.00 : 31.00 2
  • 166. Graduate Aptitude Test in Engineering Notations : Question Paper Name: MN: MINING ENGINEERING 1st Feb shift2 Number of Questions: 65 Total Marks: 100.0 Number of Questions: 10 Section Marks: 15.0 Question Number : 1 Question Type : MCQ Options : Question Number : 2 Question Type : MCQ Options : Question Number : 3 Question Type : MCQ Wrong answer for MCQ will result in negative marks, (-1/3) for 1 mark Questions and (-2/3) for 2 marks Questions. Q.1 to Q.5 carry 1 mark each & Q.6 to Q.10 carry 2 marks each.
  • 167. Options : Question Number : 4 Question Type : MCQ Options : Question Number : 5 Question Type : MCQ Options : Question Number : 6 Question Type : MCQ
  • 168. Options : Question Number : 7 Question Type : MCQ Options : Question Number : 8 Question Type : NAT Correct Answer : 280
  • 169. Question Number : 9 Question Type : MCQ Options : Question Number : 10 Question Type : MCQ Options : Number of Questions: 55 Section Marks: 85.0 Question Number : 11 Question Type : MCQ Options : Question Number : 12 Question Type : MCQ Q.11 to Q.35 carry 1 mark each & Q.36 to Q.65 carry 2 marks each.
  • 170. Options : Question Number : 13 Question Type : NAT Correct Answer: 104 to 105 Question Number : 14 Question Type : MCQ Options : Question Number : 15 Question Type : MCQ Options : Question Number : 16 Question Type : MCQ Options :
  • 171. Question Number : 17 Question Type : MCQ Options : Question Number : 18 Question Type : MCQ Options : Question Number : 19 Question Type : MCQ Options : Question Number : 20 Question Type : MCQ
  • 172. Options : Question Number : 21 Question Type : NAT Correct Answer : 4 Question Number : 22 Question Type : MCQ Options : Question Number : 23 Question Type : MCQ Options : Question Number : 24 Question Type : NAT
  • 173. Correct Answer: 7900 to 7920 Question Number : 25 Question Type : MCQ Options : Question Number : 26 Question Type : NAT Correct Answer: 25 to 26 Question Number : 27 Question Type : MCQ Options :
  • 174. Question Number : 28 Question Type : MCQ Options : Question Number : 29 Question Type : MCQ Options : Question Number : 30 Question Type : NAT Correct Answer: 4.2 to 5.0 Question Number : 31 Question Type : NAT
  • 175. Correct Answer : 4.6 to 4.9 Question Number : 32 Question Type : MCQ Options : Question Number : 33 Question Type : NAT Correct Answer : 13.0 to 13.6 Question Number : 34 Question Type : MCQ Options : Question Number : 35 Question Type : NAT Correct Answer: 0.75 to 0.81
  • 176. Question Number : 36 Question Type : MCQ Options : Question Number : 37 Question Type : NAT Correct Answer: 2.48 to 2.52 Question Number : 38 Question Type : MCQ Options : Question Number : 39 Question Type : NAT Correct Answer: 4700 Question Number : 40 Question Type : MCQ
  • 177. Options : Question Number : 41 Question Type : MCQ Options : Question Number : 42 Question Type : NAT Correct Answer : 161 to 165 Question Number : 43 Question Type : MCQ
  • 178. Options : Question Number : 44 Question Type : NAT Correct Answer : 10.2 to 10.8 Question Number : 45 Question Type : NAT Correct Answer : 82 to 86 Question Number : 46 Question Type : NAT
  • 179. Correct Answer : 20 to 21 Question Number : 47 Question Type : NAT Correct Answer : 286 to 293 Question Number : 48 Question Type : NAT Correct Answer : 83 to 87 Question Number : 49 Question Type : MCQ Options :
  • 180. Question Number : 50 Question Type : NAT Correct Answer: 3.8 to 4.2 Question Number : 51 Question Type : NAT Correct Answer : 0.5 to 0.6 Question Number : 52 Question Type : MCQ Options :
  • 181. Question Number : 53 Question Type : NAT Correct Answer : 106 to 113 Question Number : 54 Question Type : NAT Correct Answer : 220 to 228 Question Number : 55 Question Type : NAT
  • 182. Correct Answer: 116 Question Number : 56 Question Type : MCQ Options : Question Number : 57 Question Type : NAT Correct Answer : 1.88 to 1.92 Question Number : 58 Question Type : NAT
  • 183. Correct Answer : 0.8 to 0.85 Question Number : 59 Question Type : MCQ Options : Question Number : 60 Question Type : NAT
  • 184. Correct Answer : 0.6 Question Number : 61 Question Type : MCQ
  • 185. Options : Question Number : 62 Question Type : MCQ Options :
  • 186. Question Number : 63 Question Type : NAT Correct Answer : 19.10 to 19.25 Question Number : 64 Question Type : NAT Correct Answer : 13 to 14 Question Number : 65 Question Type : NAT Correct Answer : 0.055 to 0.065
  • 187. GATE 2014: General Instructions during Examination 1. Total duration of the GATE examination is 180 minutes. 2. The clock will be set at the server. The countdown timer at the top right corner of screen will display the remaining time available for you to complete the examination. When the timer reaches zero, the examination will end by itself. You need not terminate the examination or submit your paper. 3. Any useful data required for your paper can be viewed by clicking on the Useful Common Data button that appears on the screen. 4. Use the scribble pad provided to you for any rough work. Submit the scribble pad at the end of the examination. 5. You are allowed to use a non-programmable type calculator, however, sharing of calculators is not allowed. 6. The Question Palette displayed on the right side of screen will show the status of each question using one of the following symbols: The Marked for Review status for a question simply indicates that you would like to look at that question again. If a question is answered, but marked for review, then the answer will be considered for evaluation unless the status is modified by the candidate. Navigating to a Question : 7. To answer a question, do the following: a. Click on the question number in the Question Palette to go to that question directly. b. Select an answer for a multiple choice type question by clicking on the bubble placed before the 4 choices, namely A, B, C and D. Use the virtual numeric keypad to enter a number as answer for a numerical type question. c. Click on Save & Next to save your answer for the current question and then go to the next question. d. Click on Mark for Review & Next to save your answer for the current question and also mark it for review, and then go to the next question.
  • 188. Caution: Note that your answer for the current question will not be saved, if you navigate to another question directly by clicking on a question number without saving the answer to the previous question. You can view all the questions by clicking on the Question Paper button. This feature is provided, so that if you want you can just see the entire question paper at a glance. Answering a Question : 8. Procedure for answering a multiple choice (MCQ) type question: a. Choose one answer from the 4 options (A,B,C,D) given below the question, click on the bubble placed before the chosen option. b. To deselect your chosen answer, click on the bubble of the chosen option again or click on the Clear Response button. c. To change your chosen answer, click on the bubble of another option. d. To save your answer, you MUST click on the Save & Next button. 9. Procedure for answering a numerical answer type question: a. To enter a number as your answer, use the virtual numerical keypad. b. A fraction (e.g. -0.3 or -.3) can be entered as an answer with or without '0' before the decimal point. As many as four decimal points, e.g. 12.5435 or 0.003 or -932.6711 or 12.82 can be entered. c. To clear your answer, click on the Clear Response button. d. To save your answer, you MUST click on the Save & Next button 10. To mark a question for review, click on the Mark for Review & Next button. If an answer is selected (for MCQ) or entered (for numerical answer type) for a question that is Marked for Review, that answer will be considered in the evaluation unless the status is modified by the candidate. 11. To change your answer to a question that has already been answered, first select that question for answering and then follow the procedure for answering that type of question. 12. Note that ONLY Questions for which answers are saved or marked for review after answering will be considered for evaluation. Choosing a Section : 13. Sections in this question paper are displayed on the top bar of the screen. Questions in a Section can be viewed by clicking on the name of that Section. The Section you are currently viewing will be highlighted. 14. A checkbox is displayed for every optional Section, if any, in the Question Paper. To select the optional Section for answering, click on the checkbox for that Section. 15. If the checkbox for an optional Section is not selected, the Save & Next button and the Mark for Review & Next button will NOT be enabled for that Section. You will
  • 189. only be able to see questions in this Section, but you will not be able to answer questions in the Section. 16. After clicking the Save & Next button for the last question in a Section, you will automatically be taken to the first question of the next Section in sequence. 17. You can move the mouse cursor over the name of a Section to view the answering status for that Section. Changing the Optional Section : 18. After answering the chosen optional Section, partially or completely, you can change the optional Section by selecting the checkbox for a new Section that you want to attempt. A warning message will appear along with a table showing the number of questions answered in each of the previously chosen optional Sections and a checkbox against each of these Sections. Click on a checkbox against a Section that you want to reset and then click on the RESET button. Note that RESETTING a Section will DELETE all the answers for questions in that Section. Hence, if you think that you may want to select this Section again later, you will have to note down your answers for questions in that Section. If you do not want to reset the Section and want to continue answering the previously chosen optional Section, then click on the BACK button. 19. If you deselect the checkbox for an optional Section in the top bar, the following warning message will appear: "Deselecting the checkbox will DELETE all the answers for questions in this Section. Do you want to deselect this Section?” If you want to deselect, click on the RESET button. If you do not want to deselect, click on the BACK button. 20. You can shuffle between different Sections or change the optional Sections any number of times.
  • 190. GATE 2014 Examination MN: Mining Engineering Duration: 180 minutes Maximum Marks: 100 Read the following instructions carefully. 1. To login, enter your Registration Number and password provided to you. Kindly go through the various symbols used in the test and understand their meaning before you start the examination. 2. Once you login and after the start of the examination, you can view all the questions in the question paper, by clicking on the View All Questions button in the screen. 3. This question paper consists of 2 sections, General Aptitude (GA) for 15 marks and the subject specific GATE paper for 85 marks. Both these sections are compulsory. The GA section consists of 10 questions. Question numbers 1 to 5 are of 1-mark each, while question numbers 6 to 10 are of 2-mark each. The subject specific GATE paper section consists of 55 questions, out of which question numbers 1 to 25 are of 1-mark each, while question numbers 26 to 55 are of 2-mark each. 4. Depending upon the GATE paper, there may be useful common data that may be required for answering the questions. If the paper has such useful data, the same can be viewed by clicking on the Useful Common Data button that appears at the top, right hand side of the screen. 5. The computer allotted to you at the examination center runs specialized software that permits only one answer to be selected for multiple-choice questions using a mouse and to enter a suitable number for the numerical answer type questions using the virtual keyboard and mouse. 6. Your answers shall be updated and saved on a server periodically and also at the end of the examination. The examination will stop automatically at the end of 180 minutes. 7. In each paper a candidate can answer a total of 65 questions carrying 100 marks. 8. The question paper may consist of questions of multiple choice type (MCQ) and numerical answer type. 9. Multiple choice type questions will have four choices against A, B, C, D, out of which only ONE is the correct answer. The candidate has to choose the correct answer by clicking on the bubble (⃝) placed before the choice. 10. For numerical answer type questions, each question will have a numerical answer and there will not be any choices. For these questions, the answer should be enteredby using the virtual keyboard that appears on the monitor and the mouse. 11. All questions that are not attempted will result in zero marks. However, wrong answers for multiple choice type questions (MCQ) will result in NEGATIVE marks. For all MCQ questions a wrong answer will result in deduction of⅓ marks for a 1-mark question and ⅔ marks for a 2-mark question. 12. There is NO NEGATIVE MARKING for questions of NUMERICAL ANSWER TYPE. 13. Non-programmable type Calculator is allowed. Charts, graph sheets, and mathematical tables are NOT allowed in the Examination Hall. You must use the Scribble pad provided to you at the examination centre for all your rough work. The Scribble Pad has to be returned at the end of the examination. Declaration by the candidate: “I have read and understood all the above instructions. I have also read and understood clearly the instructions given on the admit card and shall follow the same. I also understand that in case I am found to violate any of these instructions, my candidature is liable to be cancelled. I also confirm that at the start of the examination all the computer hardware allotted to me are in proper working condition”.
  • 191. GATE 2014 SET- 2 General Aptitude -GA GA 1/2 Q. 1 – Q. 5 carry one mark each. Q.1 Choose the most appropriate word from the options given below to complete the following sentence. A person suffering from Alzheimer’s disease short-term memory loss. (A) experienced (B) has experienced (C) is experiencing (D) experiences Q.2 Choose the most appropriate word from the options given below to complete the following sentence. ____________ is the key to their happiness; they are satisfied with what they have. (A) Contentment (B) Ambition (C) Perseverance (D) Hunger Q.3 Which of the following options is the closest in meaning to the sentence below? “As a woman, I have no country.” (A) Women have no country. (B) Women are not citizens of any country. (C) Women’s solidarity knows no national boundaries. (D) Women of all countries have equal legal rights. Q.4 In any given year, the probability of an earthquake greater than Magnitude 6 occurring in the Garhwal Himalayas is 0.04. The average time between successive occurrences of such earthquakes is ____ years. Q.5 The population of a new city is 5 million and is growing at 20% annually. How many years would it take to double at this growth rate? (A) 3-4 years (B) 4-5 years (C) 5-6 years (D) 6-7 years Q. 6 – Q. 10 carry two marks each. Q.6 In a group of four children, Som is younger to Riaz. Shiv is elder to Ansu. Ansu is youngest in the group. Which of the following statements is/are required to find the eldest child in the group? Statements 1. Shiv is younger to Riaz. 2. Shiv is elder to Som. (A) Statement 1by itself determines the eldest child. (B) Statement 2 by itself determines the eldest child. (C) Statements 1 and 2 are both required to determine the eldest child. (D) Statements 1 and 2 are not sufficient to determine the eldest child. G A 0 2 ( G A T E 2 0 1 4 )
  • 192. GATE 2014 SET- 2 General Aptitude -GA GA 2/2 Q.7 Moving into a world of big data will require us to change our thinking about the merits of exactitude. To apply the conventional mindset of measurement to the digital, connected world of the twenty-first century is to miss a crucial point. As mentioned earlier, the obsession with exactness is an artefact of the information-deprived analog era. When data was sparse, every data point was critical, and thus great care was taken to avoid letting any point bias the analysis. From “BIG DATA” Viktor Mayer-Schonberger and Kenneth Cukier The main point of the paragraph is: (A) The twenty-first century is a digital world (B) Big data is obsessed with exactness (C) Exactitude is not critical in dealing with big data (D) Sparse data leads to a bias in the analysis Q.8 The total exports and revenues from the exports of a country are given in the two pie charts below. The pie chart for exports shows the quantity of each item as a percentage of the total quantity of exports. The pie chart for the revenues shows the percentage of the total revenue generated through export of each item. The total quantity of exports of all the items is 5 lakh tonnes and the total revenues are 250 crore rupees. What is the ratio of the revenue generated through export of Item 1 per kilogram to the revenue generated through export of Item 4 per kilogram? (A) 1:2 (B) 2:1 (C) 1:4 (D) 4:1 Q.9 X is 1 km northeast of Y. Y is 1 km southeast of Z. W is 1 km west of Z. P is 1 km south of W. Q is 1 km east of P. What is the distance between X and Q in km? (A) 1 (B) √2 (C) √3 (D) 2 Q.10 10% of the population in a town is HIV+ . A new diagnostic kit for HIV detection is available; this kit correctly identifies HIV+ individuals 95% of the time, and HIV− individuals 89% of the time. A particular patient is tested using this kit and is found to be positive. The probability that the individual is actually positive is _______ END OF THE QUESTION PAPER Item 1 11% Item 2 20% Item 3 19% Item 4 22% Item 5 12% Item 6 16% Exports Item 1 12% Item 2 20% Item 3 23% Item 4 6% Item 5 20% Item 6 19% Revenues G A 0 2 ( G A T E 2 0 1 4 )
  • 193. GATE 2014 MINING – MN Q. 1 – Q. 25 carry one mark each. Q.1 A block of weight 100 kN rests on a floor as shown in the figure. The coefficient of static friction between the block and the floor is 0.5. A force of 45 kN is applied horizontally on the block. The static frictional force in kN is (A) 22.5 (B) 50.0 (C) 55.0 (D) 100.0 Q.2 A spring of constant stiffness k is stretched from point A to point B (displacement u in the figure) by a force F. The potential energy of the spring is expressed by (A) 2 1 2 ku Fu − (B) 2 1 2 ku Fu + (C) ku F − (D) ku F + Q.3 If s σ is the induced stress and i σ is the insitu stress at a point below ground, the ‘stress concentration’ at that point is (A) s i σ σ (B) i s σ σ (C) i s σ σ (D) s i σ σ Q.4 The components of state of stress at a point in x−y plane are given as 5 xx σ = MPa, 10 yy σ = MPa and 2 xy τ = − MPa. The sum of the principal stresses acting on the x−y plane in MPa is _______ Q.5 The angle 50 15′ 25′′ is expressed in hours, minutes, and seconds as (A) 1ℎ 20𝑚𝑚𝑚𝑚𝑚𝑚 1.67𝑠𝑠 (B) 1ℎ 20𝑚𝑚𝑚𝑚𝑚𝑚 16.00𝑠𝑠 (C) 0ℎ 21𝑚𝑚𝑚𝑚𝑚𝑚 1.67𝑠𝑠 (D) 0ℎ 21𝑚𝑚𝑚𝑚𝑚𝑚 16.00𝑠𝑠 Q.6 A circular curve has a radius of 200 m and deflection angle of 650 . The length of the curve in m is (A) 221 (B) 227 (C) 235 (D) 262 Q.7 The weight strength of ANFO of specific gravity 0.8 is 912 kcal/kg. The weight strength of an emulsion explosive of specific gravity 1.2 is 850 kcal/kg. Bulk strength of the emulsion explosive relative to ANFO in percentage is ________________ 100 kN 45 kN Coefficientof static friction = 0.5 A B F u k MN 1/8 M N ( G A T E 2 0 1 4 )
  • 194. GATE 2014 MINING – MN Q.8 In a cut-and-fill stope, the main purpose of back filling is to (A) reduce ore dilution (B) prevent high stress concentrations in far field domain (C) prevent displacement due to dilation of fractured wall rock (D) improve ore rehandling Q.9 Bypass valve in a compressed oxygen type self-contained breathing apparatus is meant to (A) release accumulated nitrogen in the breathing bag (B) release excess pressure in the breathing bag (C) supply oxygen directly to wearer in case pressure reducing valve does not function (D) flush out the apparatus with oxygen on opening the cylinder valve Q.10 Given S is the setting load and Y is the yield load of a hydraulic prop, the correct relationship is (A) S < Y (B) S > Y (C) S = Y (D) S = Y2 Q.11 Solution of the differential equation dy ky dx = follows exponential decay (where k is a constant) for [ ] 0, x∈ ∞ if (A) 0 k > (B) 0 k < (C) 0 k = (D) k e = Q.12 The value of k for which the vectors 2 3 a = i - j and 4 k + b = i j are orthogonal to each other is ___ Q.13 Which one of the following is the most likely mode of slope failure for waste dump (A) Circular (B) Wedge (C) Plane (D) Toppling Q.14 The occurrence of head in a single toss of an unbiased coin is given by a random variable X. The variance of X is _______________ Q.15 The divergence of the vector ( )( ) x y y x = + − + v i j is (A) y x − (B) x y − (C) 2 2 x y − (D) 2 2 y x − Q.16 The 0 lim x x x → is (A) −1 (B) 0 (C) 1 (D) non-existent Q.17 For Indian coal mines, the ‘maximum allowable concentration’ of respirable dust containing 7.5% free silica in mg/m3 is (A) 2.0 (B) 2.2 (C) 2.5 (D) 2.7 MN 2/8 M N ( G A T E 2 0 1 4 )
  • 195. GATE 2014 MINING – MN Q.18 Given 𝑘𝑘 is the thermal conductivity, 𝜌𝜌 is density and 𝑐𝑐 is specific heat of a rock sample, the thermal diffusivity of the rock sample is (A) 𝑘𝑘𝑘𝑘 𝑐𝑐 (B) 𝜌𝜌𝜌𝜌 𝑘𝑘 (C) 𝑘𝑘𝑘𝑘 𝜌𝜌 (D) 𝑘𝑘 𝜌𝜌𝜌𝜌 Q.19 Cyclone, bag filter and scrubber can be used for control of (A) water pollution (B) air pollution (C) soil pollution (D) noise pollution Q.20 A mine waste dump of pH 5.2 can be neutralized by adding (A) urea (B) calcium carbonate (C) sulphuric acid (D) sodium chloride Q.21 A flat coal seam of thickness (t) 3 m is excavated and broken roof rock has completely filled the space created due to extraction as shown in the figure. If the bulking factor of roof rock is 1.2, the caving height (H) in m is ______________ Q.22 A piece of coal sample weighs 10 kg in air and 2 kg when immersed in water. The specific gravity of the coal sample is ___________ Q.23 In a borehole log of 1.2 m in length, recovery of rock cores in cm is given below 20, 8, 15, 8, 8, 4, 3, 9, 10, 1, 5, 10 The RQD in percentage is (A) 29.2 (B) 31.8 (C) 45.8 (D) 50.0 Q.24 An underground coal mine panel produces 520 tonnes per day deploying 220, 200 and 192 persons in three shifts. As per CMR 1957, the minimum quantity of air in m3 /min to be delivered at the last ventilation connection of the panel is ____________ Q.25 In a PERT network, the activities on the critical path are a, b and c. The standard deviations of the durations of these activities are 2, 2 and 1 respectively. The variance of the project duration is (A) 3 (B) 5 (C) 9 (D) 12 H t void space MN 3/8 M N ( G A T E 2 0 1 4 )
  • 196. GATE 2014 MINING – MN Q. 26 – Q. 55 carry two marks each. Q.26 A particle P is in equilibrium as shown in the figure. The magnitude in kN and the orientation θ in degrees of the force F respectively are (A) 52.1, 16.1 (B) 221.2, 23.2 (C) 102.3, 53.4 (D) 180.3, 73.9 Q.27 A distributed load of 4 kN/m acts on a beam of 6 m length supported by a hinge and a roller as shown in the figure. The distance in m of the point of zero shear in the beam from the point A is __ Q.28 A dry rock sample of diameter 50 mm and length 100 mm weighs 300 g. After saturating in brine solution of specific gravity 1.05, its weight increased to 330 g. The porosity of the rock sample in percentage is __________ Q.29 A joint plane of length L and dip δ intersects the toe of a slope as shown in the figure. The weight of the shaded block is W. Uniform water pressure P acts normal to the joint plane. If the cohesion and angle of internal friction of the joint surface are c and φ respectively, then the expression for ‘safety factor’ of the shaded block is (A) ( sin ) tan cos Lc W LP W δ φ δ + − (B) ( cos ) tan sin Lc W LP W δ φ δ + + (C) ( cos ) tan sin Lc W LP W δ φ δ + − (D) ( sin ) tan cos Lc W LP W δ φ δ + + θ F 200 kN 150 kN 30o x y p 6 m 4 kN/m A B P L δ W Joint plane MN 4/8 M N ( G A T E 2 0 1 4 )
  • 197. GATE 2014 MINING – MN Q.30 The lengths and standard errors of three sections AB, BC, and CD of a straight line AD are given below AB = 125.85±0.021 m; BC= 205.72±0.029 m; CD=246.21±0.025 m The standard error in total length AD in m is (A) ±0.0436 (B) ±0.0350 (C) ±0.0250 (D) ±0.0019 Q.31 The bearing of side AB of a regular hexagon ABCDEF is 𝑆𝑆 500 10′ 𝐸𝐸 . If the station C is easterly from the station B, the whole circle bearing of the side BC is (A) 650 15′ 25′′ (B) 690 50′ 25′′ (C) 690 15′ 25′′ (D) 690 50′ 0" Q.32 In a room-and-pillar stope, bench blasting is conducted using ANFO having density of 800 kg/m3 . The specific gravity of rock is 2.5, hole diameter is 100 mm and spacing to burden ratio is 1.3. The charge length of each blast hole is 80% of the hole length. For a desired powder factor of 0.48 kg/tonne, the spacing and burden of the blast pattern in m respectively are (A) 2.0, 2.6 (B) 2.3, 1.8 (C) 5.2, 4.0 (D) 1.3, 1.0 Q.33 Match the following for ore handling operations in an underground metal mine Arrangement Description (P) Drawpoint (I) arrangement that prevents oversized rock to pass (Q) Ore pass (II) a system of vertical or near vertical openings for transferring ore from a stope to a single delivery point (R) Grizzly (III) a place where ore can be loaded and removed (S) Finger raise (IV) a vertical or inclined opening used for transferring ore (A) P-IV, Q-III, R-II, S-I (B) P-III, Q-IV, R-I, S-II (C) P-II, Q-IV, R-I, S-III (D) P-III, Q-I, R-II, S-IV Q.34 The following characteristic curves (P, Q, R, S) pertain to rotary drilling in rock. Title of the curve I: Torque versus RPM II: Rate of penetration versus uniaxial compressive strength of rock III: Rate of penetration versus weight on bit IV: Specific energy versus weight on bit Match the curves with their titles (A) P-III, Q-IV, R-II, S-I (B) P-II, Q-IV, R-I, S-III (C) P-IV, Q-III, R-II, S-I (D) P-I, Q-III, R-II, S-IV P Q R S x y x y x y x y MN 5/8 M N ( G A T E 2 0 1 4 )
  • 198. GATE 2014 MINING – MN Q.35 The height H of a drawpoint in a sublevel caving stope is 3.0 m. If the angle of repose (𝜑𝜑) of broken ore is 350 , the digging depth y of the loader as shown in the figure in m is _______ Q.36 For an explosives company, the probability of producing a defective detonator is 0.02. The probability that a lot of 50 detonators produced by the company contains at most 2 defective detonators is __________ Q.37 The area enclosed by the curves 2 y x = and 3 y x = for [ ] 0, x∈ ∞ is (A) 1/12 (B) 1/6 (C) 1/2 (D) 1 Q.38 The value of a, for which the function below is continuous at x = 1 is 2 2 , 1 ( ) 4 3, 1 x ax x f x x x  + ≤ =  + >  (A) -5 (B) 0 (C) 5 (D) 10 Q.39 The sum of the infinite series 2 3 1 n a ar ar ar ar − + + + + + +  for 1 r < is (A) ( ) 1 a r + (B) ( ) 1 a r − (C) 1 a r + (D) 1 a r − Q.40 A centrifugal pump has a discharge rate of 2000 L of water per min against a total head of 200 m. If the pump efficiency is 75%, the input power to the pump in kW is (A) 87.20 (B) 49.05 (C) 13.33 (D) 7.50 Q.41 A dragline is required to remove 3,00,000 m3 of rock per month on the bank volume basis. Consider the following data for the dragline operation. Effective working hours per month = 450 Bucket fill factor = 0.8 Cycle time = 65 s Swell factor of the rock = 1.25 The minimum bucket capacity of the dragline in m3 is (A) 7.70 (B) 9.63 (C) 12.04 (D) 18.80 4 Ore Waste rock 0 45 2 ϕ − ϕ H y MN 6/8 M N ( G A T E 2 0 1 4 )